I-SAT DSAT Math

You might also like

Download as pdf or txt
Download as pdf or txt
You are on page 1of 217

Telegram: SAT_pdf

Table of Contents

Chapter 1 About the Digital SAT 1

Chapter 2 Overview of the SAT Math section 13

Chapter 3 Algebra 37
Linear equations in one variable 38
Linear equations in two variables 42
Li near functions 53
Systems of two linear equations in two variables 60
Li near inequalities in one or two variables 72

Chapter 4 Advanced Math 81


Equivalent expressions 82
Nonlinear eq nations in are variable and systems ot eq uations in two variables 91
Nonlinear functions 98

Chapter 5 Problem- Solving and Data Analysis 119


Ratios, rates, proportional relationships, and units 120
Percentages 127
One- variable data : distributions and measures of center and spread 133

Two- variable data: models and scatterplots 139


ProbaPi lity and conditional probability 149
Inference from sample statistics and margin of error 155
Evaluating statistical claims: observational studies and experinnents 159

Chapter 6 Geometry and Trigonometry 163


Area and volume 164
Li nes , angles, and triangles 172
Right triangles and trigonometry 130
Circles 1BS

Telegram: SAT_pdf
Chapter 7 Math Test 193
Module 1 194
Module 2 202
Answer Key 210
Answers 211

Telegram: SAT_pdf
ACT/SAT
Books in Test Prep Series

Math Practice Tests Winning Strategies


for the ACT For ACT Essay Writing:
With 15 Sample Prompts

Practice Tests for the Digital SAT Reading and


Digital SAT Writing Practice Questions

Telegram: SAT_pdf
Digital SAT

Telegram: SAT_pdf
The Math Section at a Glance
The table below summarizes the specifications of the types of questions and their distribution in the Math
section.

Content Domain Skill / Knowledge Question Distribution

Algebra Linear equations in one variahle -


13 15 questions ( 35% )
Linear equations in two variables
Linear functions
Systems of two linear equations in two
variables
Linear inequalities in one or two variables

Advanced Math Lquivalent expressions 13-15 questions ( 35% )


Nonlinear equations in one variable and
systems of equations in two variables
Nonlinear functions

Problem-Solving Ratios, raLes, proportional relationships, and 5-7 questions (15%)


and Data Analysis units

Percentages
One- variable data: distributions and
measures of center and spread
Two-variable data : models and scatterplots
Probability and conditional probability
Inference from sample statistics and margin
of error
Evaluating statistical claims: observational
studies and experiments

Ccumciry and Area and volume 5-7 questions (15%)


Trigonometry
Lines, angles, and triangles
Right triangles and trigonometry
Circles

Digital SAT Math Practice Questions 10

Telegram: SAT_pdf
Telegram: SAT_pdf
Introduction
There are 44 questions in the entire Math section and they are divided into 2 modules of equal lengths, one
representing each of the section 's two stages . Hence, each module has 22 questions, of which 30 are operational and
2 are pretest . Only the answers to the operational questions count towards estimating your final Math score. The
pretest questions are only used to collect data that will be used in judging whether such questions can be used in
future tests.

Table 3.1 Digital SAT Math section specifications

Characteristic Math section

Mode of administering the test The Math section is designed according to the multi - stage
adaptive model and administered through two modules that are
timed differently.

Time per module 1st module will Lake 35 minutes io complete,


2 nd module will take 35 minutes to complete.
'Ihc total Lime available for the section is 70 minutes.

Average lime per question tach question will lake 1.59 minutes Lo be completed.

Score reported You will be scored between 200-tfM.


The score fur this section represents half of ihe total score
Question formaL used The question formal is discreet, with four multiple-choice
options.
Informational graphics Informational graphs can include line graphs, tables, and bar
graphs.

Time per section and module


You will he given 70 minutes to complete the Math Section . This indicates that you have a total of 35 minutes
to complete each module. Yon will he automatically taken to the neat module once the time for the first module has
expired. You may be advanced lo a higher or Lower difficulty module based on your performance in the first module,
and you wont be able to go back to the questions in the fust module.

Average time per question


It is estimated that you will have approximately 1.59 minutes per question.

Question format used


The SAT Math section utilizes two distinct question formats; they are the four -option multiple-choice format
and the student-produced response or SPR format. For multiple-choice questions, you will be required to choose a
single best answer known as the keyed response or key. This multiple-choice format accounts for 75 percent of the
Math questions. However, for the SPR , you will do the calculations yourself and enter your correct response in the
test platform. Although you may obtain different answers in SPR questions when you do the calculations yourself ,
you are encouraged to only enter the single, best/ correct answer.

Digital SAT Math Practice Questions 14

Telegram: SAT_pdf
Context topics
About 30 percent of all Math questions have context; that is, the questions are expressed in words using
subject matters or topics that are obtained from social studies, real -world scenarios, or science. In addition to
understanding the context , which you could be familiar with or can easily understand, you will also be required to
use your knowledge of the context and Math skills to answer the questions. It is not expected that you should have
specific knowledge of the topics beforehand.

Word count by question


For those Math questions expressed in words (context ) , most of them consist of 50 words or fewer.

Informational graphics
Some of the Math questions contain informational graphics such as bar graphs, graphs of functions in the xy-
plane, histograms, dot plots, scatter plots, fine graphs, and representations of geometric figures. The informational
graphics are included in the test questions for two main purposes: first, to indicate the importance of such graphics
in Math Lo convey relevant informa lion/ data; second, to Lest your abiliLy lo solve mathematical problems by
locating, interpreting, and utilizing the information displayed in the informational graphics.

Text complexity
AbuuL 70 percent of Math questions don't have ounLexl ( words). So, iL is not possible to measure text eomplexiLy
for those questions. However, tor the remaining 30 percent of the Math questions with context drawn from social
studies, science, and other real-world scenarios, efforts have been made to modify the text . In other words, tbe
linguisLit difficulty has been removed from such context so as Lo make Lhem dear, dined, and simple for students lo
understand.

Domain structure
The Math Sections questions fall under one of the four content domains described below :

• Algebra: You are expected to carefully analyze, properly solve, and create both linear equations and
inequalities. More so* you will have to correctly analyze and solve different types of equations utilizing
different methods.

* Advanced Math: You are required to show that you have the necessary knowledge and skills indicative
of your progress toward advanced Math courses. As a matter of fact, you would need to demonstrate
your ability to properly analyze, solve, interpret , and create appropriate equations that include but are not
restricted to quadratic, polynomial , absolute value, rational , nonlinear, and radical equations. More so, you
must analyze and solve both linear and nonlinear equations in two variables.

* Problem -solving and Data Analysis: You are expected to make use of your understanding and
quantitative reasoning concerning rates , proportional relationships, and ratios to analyze, interpret , and
solve mathematical problems involving one-and-two - variable data.

* Geometry and Trigonometry: You are expected to utilize your problem-solving skills in solving questions
relating to angles, perimeter, triangles, area, volume, circles, and trigonometry.

.
wwW vibrdntpublisbefS- tOftl 15

Telegram: SAT_pdf
NOTE; Each question belongs to a single content domain, and the questions for each module are drawn from
all four content domains. Therefore, in each domain, questions are designed to test some skills/ knowledge
points.

Tips lor the Math Section


You would find these lips helpful when preparing for your Digital SAT Math lest:

* Always read the SAT' Math questions understandably before you start answering them. You don’t want to
waste your limited time providing answers to questions you vaguely understand.

• Pay attention to the choices for each question. Based on your prior knowledge of similar math questions
in high school, you may he able to quickly eliminate some incorrect answers while looking for the most
correct answer in the chokes , This practice will also help you to save time that you can use fur solving
other difficult questions.

• If a math question seems very difficult for you on the first try, do not waste your time thinking about how
you could solve it; move straight to the next question .

• Skip over questions that appear to he too wordy. You could come hack later to attempt them after you have
successfully cumplcLcd the ones wiLh shorter content.

* .
As calculators arc now allowed in the Math section iL will help you arrive at the answers more quickly.
However, yon will still reed to depend on your knowledge of arithmetic to successfully use it. Do not be
overconfident about the benefits of using a calculator during your test because it may give you a wrong
.
answer and a false hope. So it is advisable lhaL you use il responsibly and creatively in order lu make sure
that Lhe answers you are getting arc all cornet L.

* Cl citing a high score on the SAT math LtsL depends on your preparations before Lhe lest. This is why il is
veryr important that you try your hand at several practice math questions before taking the real Digital SAT
Lest. You will be able to identify the most probable answers to some of the related questions you will be
seeing while Laking the real SAT lest itself

* Pay serious attention to numbers, mathematical signs, and expressions . If you make any mistake
calculating with a wrong mathematical sign , for example, you can expect that your final answer will be
wrong.

* In a situation in which you obtained two answers for the same question , you can quickly redo your
calculations and eventually discover which is more correct. Enter ONLY one of the most appropriate
answer for each question .

Digital SAT Math Practice Questions 16

Telegram: SAT_pdf
Algebra
For Algebra questions, you will be required to do some calculations to solve some mathematical problems
relating to linear representations. In this way, you will be making sensible connections between different forms
of linear equations. Your knowledge of high- school algebra lessons/ topics will be very useful here.

Fay attention to the following skill/ knowledge testing points for Algebra questions:

Skilhknowlcdge testing points:

Linear equations in one variable


For example, 2x 4 3 = 8 is a typical one- variable linear equation having JC as the single variable.
x - 2.5
Sample Question

If 4r - + 6) = 3, what is the value of Yl

A) 3

li ) 9
C) 27
D } Si

Key: C
Level: Medium | Domain: ALGEBRA
Skill/Knowledge: Linear equations in ore variable | Testing Point: Solving for JC and using the value to solve for
an exponential term

Key Explanation: Choice C is correct. Using distributive property yields 4x - x - ft = 3.


Adding 6 to both sides of the equation and combining like terms yields 3* = 9,

Dividing both sides of the equation by 3 yields x - 3.

Hence, the value of y is ? = 27,

Detractor Explanations: Choice A is incorrect. This is the value of x. Choice B is incorrect. This is the value of
.V , Choice D is incorrect. This is the value of 34.

wwW,vibrdntpublisbefS-tOftl 17

Telegram: SAT_pdf
Linear equal! ons in two variables
For example, IOJC + 4y = 30 is an example of a linear equation with two variables jr and y.
Hence, if
^12= 3,
x = 30 -

x= u
— 10

Sample Question

Line I , given by the equation ax - + 3 = 0, is perpendicular to line k , given by the equation - ] 4x + by + 5 = 0.


:'
What is the value of -?
h
11
A)
T
-3
JJ )
14

O 4
11
-14
D) —
Kcy: li
Level: Hard| Domain: ALGFRRA
Kiel I I / Knowledge: Linear equations in two variables | Testing Point: Slope of perpendicular lines

Key Explanation: Choice B is correct . Lines Lhat are perpendicular to each other have slopes that are negative
reciproeals of the other 'lhis tan be represented as ml X m 2 - -1. The slope of the first line ean be given by —3
Ihe slope of the second line tan be given by —
14
b
, therefore, - x
'

3
—b
Multiplying both sides of the equation by —M yields —b ——14

Dislraetur Explanations: Choice A, C, and D are incorrect These options may result due calculation or
conceptual errors.

Linear functions
A linear function forms a straight line on the graph.
For the function o( x, fx ) = 3uc - 2
l( x = 4

Then, /*) = 1 4 - 2
Therefone,/ x}=12.

Digital SAT Math Practice Questions 18

Telegram: SAT_pdf
-6-
5
-*
3
f i x] — x-2
2
T
[)
j
- i - 3 -fl -U- 1
L £

-5

If plotted on a graph h ( jc ) will appear like a straight line shown above.


^
Sample Question

lf ( jt )
* — j[ x + 2) + 2 and _/( ) — 4
jc t + 2, whaL is Lhe value of the -intercept
^
A) 2
u) n
C) 8
D ) HI

Key: B
Level? Medium | Domain ; AICFRRA
Skill / Knowledge: Linear Functions|Testing Point: Function Translation

.
) , evaluate g{ jc) = j[ x 4 2 ) 4 2
Key Explanation: Choice B is correct, To determine the y -intercept of

* + 2 ) yields f i x + 2} = 4(i + 2 ) + 2,
^ jc

Solving
^
Using distributive property yields + 2 ) = 4x + 8 + 2 ,

^
Simplifying the equation yields * + 2 ) = 4JC + 10,
Substituting the equivalent expression for x 4 2 ) ing x) yields (JC ) = 4JT 4 10 4 2.
Simplifying the equation yieldsgUr)
^
= 4x + 12 .
^ ^
Therefore, they- intercept is 12,

Detractor Explanations: Choice A is incorrect. This is they- intercept of the function *) . Choices C and D
are incorrect and may be due to miscalculations or conceptual errors. ^
wwwi vibrdntpublisbefS- tOftl 19

Telegram: SAT_pdf
Systems Q1 IWQ linear equations in two variables
Tfyou are given two linear equations lx + y = 8 and 3x + 2y = 12 , you should solve the first equation fory,
-
which will give ynuy = lx + 8. Then, substitute fory in the second equation .

-
You will have, 3* + 2( 2x + fi ) = 12

3tf - 4* + 16 = 12
-Jf = -4

Multiply both sides of the equation by -1


x=4

Sample Question

ly - px + b

Jy - 2x + 9

What is the value ofp for the system of equation above if the system has no solution ?
A} 3

C) -3
D) 2

Kcy: R
level: Medium | Domain : ALGEBRA
Slci ll / Knowfedge: Systems of two linear equations in two variables | Testing Point: Solving for linear systems
that have no solutions

Key Explanation: Choice B is correct . Eor linear systems to have no solutions , the linear equations have to
,

be parallel to each other. Converting 2y = px + 6 to slope-intercept form yields y =


equation is therefore

Converting 3y = 2x + 9 to slope- intercept form yields y =


^ . The slope of the
X +3

jX + 3 The slope of the equation is therefore j .


,

Since the lines must be parallel, their slopes must be the same,

Equating the two slopes yields tj ’


_
2 3
4
Multiplying both sides of the equation by 2 yields P = J '

D istractor Explanations; Choice A and D are incorrect. This option would result to a system of equation with
one solution as the linear equations would have different slopes. Choice C is incorrect. This option would result
in system of perpendicular linear equations and would have one solution .

Digital SAT Math Practice Questions 20

Telegram: SAT_pdf
Linear inequalities in one or two variables
This is an example of a linear inequality equation with one variable, x. ax + b < c where at bt and c are real
numbers.

Hence, if a = 1, b = 4, and c = 10, :r < 10 - —1


x<6

However, for a linear inequality equation in two variables, ax + by < c, where a , b, and c are real numbers, and b
is not equal to 0,

Hence, if a = 2, b = 3, and t = 20, y = l . Jt will be x = 20 - —2


i = 3.5

Sample Question

Curly wants to buy oranges and apples. Fach orange costs ${140 and each apple costs $0.35 . If she can spend no
more than $6.90, what is the maximum number of oranges she can get if she buys at least 2 apples?

A ) 13

li ) 14
C) 15
D) 16

Key: C
Level: Medium | Domain: ALCEBRA
Skill / Knowledge: Linear inequalities in one or two variables | Testing Point; Solving linear inequalities

Key Explanation: Choice C is correct , Since Carly buys al least 2 apples, she ean buy Lhc maximum number of
oranges by buying exactly 2 apples.

Hence, the problem can be represented by 2(0-35) + x(0- 4) 6 ,9 where xis the maximum number of oranges
that Carly can buy.
Simplifying the inequality yields 0,7 + 0.4x 6.9 ,
Subtracting 0,7 to both sides of the inequality yields 0.4* 6.2,
Dividing both sides of the inequality by 0.4 yields r _ 15.5.

The maximum number of oranges would therefore be 15,

Detractor Explanations: Choices A , B, and D are incorrect and may result from a conceptual or calculation
errors.

wwW, vibrdntpublisbefS-tOftl 21

Telegram: SAT_pdf
PLML>AS Rule; 'Ihe PEMDAS Rule is a mathematical rule that helps students to know how to solve a math
question that involves different mathematical signs at the same time in a math question.

P Parentheses , Brackets , and Groupings

E Exponents

C Multiplication/Division
( perform left to right depending on which operation comes first )
,

fi Add ition/Subtra ction oo


( perform left to right depending on which operation comes first)

For example, when you have a math problem with different signs shown above, you will start first with the
paimLhescs, brackets, and groupings, followed by exponents, then you will do the imilliplicaLion/ division, and
finally addition /subLraction. When solving linear equations, functions, and inequalities, ii is veiy imporLanl to
py attention to the mathematical signs in them.

Advanced Math

For Advanced Math , you will be expected to interpret, calculate, correctly solve, and make use of structure,
and create quadratic equations, absolute value, radical equations, exponential , rational , polynomial, and other
related nonlinear equations. More so, you will be required to make appropriaie connections between different
forms of a nonlinear relationship between two variables.

For Advanced Math questions, these are the skill/ knowledge Lesting points:

SkiHs / kjioH'Jedge testing points

Equivalent expressions
Equivalent expressions are defined as expressions that will have the same value, even if they look different if the
same values are used to substitute the variables in the expressions.

Digital SAT Math Practice Questions 22

Telegram: SAT_pdf
For example.
Do you think that 5 x + 2 and 5(x - 1) + 7 are equivalent expressions?
Yes, they ard

NOTE: When solving for equivalent expressions, use the following ideas:
• Discover the coefficient in the expressions ( in a ( hr + c ) = abx + ac, a, bt and c are the coefficients!

• Combine the variables, which could be JC or y

• Solve for the unknown variable, this could he x axy


• Rearrange the final formula

Sample Question

If 2 is equivalent to s ?, what is the value uf ri?


^ /
A) 1
B} 2
C) 3
D} 4

Key: D
Leveli Medium | Domain: ADVANCED MATH
SkilEKnowledge: Equivalent Expressions | Testing Point: Exponential expressions

/J
"

Key Explanation: Choice D is correct . Tf 2i and v " is equivalent to 4', then 2: - -1” . Making the

^i f
-\ 1 3 -)
Hence, 21 = 2 { n .
)
base of 4" to 2 yields 2 "\

Equaling the exponents yields — = 2[ —


2 {n)
1
Simplifying the equation yields —2 = —n .
Multiplying

^ to both sides of the equation yields

Therefore, n is equal to 4,
n=

^ ,

Distractor Explanations: Choices A , B, and C are incorrect. These options may be due to conceptual or
calculation errors.

wwni vibrdntpublisbefS- tOftl 23

Telegram: SAT_pdf
Nonlinear equation ? in one variable and system? Q1 equation? in two variables
When nonlinear equations are plotted on the graphs, they give curves or nonlinear representations on the
coordinate axis, while linear equations give straight lines ( see the diagram below ).

3r
~

z
M 43
zz 2 3 4

Nonlinear equations can have either one variable (* or y) or two variables ( r and y).

lhere are three approaches ilia l can be used Lo solve nonlinear equations:
' *

* By using a graph: After obtaining real numbers for each of Lhe variables, yon can then plot y against x ,

* By Substitution: You tan substitute the variables with Lhe given or calculated numbersL

For example, in these equations, solve for yi

x' - y ^ ti

y= x - 2

To solve this* this equation solves for y, y x - 2
Then, solve for xby substituting in the equation i(? - y = Q
jri - x 4 2 = 0 ( a quadratic equation)
You can use a quadratic formula lo solve for x.

Wb: - 4ac
x — -

2a
» By eliminatiun: You can eliminate each variable from the equation using the real values ( numbers)
obtained from your calculations.

Sample Question

3( x - 3) ( x - 6 ) = (x - 4)(x - 3)
How many solutions does the system below have?

Digital SAT Math Practice Questions 24

Telegram: SAT_pdf
A ) Zero
B ) One
C) Two
D) Infinitely many

Key : C
Level? Hard | Domain? ADVANCED MATH
Skill / Knowledge: Nonlinear equations in one variable and systems of equations in two variables | Testing
Point: Finding the number of solutions for a system of nonlinear equations

Key Explanation: Choice C is correct , lb find the number of solutions to the equation above, use the
distributive property which yields Sfjd - 3* - fix + 1 ft} = ( A* - 4JT - 3JC + 12 ).
Simplifying the equation yields Sx1 - 27 x + 54 = jr2 - 7* + 12
Subtracting JT and 12 and adding lx to both sides of the equation yields 2jd - 2(bc + 42- 0 .
To find the number of solutions in a quadratic equation, use the discriminant b~ - 'hiC . ihe value of a = 2. b -
- 20, and c - 42
- .

Solving the value of discriminant yields ( - 20)^ - 4( 2)( 42 ); 400 - 35b - 64.
Since the discriminant is greater than 0, the number of solutions for the system of equations is two.

Distraclor Explanations: Choices A , B, and D are incurred, these options may be due to conceptual or
calculation errors.

Nonlinear functions

As its name implies, nonlinear functions are not linear, and they are not in the form ofjfjr ) = ax + b

NOTE: If you are not fast at plotting graphs to solve linear equation problems, concentrate only on using
substitution and elimination methods because you don’t have a lot of time to waste struggling with graph-
plotting.

Some examples of nonlinear functions are;


* Quadratic function: fix ) = x*
* Exponential function: fix ) = 2*
Cubic function: (.t) = x* -
*
^
Sample Question

The population of bacteria in a pond is 3 ,030 and the population triples every 32 days. Which of the following
functions best represents the population of the bacteria P(0 in the pond after t daysf

itrnt vibrdntpublisbefS- tOftl 25

Telegram: SAT_pdf
L
,
-
A ) P( t } = 3 030(3) “
B) £(0 = 3, 030(3)
C) £(f ) = 3,030(3)*
D) £(0 = 3,030(3) ”'
Key: A
Level: Medium I Domain: ADVANCED MATH
Skitt/ KnowLcdgc: Nonlinear Functions|Testing Point: Exponential growth function

Key Explanation : Choice A is correct The exponential growth function has a formula of /fx ) = a (1 + TY where
a is the initial amount, { 1 + r) is the growth rate and x is the time interval.

Since the initial number of bacteria in the pond is 3,030 then a = 3,030.

Since the population triples then (1 + r) = 3.

Since the population triples every 32 days then x = —32 .


There lore, the equation for the function is P(f ) = 3,030(3)” .

Distractor Explanations: Choices B and D are incurred and may result from using a wrung lime interval .
Choice C is incorreeL and may residL from using a wrung growth raLe.

Problem- Solving and Data Analysis


In this question domain, yon will be utilizing your quantitative reasoning to solve math questions relating to
rates, units, percentages, and proportional relationships. More so, you will be required tu properly analyze and
curreelly interpret some data distributions and then use that data to identify related probabilities, frequency,
and conditional probabilities. You will also need to use fir models to discover and compare instances of
exponential and linear growth .

You may be asked to do some calculations on the mean, median , and range of different values, including their
standard deviations. You will be expected to have a basic grasp of study design and delect any margin of error
in the data while doing your calculations.

Most of the skill/ knowledge and testing points highlighted below may have been taught in your high school ,
and they will be tested by SAT for your college or career requirements.

Skiil / knowledge testing puints

Ratios , rates, proportional relationships, and mil percentages


The questions in this skill / knowledge will ask you to solve questions using proportional relationship between
quantities, calculating or using a ratio or rate, and or using units, derived units, and unit conversion.

Digital SAT Math Practice Questions 26

Telegram: SAT_pdf
Sample Question

A cube has a volume of 96 cm1. A larger version of this cube is made with twice the side lengths of the original.
What is the volume of the larger cube?
A ) 192
B ) 76R
C) 384
D) 288

Kcy: B
Level; Hard | Domain: PROBLEM SOLVING AND DATA ANALYSIS
Skill/ Knowledge: Ratios, rates, proportional relationships, and units | Testing Point: Ratios in similar figures

Key Explanation: Choice E is correct. The two cubes are similar and therefore the ratio of their side length is
-
given by 12. 'lhe ratio of their volumes will be lh 2' which is equivalent to 1:8,

Therefore, the volume of the larger cube will he 8 x 96 - 76S.

Distraelor Explanations: Choices A , C, and D are incorrect. These Options may result due to calculation or
conceptual errors.

Percentages
Familiarize yourself with how to do simple calculations or unit percentages. Most importantly, memorize the
necessary steps in achieving correct answers while attempting questions on these math topics. So, diligently do
your prat lice by laying your hands on several examples relating to these testing points.

Sample Question

The price of a book at a bookstore is $45- Maria bought the book at a discounted price of $40- What is the value
of the discount she received in percentage?
A ) 5%
B ) 1.11%
C) 11.11%

D) 12.5%

Key: C
Level: Easy | Domain: PROBLEM-SOLVING AND DATA ANALYSIS
Skill / Knowledge; Percentages|Testing Points Discounts and percentages

Discounht
Key Explanation: Choice C is correct , Percentage discount is found by sclOO . The discount is
original price
$45 - $40 = $5 . The original price Ls $45 . —
45
*100 = 11.11%
.

.
mvW ribriintpublisbCfS-tOftl 27

Telegram: SAT_pdf
Di$ tractor Explanations; Choice A is incorrect. This is the value of the discount. Choices B and D are
incorrect . These options are incorrect and may be due to conceptual or calculation errors.

Ore- variable data: distributions and measures ol center and spread

In math, you can do the distribution and measuring of center and spread in data sets by calculating their mean ,
media, range, deviation, etc. Revise the formulas for calculating the mean, median, range, and deviation of data
sets.

Sample Question

Temperatures in City X and City Y were recorded on eight different days. The temperatures were then plotted
on a scallcrplol shown below. What is the range in temperature for City IT

15 -

] ft
£
.
*
-B
£
*
k.
- £ 5

0 to 15
Temperatures In city X

A) 8
R ) 14
C) 10
D) 12

Key, A
J*vel; Medium | Domain ; PROBLEM SOLVING AND DATA ANALYSTS
Sitill/ Knowledger One-variable data; distributions and measures of center and spread | Testing Point: Finding
range using a scatter plot

Key Explanation ; Choice A is correct. The range in temperature in City Ycan be found by the difference
between the maximum and the minimum temperature in City Y . The maximum temperature is 14 and the
minimum temperature is 6. Therefore, the difference is 8.

Digital SAT Math Practice Questions 28

Telegram: SAT_pdf
11Lit rat l or Explanation*; Choice 0 is incorrect . This is the value of the maximum temperature in City Y.
Choice C is the value of the range of temperatures in City A . Choice D is incorrect. This option gives the
maximum temperature in City X .

Two- variable data : models and scatterplots


In math, we use models and scatterplots to understudy and measure the distribution of two-variable data . The
scatterplots can show clearly that a data set has: (i) a positive correlation ; ( ii ) a negative correlation ; or ( iii) no
correlation. Concerning the models, a data set may be o£ (i) a linear model ; (ii) a quadratic model; or ( iii) an
exponential model.

Sample Question

A meteorologist measures the amount of rain in inches on 8 consecutive days. On which days is Lhc average rale
of change greatest ?

-te-

i -**-
S
.5

n
£

I -
0
[ Jays of the week

A ) Day 3 and Day 6


B) Day 6 and Day 7
C) Day 2 and Day 3

D) Day 4 and Day 7

Key: B
Level: Hard | Domain; PROBLEM-SOLVING AND DATA ANALYSTS
Skill / Kuowledge: Two-variabledata: models and scatterplots | Testing Point; Average rate of change oil a
scatter plot

Key Explanation: Choice B is correct. The average rate of change between points is similar to the slope between
26 -10
the points. Using the points (6, 10) and (7, 26), the slope is

7 6
= 16. This is the greatest average rate of

.
wwW vibrdntpublisbefS- tOftl 29

Telegram: SAT_pdf
change in the days on the scatter plot.

Distractor Explanations: Choice A is incorrect. This would result in a negative rate of change. Choices C and
D are incorrect. These options may result due to conceptual or calculation errors.

Probability and conditional probability


For probability questions, you will have to use one-and-two-way tables, area models, and other representations
to find the relative frequency, probabilities, and conditional probabilities.

Sample Question

If a twelve - sided dice numbered from l to 12 is rolled, what is the probability that it lands on a multiple of 3?

A > 4

1
C)
12
1
nj
6

Key: F
Level: Medium | Domain: PROBLEM SOLVING AND DATA ANALYSTS
Slci 11/ Knowledge ; Probability and Conditional Probability | Testin g Point: Finding the probability of an event

Favourable Outcomes
Key Explanation: Choice B is correct , Probability is found by , The multiples of 3 in a
Toted Outcomes
dice of 12 are 3, ft , 9 and 12. The number of favorable outcomes is 4. Tire total number of outcomes is 12.
4 1
Therefore, the probability is — , which is -,
II 3
Dis tractor Explanations: Choices A and D are incorrect. These options may be due to conceptual errors.
Choice C is incorrect. This is the probability of the die landing in any number.

Inference from sample statistics and margin of error

In this case, you will be asked to make inferences by analysing some data sets and using that information to
answer some questions.

Sample Question

A researcher is conducting a study on the weight of students in a particular school. He randomly chooses JO
students and finds that their mean weight is 53 kg. He concluded that the margin of error in relation to the
study is 3,5 kg. Which of the following statements can best be concluded about the weight of the students?

Digital SAT Math Practice Questions 30

Telegram: SAT_pdf
A ) The JO students in the survey all weigh between 49, 5 kg and 56, 5 kg ,
B ) The range weight of the students is 7kg ,
C) It is plausible that the median of the 30 students in the survey lies between 49.5 kg and 56.5 kg .
D) It is plausible that the mean of the 30 students in the survey lies between 49.5 kg and 56.5 kg .

Key : D
Level! Medium | Domain: PROBLEM -SOLVING AND DATA ANALYSIS
Skill / Knowledge: Inference from sample statistics and margin of error | Testing point: Discounts and
percentages

Key Explanation: Choice D is correct. The margin of error implies that the plausible value of the mean would
he 53 kg ± 3.5 kg. This means that the plausible value of the mean lies in the range of 49.5 kg and 56.5 kg. This
statement would iherefurc be true.

Distraelor Explanations: Choices A , B, and C are incurred. These options arc false based on the information
given in the study.

Evaluating statistical claims: observational studios and experiments:


To evaluate statistical claims, you will noL be required to do any calculations. All you need to do is In:
(1) Recognize good and bad sampling methods used in collecting Lhc data Sets.
(li) Make your convincing cor elusion based on the results of the surveys and experiments used in obtaining the
data sets.

Sample Question

A researcher conducted research or different types of tea and it ' effect on the quality of sleep. A random
sample of 30 university s Indents was selected for Lhe study. 10 of the students drank chamomile tea before sleep,
another 10 students had hibiscus tea before sleep and the rest had peppermint tea before bed . 7 of those who
drank chamomile tea, 5 of those who bad hibiscus tea and 1 who bad peppermint tea reported having slept
belter. Which of the conclusions can be drawn from the study?

A ) Peppermint lea doesn't improve the quality of sleep.


B) 39% of those who drank either chamomile, hibiscus, or peppermint lea reported having slept better,
C) Tea is essential to improve your quality of sleep.

D) Hibiscus tea is best to improve the quality of sleep

Key: B
Level: Hard | Domain; PROBLEM-SOLVING AND DATA ANALYSIS
Skills / Knowledge; Evaluating statistical claims: observational studies and experiments|Testing point:
Conclusions from Observational studies

wwW, whrOntpijblishtfri.COrtl 31

Telegram: SAT_pdf
Key Explanations; Choice E is correct , The total number of students from the study who reported having
better sleep is 13 out of the 30 students surveyed . This proportion can be represented as 39% making option B
true.

Distractor Explanations: Choices A, C, and D are incorrect. These statements are false and cannot be
concluded from the survey,

Geometry and Trigonometry


Geometry and Trigonometry questions require you to solve mathematical problems related to area , volume,
length, and scale factors concerning geometric figures. You will also need to use concepts and theories to
identify similarity, congruence, and sufficiency in vertical angles, parallel lines that are cut by a transversal, and
Irianglcs.

You will he asked to solve math problems by utilising the Py ihagurcan theorem, and your knowledge of right
triangle and circle trigonometry. Do not forget the properties of certain types of right triangles. More so, you
will need to use some properties and theorems relating to circles to solve problems. You will have learned all
abuuL these math topics in high school, and SAT will test you on them So as Lo mceL the requirements for your
college admission or future career.

Spend some time to practice any inath questions you can lay your hards on that are related to these skill /
knowledge testing point ?:

Skill / knowledge testing points

Area and volume


You will have io find the area, perimeter, volume, surface area of a geometric figure by using the given
information .

Sample Question

A cone has the same volume as a cube. If the cone has a volume of 729 cm 3, what is the area of one surface on
the cube?
A) 9
R ) 4S6
C) 81
D) 3«

Key: C
Level: Medium | Domain; GEOMETRY AND TRIGONOMETRY
Skill / Knowledge : Area and Volume | Testing Point : Volume of a cube

Digital SAT Math Practice Questions 32

Telegram: SAT_pdf
Key Explanation; Choice C is correct , The volume of the cone is equal to that of the cube and is therefore
729cm\ The volume of the cube is found by s* - 729 .

^
Hence, the value of s = 729 = 9 rm.

The area of one surface of the cube can be found by s' = 91 = 31cm1.

Detractor Explanations: Choice A is incorrect This gives the value of one side of the cube. Choice B is
incorrect. This gives the surface area of a dosed cube with a side length of 9cm. Choice D is incorrect. This
gives the perimeter of one side of the cube.

Lines, angles , and triangles


Do you remember your higji-schoul math lessons on lines, angles , and triangles? Some questions you will meet
in the Digital SAT' math are related to lines, angles, and triangles. Here, yon will solve questions rdaling to the
congruence the and similarity of triangles.

Sample Question

The sum of interior angles in a regular 6-sided figure is an in radians. What is the value a ?

A) 2
B) 4
C) 8
D} 6

Key: B
Level: Hard | Domain: GEOMETRY AND TRIGONOMETRY
Shill / Knowledge: Lines, angles, and triangles|Testing Point:Converting angles to degrees

Key Explanation: Choice B is correct. The sum of interior angles of a regular polygon can be given by 180( n -
2 ) where n is the number of sides , Therefore, the sum of interior angles of a 6-sided figure can be given by 130(6

- 2 ) = 720*. To convert this to radians, multiply by



180
, This yields 720 x —
130
=4 JT. Therefore, a = 4.

Detractor Explanations; Choices A, C, and D are incorrect. These options may result due to conceptual or
calculation errors.

Right triangles and trigonometry

You will need to look at the methods for solving right triangles’ questions, which may include their area,
perimeter, and semi- perimeter. More so, revise the different approaches for doing trigonometry calculations,
estimating their hypotenuse, adjacent, and opposite. Don't forget to also revise sine, cosine, tangent, cosecant,
cotangent, and secant functions.

vibrdntpublisbefS- tOftl 33

Telegram: SAT_pdf
This trigonometry ^ ill help you to solve your trigonometry questions;

TRIGONOMETRY TABLE

0° 30° 45” 60 B 90° 120° 180° 270" 36fl°


sin a 0 1 0 -1 0
t & 4i £
2 2 2 2
cos Ct 1
s 42 i
2
0
2
-1 0 1

2 2
tan « 0 1 1
s -S 0 50 0
s
col a 50
43
~
i 1 0 1 <30 o <c

4i S
sec U 1 2 on -2 1 1
2 4i GO

J5
cnsec a an 2
4i 2 1 2 QO -1 OO

4i 43

Sample Question

Triangles ABE and ACD arc right triangles. The length of Alt - 3cm, UC - 6cm , and AE = 5cm. What is cus
ZCDA ?

C [l

3
A)
5

B)
5

Q
i
D)
<!)
Digital SAT Math Practice Questions B4

Telegram: SAT_pdf
Key: B
Level : Medium | Domain: GEOMETRY AND TRIGONOMETRY
Skills Knowledge: Right triangles and trigonometry | Testing Point: Trigonometry in similar triangles

Key Explanation: Choice B is correct. Since triangle ABE and triangle AC.D are similar triangles, then cos
ZCDA is equivalent to Cos ZAEB.
BE
Using SOHCAHTOA, cos AEB is adjacent/ hypotenuse which is
AE
The length of BE can be found using the Pythagoras theorem (a * + t2 = c* ) which yields 3J + b* =

Simplifying the equation yields b = 25 - 9 = Vlfi = 4 ,

Hence, BE = 4cm.
^
'Ihcrdone, cos ZAEB — cos ZCDA - —45
Distract or Explanations: Choice A is incorrect. This option would give the value of sin CDA. Choices C and D
are incorrect. These may result due to conceptual nr calculation errors.

Circles
Find the arc lengths, sector areas, and angles in circles using properties, definitions, and theorems related to
circles.

Sample Question

The equation of a circle is represented by x1 + ( >x + y1 - % y - 24. Which of the following represents the
coordinates to the center of a cirele if it is translated Lu the left by 3 units?
A ) (- 3, 4)
B > (3, - 4)

C) (0, -4)
D ) (-6, 4)

Key: D
Level: Hard | Domain:GEOMETRY AND TRIGONOMETRY
Skill / Knowledge: Circles | Testing Point: Finding the center of a circle

Key Explanation: Choice D is correct. First, transform the given equation to standard form by completing the
perfect square trinomials.
Adding 9 and 16 to both sides of the equation and grouping the trinomials yields + 6x + 9) + - By + 16) =
24 + 16 + 9,

Factoring the perfect square trinomials and simplifying the right side of the equation yields ( x + 3)3 + { y - 4)2 =
49,

vibrdntpublisbefS-tOftl 35

Telegram: SAT_pdf
Hence, the center of the circle is (-3, 4 ).

-
Therefore, if this coordinate is translated 3 units to the left, then the new coordinate is ( 6, 4).

Detractor Explanations: Choice A is incorrect, 'this is the original center of the circle before translation.
Choice B is incorrect. This option may be due to conceptual or calculation errors. Choice C is incorrect . This
option is due to translating the equation of the circle 3 units to the right.

Digital SAT Math Practice Questions 36

Telegram: SAT_pdf
Algebra
This chapter includes questions on the following topics:
Linear equations in one variable
• Linear equations in two variables
• Linear functions
• Systems of two linear equations in two variables
• Linear inequalities in one or two variables

Telegram: SAT_pdf
I ALGEBRA
I
Irene and Tabitha go out to lunch at a local When 3 times the number m is added to 12 , the
restaurant. Irene's lunch costs Sr and Tabitha's result is 33. Wrhat number results when 15 times
lunch costs $2 more than Irene's . If they split m is added to 9 ?
tile bill evenly and both paid a 20% tip, which
expression below represents the amount of
money that Irene spent!

A ) l .2[c + 1 }

B) 1.2 (2c)
C) 0 , 2 ( t + 1} Three cars have traveled 33ft miles in total. Car
A traveled 3 times as far as Car F, and Car C
D) 1.2 ( 2r + 2)
traveled twice as far as Car 13. How many miles
did Car C travel?

When Jen is 15 years old, her brother’s age can


be calculated using the expression 3JC - 12 where
xis Jens age at that time. What is the difference
1
between Jen’s and her brother 's ages? When — is divided by the reciprocal of a
particular number, ihe result is 16 more than the
number.

What is Lhal number ?

In the jry- plane, the graph of which of the


.
fh I lowing eqii it ions is a line with a slope of -4?

:
A) y= - x
4
BJ y - x -4 What is the value of x in the equation below?

C) y = -4x + 7 3* + 18 = 27
D) y = - 4 + A x

Digital SAT Math Practice Questians 38

Telegram: SAT_pdf
I Linear equations in one variable
I
Tyrone is creating a study schedule for his
midterms for the whole week including
weekends . If he wants to study for at least SO
hours per week, with 10 hours per day over the
weekend , then how many hours does he need to
study per weekday?

A) 3

li ) 4
C) 5
13) 6

wwwt vibrdntpublisbefS- tOftl 39

Telegram: SAT_pdf
I
1. Level: Hard| Skill / Knowledge: Linear equations
ALGEBRA

y = flu + b where m is the slope and b is the y


I-
in one variable|Testing Point: Converting - -
intercept. The slope is 4* so m is 4. The only
English to Algebra with percentages -
answer choice with a slope of 4 is Choice C.

Key Explanation: Choice A is correct. In order Dislractor Explanations: Choices A, li, and D
to determine which expression represents the are incorrect and reflect errors in identifying ihe
situation , first solve the total cost of their foods. slope in a linear equation .
Adding the prices of Irene and Tabitha's lunches
yields c + ( r 4- 2 ) or 2c + 2. If they split the bill
4. Level: Medium|Skill/ Knowledge: Linear
in half , then each of them needs to pay +
2
^ equations in one variable|Testing Point :
or c + 1 for ihe food only. If both of them gave Solving a linear equation in one variable
a 20% tip, then multiply the expression by 1.2.
Therefore, the expression 1.2 (c + l ) represents Key Explanation : The correct answer is 114. An
the total amount of money that Irene spent. equation can be set up and solved with the first

sentience which yields fm + 12 33.

Distractor Explanations: Choice B is incurred


and may result from not considering lhaL Irene Subtracting 12 from both sides of the equation by
and labilha split the bill evenly. Choice C is 12 yields 3m - 21.
incom e L and may result hum calculating only Dividing both sides of the equation by 3 yields m
the tip that each of them gave. Choice D is =7 ,

income L and may result horn calculating the


The second sentence can be converted into the
total amount of money they spent together.
expression 15m + 9. Substituting the value of tn
to the expression yields 15 (7 ) + 9 - 114.
2, Level: Easy | SkilJ / Knuwlcdgc: Linear equations
in one variable | Testing Point: Interpreting word
5. Level: Medium | Skill / Knowlcdge: Linear
problem to solve a linear equation
equations in one variable | Testing Point : Solving
a linear equation
Key Furplanation : The answer is 18- Calculate
the brother’s age by substituting Jen’s age with the
Key F.xplanation : The correct answer is 112, I^et
given expression which yields 3(15 ) - 12 or 33
the variables A, B and C be the distance traveled
years old. Therefore, the difference between Jens
by Car A, Car B, and CarC, respectively. Since
age (15 years old ) and heLr brother’s age is 33 - 15 they traveled a total of 336 miles, then A + B + C
= 18.
= 336. Since Car A traveled 3 times as far as Car
B, then A = 3B. Since Car C traveled twice as far
3, Level; Easy | Skill / Knowledge; Linear equation _
as Car B, then C = 2B Substituting the values of
in one variable | Testing Point: Determine the A and C in terms of B to the first equation yields
slope from the slope- intercept equation of a line 3B + B + 2B = 33b, Combining like terms yields
6B = 336, Dividing both sides of the equation by
Key Explanation : Choice C is correct. The slope- 6 yields B = 56, Since C = 2B, then C = 2(56 ) =
intercept form of a linear equation is written as 112 . Therefore, Car C traveled 112 miles.

Digital SAT Math Practice Questions 40

Telegram: SAT_pdf
I
6.
Linear equations in one variable ( Answers )

Level: Medium | Skill / Knowledge: Linear 8. Level: Medium | Skill / Knowledge: Linear
I
equations in one variable | Testing Point: equations in one variable | Testing Point:
Creating a linear equation and solving Creating and solving an Linear equation in one
variable
Key Explanation: -24 is correct . Let Jtbc the
unknown number. Therefore, the equation will Key Explanation; Choice P is correct. Begin
.be —1 ^ —1 = jf + 16. by creating an equation to represent the total
3 je
number of hours Tyrone needs to study in terms
Simplifying the left side of the equation yields of JC hours per 5 weekdays, if he studies 111 hours
- = JC + 16. per day over the two weekend days:
3 50 - 20 + 5*
Multiplying 3 Lo both sides of Lhc equation yields 30 = Sac
JE = 3x + 48.
x = 6 hours per day
Subtracting 3x from both sides of the equation
yields -lx - 48 . D is tractor Explanations: Choices A, B, and C
are incorrect and reflect errors in interpreting the
Dividing both sides of the equation by -2 yields
value of variables in the word problem.
JE = -24-.

7. Level: Easy | Skill / Knowledge: Linear equations


in one variable | Testing Point: Solving for .r in a
linear equation

Key Explanation: To solve tor the value of JE,


subtract 13 from both sides.

This yields 3JE + 18 - 18 = 27 - l 8 or 3jf = 9.


Divide both sides of the equation by 3,
The result would be x = 3.

wwW, vibrdntpublisbefS- tOftl 41

Telegram: SAT_pdf
I ALGEBRA
I
When x = 1\ , what is the value aiy in the L -2x + y = 3
35 y
function 21* + 21 =
2
? n 2y - x = \2

A} 21 ITT. y - — _v
1
2
+7
, 33
7 TV. y - -2 - 7
JC

132 Which one of the foil owing statements is true


C)
T based on the four linear equations above?
D) 330
A ) Lines T and TT are perpendicular
B) Lines T and TV are parallel
C) Lines I and 111 arc perpendicular
D) Lines U and 111 are parallel


When - jX + 1 10 y , and x - 3, whaL is the value
of}? 1

10
A}
13
A firmer started with ft rows nf corn . He planned
« S lo add 3 nows during the summer months of
18 June, July, and August for ihc nent 7 years. If
C)
1 an equation is written in the formy - mx + b
DJ 5 lo represent the number of rows of com y the
farmer planted * years after his first summer
crop, what is foe value of fr?

1
4
If & = 2 in the equation above, what is the value
ofc?

Digital SAT Math Practice Questions 42

Telegram: SAT_pdf
I Linear equations in two variables
I
Leslie is a technician for a cable company. Each Students in a geography bee score 10 points for
day, she receives multiple locations with cable every correctly answered question and lose 5
boxes that need repair, 'the number of boxes left points for every incorrectly answered question .
that she still needs to Ik at the end of each hour If a student answers 14 questions correctly but
can be estimated with the equation C = 12 - 8 fi misses the final question , what is his final score?
where C is the number of cable boxes left and h
is the number of hours worked lhal day What is
the meaning of the value 12 in this equation ?

A ) Leslie will complete the repairs within 12


hours.

U ) Leslie starts each day with 12 boxes to (k .


Jenna deposits an initial amount of motley into
C) Leslie repairs cable buxes at a rate of 12 an account with simple interest. If the interest
boxes per hour rate is 3% per year and she originally dqrosited
SI ,000,. which equation below represents the
D } Leslie repairs cable boxes at a rate of 12 per
hour.
value ($y ) of her account after t years?

A) y — 30i + 1,000
li )
1,000
= 30T
C) / = 1 ,000 - 30?
D ) / = 1 ,000? - 30

/ = 28 ,2 - 2.5*

What are the coordinates of the / - intercept of


th e above equation ?

A ) (0, 28,2 )
h ) ( 2,5, 0)
What is the value of the x-intencepl of a line that
is parallel to 6x - 3/ = 8 and passes through the
C) (0, 2.5) point (5, 9) ?
D) ( 28.2, 0 )
A) 3

B) 4
C) 2

D) -6

wwwi vibrdntpublisbefS- tOftl 43

Telegram: SAT_pdf
I ALGEBRA
I
Sally sold / fries and b burgers at her snack bar The formula to convert degrees Celsius to
this weekend. If the price for fries is $3.50 and degrees Fahrenheit car be represented by
the price for a burger is S5.00, nod there is no 9
the equation -C + 32 = F , where Cis the
sides tax or tip, which of the following represents
the total amount of money Sally made at her temperature in degrees Celsius and F is the
snack bar this weekend? temperature in degrees Fahrenheit

A} Sjb + 1.5fl Which of the following equations represents C in


terms of F?
13) 5b + 3.50/

C) 3.50b + 5f A , »5 o
-( F - 32)

D) 3.50/h + 5

C) C ^
B) C = ( F -32)

- 5— F + 32
( )

D) C = -( F + 32)
El
1
If / is 2 more than 3 Limes i and r ~ , what ii
the sum of x + y?

1
A)
i
13) 2 9
The formula -C + 32 - F is used to convert the
5
temperature from degrees Celsius to degrees
D) 4 Fahrenheit . If the temperature increases by two
degrees Celsius, what would the equivalent
increase be in degrees Fahrenheit ? Round your
answer to the nearest tenth.

A) 2
B) 1.1
C) 3.6

D) 35,6

Digital SAT Math Practice Questions 44

Telegram: SAT_pdf
I Linear equations in two variables
I
A line passes through points
H) and (4 2 ).

What is the equation of a line perpendicular to


h
If 5 y + ly + 15 y + 42 y = 3 y + 14 p + z, what is the
value of j?
52
this line, which passes through ( 3, -2)? A) y=-

:
:
A ) y = -1
~ -2 , B) y=
yi

u) y =l
~
*+ 2 Q y= -51
]

\
C) y= x ~ l
D) r - S1

4
D} y =- ~ JC + 2


If - - - ft Lind & - 2, what is the value of a + 1 ?
:
A furniture store is having a sale on lamps and A) 1
mgs. Which iunction below represents the total B) 5
cost ofpurchas. e fT) after the 20% discount nf ( f )
C) &
lamps and ( r) mgs if both cost $75 each?
D > 7
A ) T = 0.2 (75ri)
B) T = 0.S (75r + 75 J)
C ) T = 1.2 (75r + 1)

D ) T = 0 - 2 ( J + 75r)

64im
If (ym - n = 24 ivhat is ?
4"

A ) 4J <
B ) &4‘
C) 2“
D ) The value cannot be determined from the
information given.

wwui wftrCrntpijWishers.CUrtt 45

Telegram: SAT_pdf
I ALGEBRA
I
y = 22 + $.9x

One end of a spring is attached to the top of


the tent. When an object of mass x grams is
attached to the other end of the spring, the spring
stretches to a length ofy inches as shown in the
equation above. What is x when y is 69?

A} 47
12.05

C) 23.35
D) 17,69

( 7 v - 4)
If x = , what is the value of y in terms of

x?

A} -
y
4x
b
'T
c) iti
4x + 4
D)
7

Digital SAT Math Practice Questions 46

Telegram: SAT_pdf
I
9.
Linear equations in two variables (Answers)

Level : Easy | Skill / Knowledge: Linear equations 1 L Level: Easy | SkiJl / Kuowledge: Linear equations
I
in two variables | Testing Point: Evaluating a in two variables | Testing Point: Determine
linear equation given an input value whether lines are perpendicular or parallel

Key Explanation: Choice C is correct. Key Explanation: Choice C is correct . I'o


Substituting 21 for jein the equation yields compare the slopes of the four lines given, the
equations should be in the slope -intercept form
21( 21) + 21 = . Simplifying the left side
of a line y = mar + b, where m is the slope and
of the equation by multiplication and then b is the / - intercept of the line. Lines Til and
addition yields 462 = —35 y . Multiplying both TV are already in the slope - intercept form . To
get Line T into si ope- intercept form add 2* to
sides of the equation by 2 gives 924 = 35/.
both sides of the equation to get / = 2JC + 3 . To

Then dividing both sides of the equation hy 35 get Line II into slope-intercept form add x to
132 132 buLh sides of the equation and then divide all
gives
5

y or y =
5 terms of the equation by 2 to get y = —r+ 6 .

Distrae lor Explanations: Choices A and C Parallel lines have the same slope and diilenenl
are incorrect and may result from a conceptual
/-intercepts and perpendicular lines have
or calculation error. Choice B is incorrectand opposite sign reciprocal slopes. Lines 1 and 111
may result frum dividing 162 by 2 instead of are perpendicular because they have opposite
multiplying by 2. sign reciprocal slopes. The slope tor Line T, is -2
and for Line ill iL is — . This makes Choice C
111. Level: Easy | Ski. il / KnowJcdge: Linear equations 2
the correct answer.
in two variables | Testing Point: Solving linear
equations in Iwu variables for a value of one uf Distraclur Explanations: Chorees A , B, and D
the variables are incorrect.They would be selected because
'

of not understanding correctly the relationships


Key Explanation: Choice B is correct - When x = between parallel and perpendicular lines and
3, substitute 3 for x in the equation + 4 = 10 y slope -
3
to gel - ( 3) + 4 = 10 / . Simplifying the left side of
11 Level: Medium | Skill / Knuwkdge: Linear
the equation by multiplication and then addition ,
equations in two variables | Testing Puint:
yields 6 = 10/ , Dividing both sides by 10 gives,
Converting English to algebra and solving a
_ 6 _3 linear equation
^ 10 5
'

Detractor Explanations; Choice A is incorrect Key Explanation; The correct answer is «, The
and may result from inverting the fraction slope-intercept form of a Linear equation is y
— , Choice C is incorrect and may result = mx + b where m is the slope, and b is the y-
intercept or initial / value. The problem states
from adding the values of x and / . Choice D is
incorrect and may result from dividing the value
that y represents the number of nows of corn
of x by the value of / .

.
wwW vibrdntpublisbefS- tOftl 47

Telegram: SAT_pdf
I and that the farmer started with £ rows of corn .
ALGEBRA

Simplifying the equation yields = 28.2 when x =


I
Therefore, b = ft. ^
0. Therefore, the coordinates are ( 0, 2R.2).

Detractor Explana lions: Choices If , C, and D


13. Level: Easy| Skill / Knowledge: Linear equations
are incorrect and reflect errors in interpreting
in two variables|Testing Point: Solve a linear
linear functions.
equation for one variable with the given valne of
the other variable
16 , Level: Easy|Skill/ Knowledge; Linear equations
Key Explanation; The correct answer is 8 . in two variables | Testing Point : Creating and
Substituting 2 for b in the given equation yields using a linear equation in two variables

4

2 - c 0. Subtracting 2 from both sides of the
Key Explanation : 133 is correct. Begin by
equation results in —i c = -2. Multiplying both creating an expression to represent the situation
sides of the equation by -4 yields c = 8 of x, correct answers, andjy, incorrect answers
which yields KIJL - 5 y.

14. Level: Medium | SkilJ / Knuwlcdge: Linear Then substitute the number of CurneClly and
equations in two variables | Testing Point : incorrectly answered questions which yields
Determining the meaning in context of a Lorm in 10(14) - 5( 1 ) = 140 - 5 = 135.
a linear equation
17. Level: Easy|Skill/ Knowledge: Linear equations
Key Explanation ; Choice R is correct. At the in two variables | Testing Puint: Creating a linear

start of the day when h 0, the value of C is
equation from information in quesiiun
.
equal to 12. Therefore Leslie starts each day with
12 cable buses to fix since C is the number of Key Explanation : Choice A is correct. Tn order
boxes left and h is the number of hours worked to create an equation that represents the linear
that day. relationship, first identify the amount that the
deposit increases per year, or the slope of the
Detractor Explanations: Choices A , C, and D
linear equation. Since the interest rate is 3%
are incorrect and reflect errors in interpreting of the original deposit, the interest per year is
functions in word problems, 1,000(0.03) = $30. Hence, the interest after t
years is 30(, Since her initial deposit is $1,000, the
15. Level; Easy|Skill / Knowiedge; Linear equation correct equation isy = 30f + 1,000,
in two variables|Testing Point: Finding the
y- intercept of a linear equation Distractor Explanations: Choices R, C, and
D are incorrect and are likely the result of an
Key Explanation: Choice A is correct. The y- incorrect combination of variables.
intercept is the y-value where x is equal to 0.
Substitute 0 for x and solve y which yields y = 18, Level; Easy|Skill / Knowledge; Linear equations
282 - 2.5(0).
in two variables | Testing Point: Finding the
x-intercept with parallel lines

Digital SAT Math Practice Questions 48

Telegram: SAT_pdf
I Linear equations in two variables (Answers)

Key Explanation: Choice C is correct. The given the number of fries and burgers sold are
I
-
equation of the line is bar 2y = 8. The slope of multiplied by the wrong price.
a line can only he found when the equation is
in the dope-intercept form which would be y
20. Level: Easy | Skill / Knowledge: Linear equations
= 3*- 4. The slope m is equal to 3. The parallel in two variables|Testing Point: Convert English
line would also have a slope of 3 which makes its
to Algebra and solve a linear equation for the
equation y - 3* + b where b is the intercept.
^
Plugging in the puinl (5, 9) to gel the value of
sum of variables given one variable

it yields 9 = 3( 5) + b. Simplifying the equation Key Explanation: Choice D is correct.


yields b = -6. Hence, the equation of the parallel TransUling ihc words into equation form yields
line isy = 3x - 6. The x-intercept is the value of
x wheny is equal to 0. Substituting 0 to y yields

y 3JC + 2 . Subs lituiing the value of x yields
1 7
y = 3 - + 2 or y
J — —
^
3x - 6 0. Simplifying the equation yields r
= = 2. Adding the values of x
.

The x-intercept would he 2.


an dy yields + or - Therefore, x + y - 4
1 7 8
. .

Dislratlor Explanations: Choice A is incorrect. ^


This is Lhe dupe of the lines . Choice B is DistractoT Explanations: Choices A and C are
incorrect. 'Hus is the value of the y-inlercepL of incurred because they give the individual values
the given equation . Choice D is incorrect. This of x and y and not the sum of Lhe two. Choice B
is they- intercept of the parallel line to the given is incorrect and may result from a conceptual or
equation. calculation error,

19. Level: Medium | Skill / Knowledge: Linear 21. Level : Easy|Skill / Knowledge: l inear equations
equations in Iwu variables|Testing Point: in two variables|Testing Point: Solving for one
Construct a linear equation given the variables variable in terms of another
and their coefficients
Key Explanation: Choice B is correct - In
Key Explanation: Choice B is correct. To create order to express the relationship in terms
an expression lhal represents the total amount of of F, rearrange the equation to equal to C.
money Sally made, the variables, which represent Subtracting 32 from both sides of the equation
the number of fries and burgers sold, must be
yields = F - 32 . Multiplying 5 and dividing
matched and multiplied with the corresponding
prices. The number of fries sold (fl, hi multiplied 9 by from both sides of the equation yields
with the price $3,50, and the number of burgers —- —
C = ( F 32 ) .
sold (f >), is multiplied with the price $ 5- The total
amount of money Sally made can be calculated DistractoT Explanations: Choices A, C, and D
by adding the two products. Therefore, the are incorrect and may result from a conceptual

expression must be 5 fr + 3.50/. or calculation error.

Distractor Explanations: Choices A , C, and D


are incorrect because the variables representing

.
wwW vibrdntpublisbefS-tOftl 49

Telegram: SAT_pdf
I
22. Level: Easy|Skill / Knowledge: Linear equations
ALGEBRA

Nest, to find the equation of the line in slope


I
-
in two variables|Testing Point: Solving tor one intercept form ( y = tnx + b ) t find the value
variable in terms of another of y- intercept ( t ). To find ( h ) , substitute the

Key Explanation : Choice C is correct. Modify


-
given point ( 3, 2) and the slope to the

the equation in order to express an increase slope-intercept form equation which yields

|j
in Celsius by two degrees. Therefore, the
new temperature in degrees Fahrenheit is
9
F „ = -(C + 2) + 32 . Gelling the difference
between the new and original equation yields
-2 = -

^ (3) + fi - Simplifying the equation

yields b = 2 Therefore, the equation of the


.

4
perpendicular line is y = - — * + 2 .

F
^
-F
|[C + 2 + 32
= ) C + 32 j . Using Distractor Explanations: Choice A is incorrect
the distributive properly and combining and may result from an error in calculating
like terms to simplify the equation yields they-intercept value. Choice B is incorrect and
— — 9 9
— _ 9
F = (C + 2) - C Factoring out yields — may result from using the slope of the original
line. Choice C is incorrect and may result from
9 9
— ( C + 2 - C) or — (2) . Therefore, the equivalent solving the equation of the original line.
increase in degrees FahrenheiL is —
18
or 3.6.
24. I ^vel: Ffard | Skill /Knowledge: linear equations
Distractor Explanations: Choice A is incorrect in two variables | Testing Point: Converting
and may result from an error in applying the F.nglish to algebraic equation with percentages
distributive property. Choices B and D are
incorrect and most likely feature arithmetic Key Explanation: Choke B is correct . To
errors in calculating the change in degrees determine the equation of the total cost of
Fahrenheit. purchase, begin by determining Lhe total cost
of ( 1) lamps and ( r) rugs before lhe discounLis

^=
applied which is T 75( r + /), Using
23. T veb Medium |Skill / Knowledge: Linear ^
^ distributive property, the equation then becomes
equations in two variables | Testing Point:
TbtfanJiscaast = 7 + 75f When the 2(1% discount
Finding the equation of a line given two points
is applied, the cost is 80% of the original. This
and knowledge of perpendicular line slopes
yields T = 0,8( 75r + 751) ,
Key Explanation : Choice D is correct. In order
Distractor Explanations : Choices A, C, and
to find a line perpendicular to the given line, the
D are incorrect and are most likely the result of
slope of the original line must be determined first
errors in applying discounts to equations with
which yields
variables.

"W =
- H) i -2=
3
j
i ^
sloPe uf d*
25. Level: Easy | Skill / Knowledge: Linear equations
perpendicular line is the negative reciprocal of
in two variables | Testing Point: Solve equation
the slope of the original line Therefore, the slope
4 in two variables
of the perpendicular line is .

Digital SAT Math Practice Questions 50

Telegram: SAT_pdf
I Linear equations in two variables (Answers)

Key Explanation: Choice B is correct. To solve Key Explanation: Choice A Is correct.


I
for y, combine like terms on both sides of the Solving for n in the first equation yields n = 6 m
equation as follows: - 24.
5> + 7y + 15y + 42 y - 3y + 14y + z
69 y = L7y + z
Substituting the value of n in the expression
64lm 64!m
Nest, subtract 1 ly from both sides of the yields -
. :i

- Changing the base in the
4" 4
i

—(£. f
equation to get:
52y = z numerator to 4 yields ( :i = , Since the

Ihcn, divide both sides of the equation by 52 to terms have the same base and are being divided,
gel y as follows: the exponents will he subtracted. This yields
hiw + 24 14
— . This gives Choice B as the correct
52
^
answer. Distractor Explanations: Choices it, C and D .
are incorrect and reflect errors in using exponent
DislracLur Explanations: Choices A , C, and
rules to simplify equations.
D are incorrect and are likely the result of
arithmetic errors.
28. Level: Easy | Skill / Knowledge: linear equations
in two variables|Testing Point: Evaluating a
26. Level: Easy | Skill / Knowledge: Linear equations
linear equation given one variable value
in Lwo variables | Testing Point: Solving linear
equation given value of one variable Key Explanation: Choice B is correct. Substitute

Key Explanation: Choice C is correct.


6? into the equation y 22 + which —
yields 69 = 22 + 3.9x. Solve for x by isolating x.
Substituting 2 for b yields —— = 2. Subtracting 22 from both sides of the equation
yields 47 = 3.9x. Dividing both sides of the
Multiplying 2 to both sides of the equation yields
47
a - 1 - 4. equation by 3.9 yields
3.9
—- X or x = 12.05.

Adding 2 to both sides of the equation yields a + Distractor Explanations: Choices A, C, and D
1 = 6.
are incorrect and may result from a conceptual
or calculation error
Distractor Explanations: Choice A is incorrect
and may result from solving the value of a - 1.
Choke B is incorrect and may result from 29, Level: Easy | Skill / Knowledge: Linear equations
solving the value of a. Choice D is incorrect and in two variables|Testing Point: Solving for one
may result from solving the value of a + 2. variable in terms of another in a linear equation

Key Explanation; Choice D is correct .


27. Level: Hard | Skill / Knowledge: Linear equations
in two variables | Testing Point: Solving a linear Since x = — -
(7 y 4l
. solve fory by isolating the
4
equation for one variable and evaluating an variable,
expression

iml vibrdntpublisbefS-tOftl 51

Telegram: SAT_pdf
I Multiplying both sides of the equation by 4 yields
ALGEBRA
I
4r = 7y - 4.

Adding 4 to both sides of the equation yields Ax


+ A = 7 y or 7 y = 4* + A
Dividing both sides of the equation by 7 yields
4 JC + 4

Dis tractor iispLana Lions: Choices A, U and C


are incorrect and reflect errors in manipulating
algebraic functions.

Digital SAT Math Practice Questions 52

Telegram: SAT_pdf
I Linear functions
I
Daniel owns a website that sells tickets to live
, A car maker operates production plants that saw
-
events. His website charges an initial , one time an increase in production in 2018 to S.fS million
fee for purchasing tickets through his website. units annually from 5 million units in 2013. if
The equation C = 29.99s + 14.99, where s is the the production of the cars increases at a constant
number of tickets to purchase, represents the rate, which function , ft in millions of units, best
total amount of money that first-lime buyers models the production of cars x years after 20131
must pay. What docs the number 29.99 represent
in the equation ? A) f ( x )= ~ x+5

A ) The price per ticket, in dollars.


B } The initial tee that Daniel charges ihr first-
B) m= ~
hx s
+

lime buyers.

C) Mhc Lolal amount lhaL Daniel will earn per


Q
^
f { x )= x+ 5

transaction .
D } The number nf tickets purchased in the
transaction .
D)
^
/ (*) = * + 5

p( n) = 2 n - 2
4( H ) = 2 - p ( n )
A manager is calculating the amount of overtime
pay she owes her employees. The equation y - Ihe functions p and q are defined above. What is
20 x + 25z represents ihe value of an employee s the value of 4( 2) ?
.
weekly paycheck in dollars (y) where x
represents the number of regular hours worked A ) -2
and z represents the number of overtime hours
Ti ) -A
worked. What is the best interpretation for the
value 25 in the equation ? C) 2
D) 0
A ) Total number of hours worked
h ) Number of overtime hours
C) Rate of pay per norm al hour of work
D) Rate of pay per overtime hour of work

.
wwW vibrdntpublisbefS- tOftl 53

Telegram: SAT_pdf
I ALGEBRA
I
25 = 2H + 4, what has the same value as £( 4 )
20
+£ < 6) ?

15 A) gm
B) £fl 0)
10
C) £( 12)
5
D) £( 28)
0
Week ] Week 2 Week 3 Week 4 Week 5
Weeks

Dan collects baseball cards and tracks the


number of baseball cards he has on the graph
above. During which inLerval did the number of
baseball cards increase the fastest?
If j { y ) - —4 y + 6 and fty ) - 10 what is the value
A ) Between 1-2 Weeks
of jr1 ?
B) Between 2-3 Weeks
a
C) Between 3-4 Weeks
D) Between 4-5 Weeks
a
a
C)
5
5
D)
a
An author sells the rights Lo her book to a
publishing house. Tfshe sold the hook tor an
initial fee of if 1,500 and she makes $18-. 50 for
every purchase of the book (*), which expression
represents her total profit from sales of tire hook ?
Die function / is defined byjf *) - 2x - 2 , Which
A } 1 ,500 - 18-5* _
of the following is equal to flx ) + 2?
R ) 18.5* - 1 >500
A ) 2x + 2
C) 18-5* + 1 , 500
B) 2* - 2
D) 18.5 t 1 ,500
C) -2x

D) 2JT

Digital SAT Math Practice Questions 54

Telegram: SAT_pdf
I Linear functions
I
The cost of a bicycle rental on Beautiful Island is Linep is perpendicular to line m. What is the
$20, plus $ 7 per hour for each hour that the bike slope of linep if line m passes through points ( 7,
is rented. If a bike is rented for x hours, which of 3) and ( 2 6 )? .
the following functions of / represents the total
cost in dollars to rent the bike? » -!
A)
B}
j[ x ) = 2Q + 7 x
fix ) = 7 - 20x
> ?i
5

C ) ftx ) =7x 2Q -
13) j{ x ) = -7* + 20
c> ?
°> -
f

If the equation of a line represented by the table



below in the form ax 4 by c, what is Lhe value
of b, where b > 0?
If /fr) - -3
* 4 11 what is Lhe value offl 4)?

y
3 6
6 11
9 16

On a Sunday morning, Courtney sent * text


A) 5
messages each hour tor 4 hours, and Lauren sent
B) 3 y text messages oath hour for 6 hours. Which
3 of the following represents the total number
c> of messages sent by Courtney and Lauren on
i
D) 1 Sunday morning?

A ) lOxy
B) 2Axy
C) 6* 4 4/

D ) 4* 4 6/

vibriintpublisbefS .tOftl 55

Telegram: SAT_pdf
I
30. Level: Easy| Skill / Knowledge: Linear functions
ALGEBRA

Key Explanation: Choice D is correct. The


I
Testing Point: Determining the meaning in function can be written in the slope- intercept
context of coefficients of linear equations form of an equation = HIX + b, where m is
the slope and bis the -intercept . To calculate
Key Explanation: Choice A is correct . Since ^
the slope use the slope formula rtr = ( the change
29.99 is multiplied by the variable f , which In they values divided by the change in the x
represents the number of tickets to purchase, values) , There are 5 million units in 2013, at time
then it is the price per ticket in the equation. equal to zero. Ihis is thus the -intercept and
^
thus b. llhcrc was a 0.6 million unit increase OYCT
Dis tractor Explanations: Choice B is incorrect , 5 years from 2013 to 20 tit. This indicates a slope

—£ _ — million units per 1 year. 'Ihcrefonc, the


lhc initial fee is $14.99 because it is not A .

multiplied by the number of tickets purchased.


Choice C is incorrect because variable C
represents the Lolal amount lhaL Daniel will earn
hmetion is —
/ ( JC) = 25 x + 5.
per transaction. Choice D is inCOrrvCL because Dislractor Explanations; Choices A and B arc
the variable s is the number of tickets purchased . incorrect as they incorrectly show a decrease in
units produced each year. Choice C is incorrect

31. level: Easy | Still/ Know ledge: Linear functions as iL is the residL of a math ciror involving

Testing Point : Interpreting a value in a linear calculating the slope.


function
33. Level: Easy|Still/ Know ledge: Linear functions
Key Explanation : Choice D is correct. In the Testing Fuint: Evaluate linear functions
equation , the variables rardr represent the
number of hours worked by an employee. Since Key Explanation : The correct answer is Choice
the total pay (y) is determined by multiplying x D. Substitutingp( n ) = 2n - 2 tdrp( M ) in the
and y by 20 and 25 respectively, it can be deduced second equation and combining like terms yields
that 2Q and 25 arc the rales of pay. And since 25 q{ n ) - 2 - pin ) = 2 - (2n - 2) or 2 - In + 2 or
is being multiplied by z. it is the rale of pay per 4 - 2n
overtime hour of work. Substituting 2 for n in q( n ) = 4 n - 2 results in
q (2) = 4 - 2(2) = 4 - 4 = 0, which is Choice D.
Distractor Explanations; Choice A iis incorrect
because x + y represents the total number of Distractor Explanations; Choices A , Tt , and C
hours worked. Choice B is incorrect because e are incorrect and reflect errors in knowledge of
represents the number of overtime hours. Choice li nea r functions or si mpl ifi cation ,
C is Incorrect because 20 represents the rate of
pay per normal hour of work.
34. Level: Easy | Skill / Knowledge: Linear functions
Testing Point: Interpret a linear function from a
32. Level; Medium | Skill / Knowledge; Linear graph
functions|Testing Point; Interpret a linear
function and find the slope

Digital SAT Math Practice Questions 56

Telegram: SAT_pdf
I Linear functions (Answers)

Key Explanation: Choice A is correct. Between equal to 2R and solve for n which yields 2n + 4 -
I
weeks 1 and 2, there was the steepest increase in 28.
slope on the graph . Subtracting 4 from both sides of the equation
yields 2 n = 24.
Distractor Explanations: Choices If C, and D
are incorrect and are the result of incorrectly Dividing both sides of the equation by 2 yields H
interpreting the data in the graph or not reading = 12 .
the question carefully enough . Therefore, 12) = g( 4 ) + g(6}.
^
Distractor Explanations: Choice A is incorrect
35. Level: Easy | Skill / Knowledge: Linear functions
and may result from following the pattern 4, 6 ,
Testing Point: Interpreting terms of a linear
8. Choice B is incorrect and may result from
function
adding. 4 and 6 together. Choice D is incorrect
Key Explanation: Choice C is correct Since the and may result from finding the value of jf ( 4) +
author sold the book for Si ,500, she has an initial g( 6 ) but failing Lo find the value of n ,
profit of $1,500. Thereafter, each book sale adds
$13.50 Loher profits. This makes her total profit 37. Level: Easy | Skill / Knowledge: Linear functions
to be f ,5G0 + 18, 5xor 18.5x + 1,500, where x is Testing Point: Evaluating a linear funcLion and
the number of books sold. using negative exponent rule

Distractor Explanations: Choice A is incorrect Key Explanation: Choice B is correct. First, use
and may result if she loses $18.50 per book sale the information provided to determine the value
Choice B is incorrect and may result if she paid ofy, before finding the value of (y} L."

$ 1 ,500 initially, rather than gaining 51,500-


Chuice D is incorrect and may result if she only Equaling the values otfijr ) yields —4 y + 6 = 10-
sold one book, as there is nu variable x in the
Subtracting 6 from both sides of the equation
expression.
yields - y = 4,
4

36. Level ; Medium | Skill / Knowledge: linear Multiplying 4 to both sides of the equation yields
functions | Testing Point; Evaluating a linear 6y = 16,
function given an input value
Dividing both sides of the equation by 6 yields
16 3
Key Explanation: Choice C is correct , JJ ( 4)
' “3 -
means to substitute 4 for H in the equation gift )
1 3
- I n + 4 , and likewise for g( 6 ) . Therefore, the
value ofg( 4) + gfG ) = [2 ( 4) + 4] + 12(6} + 4] ,
Therefore, y 1
=
r*
«
Simplifying the equation yields g( 4) + £ >) = [8 + Distractor Explanations: Choice A is Incorrect
4J + 112 + 4] = 12 + 16 = 23, and may result from solving the value of y.
Choices C and D are incorrect and may result
The question asks which has the same value as
from a conceptual or calculation error.
28, To determine the answer, set up the equation

wwW, vibrdntpublisbefS- tOftl 57

Telegram: SAT_pdf
I
38. Level: Easy| Ski 11/ Knowledge: Linear functions
ALGEBRA

Plugging in point [3 , 6) to the equation of the


I
Testing Point: Evaluating a linear function

Key Explanation: Choice D is correct. Since /( jc)


- 2x - 2, then the expression !*) + 2 = 2x - 2 +
2 = 2x . ^
^
line yields 6 = (3) + c . Simplifying the equation

yields 6 = 5 + c. Subtracting 5 from both sides


of the equation yields 1 = c or c = 1 . Hence,


the equation of the line is y = x +1 . Rewrite
ihe equation of the line in general form by
Distractor Explanations: Choice A is incorrect
and may result from solving /fcj + 4. Choice B
is incorrect and may result from not adding 2
to the equation . Choice C is incorrect and may
result from solving -Jfx) - 2.
^^
multiplying 3 which yields 3 y = x + 1 or
J
3 y = 5 x + 3 . Subtracting 5x from both sides of
the equation yields 3y - 5x - 3. Since b is Lhc
coefficient of y, b = 3-

Distractor Explanations: Choice A is incorrect


39. level: Easy|.Skill / Knowledge: Linear functions
This is the negative value of a. Choice C is
Testing Point: Creating a linear function from incorrect Ihis is Lhc slope of Lhc line. Choice D
information in question
is incorrect Ihis is the value of the y-intercept of
the line.
Key Explanation: Choice A is correct. SlarL
with the initial £ 2fl to rent the hike and add the
additional tee per hour. Since the rate is £7 per 4 ] . I ^vel: Easy |£kill / Knoivfedge: l inear functions

hour, the additional foe for renting a hike for x Testing Point: binding the perpendicular slope
hours is expressed as $7 xx = 7 x. Therefore, the of a line given Lwo points
hinction is defined by /tx) _ — 20 + l x
.
Key Explanation : Choice C is correct - The
Distractor Explanations: Choices B, C, and
slope of line m given can he found hy Xt
-y
.
D arc incorrect and are the result of errors in X1 ~ X!

creating linear equations.



Substituting the given points yields - - or

10. level: Easy|Skill/ Knowledge: Linear functions


, Aslope ofline perpendicular to this can be
Testing Puint : Finding the equation of a line Found by the negative inverse of this slope. The
slope of line p would therefore be .

^
from data

Key F.Tplanation ; Choice B is correct . To find Distractor Explanations: Choice A is incorrect.


the equation of the line, we are required to find This is the slope of a line m or a line parallel to
the slope of the line. Using (3, 6) and (6, 11) we m , Choice B is incorrect and may result from not
, , 11 - 6 5 getting the inverse of the slope ofline m. Choice
= --
can find it by

6 3 3
D is incorrect and may result from not getting
Substituting the slope to the equation of the line the negative value of the slope of line m.
in slope- intercept form yields y = x + C . -

Digital SAT Math Practice Questions sa

Telegram: SAT_pdf
I Linear functions (Answers)

42. Level: Easy | Skill / Knowledge: Linear functions


I
Testing Point: Using the value of x to Hndytjc)

Key Explanation: Substitutexfor 4 in the given


equation .

-
This yields j( 4 ) = 3( 4 ) + 14. Simplifying the
equation yields /(4) = 2.

43. Level: Easy | Skill / Knowledge: Linear functions


Testing Point: Creating a linear expression from
data given

Key Explanation: ChoiceI> is correct. Ihe


number of IcxL messages Courtney sent on
Sunday morning was 4 x . Iho number of Lexl
messages Lauren sent on Sunday morning was
fry. To find the total amount, find the sum of
Courtney's and Lauren's number of text which is
4x + 6y.

DistracLur Explanations: Choice A is mcorrocL


and may result from a conceptual error. Choice
B is incorrect and may resulL from gelling the
product of Cuurtney’s and Laurens number of
lexis. Choice C is incorrect and may result from
swapping the respective variables for Courtney's
and Tauren's number of te*ts-

itmt!vibrdntpublisbefS-tOftl 59

Telegram: SAT_pdf
I ALGEBRA
I
- jr
-
+ 2 y = it 6a - 4b = 24

What is the solution


-
4x y = 25

to the system of
-a + 2 b = — S

If [a, b) is the solution to the system of equations


equations above? above, whaL is the value of a + f >?

A} ( -1, 6 ) A) 1

B) (6, -1) B) -6

Q (1, -6) ) 5

D) (6, -6) 2
O)
3

A farmer’s market is selling spots for their nest 3* + y - 9


event. 'Ihey have 64 total square feel available
tior rent, with two options in size: 6 square feet
-x + = 21
and 4 square feet. If the market rented out a total
Tn the system of equations above, what is the
of M spots, which system of equations below
value of x + y?
represents the relationship between the number
of 6 square feet spots sold (s ) and 4 square feet
spots sold (/)?

A ) 6 s + 4 f = 14, s + f — 64
B) 6/ + 4J = 64, s - f = 14
C) 6s + 4/ = 64, s + f - 14 -* + 2y = 3.1
D) 4s + 6/ = 14, sf = 64 122
5x - 3 y =
5
The system of equations above can be solved by
.
solution ( jr y). What is the value of 2x - y?

A) -3.5
B) 2.3

a 7

D) -9.J

Digital SAT Math Practice QuCitiOns 60

Telegram: SAT_pdf
I Systems of two linear equations in two variables
I
A group of 100 students went on a trip for a chair - -
In the ly plane, the lines 4 x 2y = 6 and y = nil
competition and lodged in 20 hotel suites that + b are perpendicular. What is the value of m ?
either held 4 or 6 students. If all of the rooms
were filled to capacity, what is the difference
between the number of 4 and fi person suites?

In the xy- plane, Lhe line -9* + 'by - 81 intersects


they-aiis aty = b. What is the value oft ?

2 x - 4y = -17 A) 3
27
- x + Sy — Y E) 9
C) 27
What is Lhe suluLion to the system of equations
above ? D ) -9

A ) ( - 5, 1.5)
B } (15, 5 )
C) (3, -3)
13} (1.3, 3)
2x - y = -$
-X + 4 y - 23
What is the solution ( JT, y ) to the system of
equations above?

A ) (-1, 6 )
-3x + 4 y =s m (6 , - D
rx - &y = -2&
c) (1 , -6 )
What values of ( r, s) mate the system of 13 } (6, -6 )
equations above have infinitely many solutions?

A ) (6, 14)
B) (14.6)
C) (3, 4 )

D) (9, 5)

vibrdntpublisbefS- tOftl
itmt! 61

Telegram: SAT_pdf
I ALGEBRA
I
IfO*, ) are solutions to the system below, what is A test has 22 questions in which there are
^ -
the value of jr y? m Multiple Choice Questions and s Student
Produced Responses. The Multiple Choice
2
y=~
r Questions are 3 points each and Student
Produced response questions are 2 points each .
-
5 x 2y = -n If the maximum score on a test is 61 points, how
many MCQs arc on the test ?
A} 2
B ) -3 A) 5

C) -6 3) 17
D) -5 C) 51
D) It)

6 x - lfty = 5
2ix - 6 y = 53
Tfthe equation above is one of the equations in a
linear system with infinite solutions, which of the 8x - $y = -12
following equations is a pari of that system?
For what values of s does the system of equations
above have no real solutions?
A ) - 12JC + 20 y = - 10
B) 12x + 10y = 10
C) l 2x - 20y = 20
D) Mtx + 2i)y = - M )

Digital SAT Math Practice Questions 62

Telegram: SAT_pdf
I Systems of two linear equations in two variables
I
-
dx = ly + 4
4JC + 14y = e
II I
In the system of equations above, for what value
of d is the system true for all real numbers?

III IV

9x + y = -\9
—4 x - Sv — —2
7
Jerry is buying cups and napkins for a party
which costs S3 and $2 each, respectively. If he
For the system of equations above, in which buys twice as many napkins as Cups, how many
quadrant dues the solution lie? cups did he buy if he spent $105 in total ? Note:
Disregard tax in your answer.
A) I

U ) II
C) Ill
D) IV

Jennifer is putting lugclher panels of speakers for


a conference. There are two panels, one that is
6(1 minutes and another ore that is 12( 1 minutes.
Jennifer is offering speakers on Lhe 60-minute
panel 550 for their time, and the speakers on the
JE + by =17 120-minute panel $110 for their time. If Jennifer
5 budgets 5900 for 12 total speakers, which of
aac - 4y = c the following systems of equations represents
the number of 60-minute panelists ( x ) and 90-
If the system of equations above has infinitely mimile panelists (y)?
many solutions, what is the value of a + b + c, if
b = 2? A ) * + y = ]2, 50* + 110y =
B) x + y - 900, Sfl* + llGy = 12
A ) -33,6
B}
S C) .
xy = 12 50y + 110JC = 900
5 D > * - y = 900, 50* + 110y = 12
C) -34
16
D)
5

vibrdntpublisbefS-tOftl 63

Telegram: SAT_pdf
I ALGEBRA
I
Is (25, 11 ) the solution to the following system of According to the preceding system of equations,
equations? what is the value ofy? ( round the answer to two
decimal places)
y + x = 36
2y + Sx = HO y - 2x - -6
2y - x = 5
A ) Yes, (25, 11) is the solution because it
satisfies both equations.
H ) No ( 25, 11 ) is not the solution because it
does not satisfy the first equation .

C) No, ( 25, 11} is not the solution because it


does not satisfy the second equation.
If the system of equations below has no solution,
D) Mure information is needed in order to
whaL is Lhc value off ?
answer the question .

y = 3( jr - 5 ) -2( -5 + r)
y = p( 2 - x ) + fa

-le +/ — 2
e -f=3

If the puinl ( e, f ) satisfies the system of equations


above, what is the value of e?

Quentin’s sales: T Sb + 48
^
Alices sales: 7 = 5.5b + 159
^
Quentin and Alice sell bagels before school. The
equations above represent the number of bagels
(6) they sell oil a given day. How many bagels did
they sell on a given day if Alice made twice as
much money as Quentin?

Digital SAT Math Practice Questions 64

Telegram: SAT_pdf
I Systems of Iwo linear equations in two variables ( Answers )

44. Level: Easy.|SkiJJ / Knowledge: Systems of two variable with the wrong area or incorrectly add
I
linear equations in two variables | Testing Point: or multiply the variables.
Finding the correct value by elimination
46. Level: Easy | Skill / Knowledge: Systems of two
Key Explanation: Choice E is correct. Use an
linear equations in two variables | Testing Point:
elimination method to salvo for the value of *
Solving a system of two linear equations
andy. To eliminate , first, multiply the bottom
^
equation by 2 which yields ftx - 2y = 50. Adding
Key Exp Jana lion : Choice A is correct. The
the two equations yields 7 x = 42. Dividing
system of equations can be solved by either
both sides of the equation by 7 yields x = 6. By
substitution or elimination, but the standard
plugging x = 6 into the first equation yields
form of the equation indicates that elimination
-{ 6) + 2y = -I!. Adding 6 and dividing both sides
is best. Multiplying 2 to both sides of the second
—ft + 6
of the equation by 2 yields y ——
= - or y = -l . —
equation yields -2a + 4b -1 ft. Adding the two
Therefore, the solution to the systems of equation equations yields 6a - 4h - 2a + 4b - 24 -16.
is -1). Combining like terms yields 4a - ft. Dividing
both sides of the equation by 4 yields a - 2.
Distractor Explanations: Choice A is incorrect Substituting the value of a tn the original second
and may result from interchanging the values —
equation yields - ( 2) + 2b -H. Adding 2 to both
ufjc and v. Chuiec C is incorrect and may resulL sides of Lhe equation yields 2i? - -6. Dividing
from interchanging the signs of JC and y. Choice buLh sides of the equation by 2 yields b = -3.
D is incorrect and may result from a conceptual Therefore, Lhe solution Lo the syslem of equations
or calculation error. is (2, -3), Adding the values of a and b yields 2 -
3 -1.
=
45. Level: Easy | Skill / Knowledge: Systems of two Distractor Explanations: Choice R is incorrect
linear equations in Lwo variables | Testing Point : and may result from calculating the product of
Creating a system of two linear equations from a and h Choice C is incorrect and may result
data in the problem from calculating the difference between a and
b. Choice D is incorrect and may res alt from
Key Explanation; Choice C is correct. Tn order
calculating the quotient of a and b,
to create equations that represent the data from
the market, the variables s and / must be matched
and multiplied with their correct totals and 47, Level : Medium | Skill / Knpwledge: Systems of
constants. If the total number of spots sold is 14, two linear equations in two variables | Testing
then the first equation will be r + / = 14. If the Point ; Creating and using a system of two linear
total rented area is 64, then the second equation equations
will be 6s + 4 f = 64,
Key Explanation; Use the elimination method to
Distractor Explanations: Choices A , B, and solve for x arid y. Multiplying 3 to both sides of
D are incorrect because they match the wrong the second equation yields H -x + 5y ) = 21 (3),
Using distributive property yields -3x + I 5y = 63,

wwwi vibrdntpublisbefS-tOftl 65

Telegram: SAT_pdf
I Adding the first and the second equation yields
ALGEBRA

Key Explanation: 0 is correct. Let JC represent


I
3JC + y - 3JT + 15y = 9 + 63 Combining like
. the number of 4 person suites and let /
terms yields 16/ = 72. Dividing both sides of the represent the number of 6 person suites. Since
the number of hotel suites is 20, x + / = 20.

equation by 16 yields / = = 4.5 .
ki Since them arc 4 people in each JC suite and 6
lb determine JC, substitute 4.5 for / in the first people in each / suite, 4x + 6 y - 100. Solving
equation which yields 3JC + 4.5 = 9. Subtracting for x in the first equation; JC + y = 20, results
4.5 from both sides of the equation yields 3JC = in JC = 20 - y.Substituting this value for JC into
4.5. Dividing 4 from both sides of the equation the second equation results in 4 ( 20 - /} + 6/
yields JC = 1.5. = 100. Multiplying the 4 by ( 20 - / ) results in
Therefore, the sum of x and / is JC + / = 1.5 + 4.5 -
SO - 4/ + 6/ 100. Combining like terms gets
SO + 2/ = 100. Subtracting SO from both sides
= 6.
-
of the equation results in 2y 20- Therefore /

43. level: Medium | Skill / Knnwledge: Systems nf



= 10. Since / 10, solving tor x by substituting
the value for y into the first equation JC + / = 20,
two linear equations in two variables | Testing
results in JC +10 - 20, or * = 10.
Point: Solving a system of two linear equations
The difference between the JT and y values is 10 -
Key Explanation: Start by solving the system 10 = 0.
of equations using the elimination method .
Multiplying the first equation by 5 yields -5x
50. Level: Medium|Sldll/Knowledge; Systems of

+ L0y 40.5. Adding the firsL and the second two linear equations in two variables | Testing
equaLion yields -5x + 10/ + 5JC - 3/ — 40.5 -
5
Point: Sulving a system uf two linear equations
Combining like terms yields 7 y = 16.1. Dividing
7 from both sides of the equation yields y = 2.3 . Key Explanation : Choice D is correct.
Eliminate one of the variables. Multiplying 2 lu
Then, substitute this value into the original first
27
equation and solve for X . This yields -JC + 2( 2.3) = the 2nd equation yields 2( ~x + 3/) = 2 — or
3.1 or JC = -3.5. -2x + 6y = 27, '

Therefore, 2JC - / = 2(-3,5) + 2.3 = -9,3, Adding the two equations yields 2r - 4y - 2i +
6/ = -17 + 27.
Dis tractor Explanations: Choices A, B , and
Combining like terms yields 2/ = 10.
C are incorrect and are most likely the result
of arithmetic errors in solving the system of Dividing both sides of the equation by 2 yields y
equations = 5.
Substitute this value for / in one of the original
49. level: Medium | Skill / Knowledge: Systems of
equations and solve for JC.

two linear equations in two variables | Testing Substituting 5 to the 1st equation yields 2x - 4(5)
Point: Create and use a system of two linear = -17 or 2x - 20 = -17.
equations Adding 20 to both sides of the equation yields 2JC
= 3.

Digital SAT Math Practice Questions 66

Telegram: SAT_pdf
I Systems of Iwo linear equations in two variables ( Answers )

Dividing both sides of the equation by 2 yields Key Explanation: To find the slope of 4JT - 2y -
I
6, express the linear equation in slope- intercept
* = -2 = 1.5.
form . Subtracting 4x from both sides of the
Therefore, the solution to the system of equations equation yields -2 y - -4x + 6 . Dividing both
is ( 1-5, 5). sides of the equation by -2 yields y = 2 x - 3. lhe
slope is 2, so A perpendicular line would have a
Distractor Explanations: Choices A , B, and C negative reciprocal slope. Change the sign from
arc incorrect and may result from a conceptual negative to positive and hip the fraction. It will
or calculation error.
yield m = —2 or -0.5.
51. Level; Medium | Skill / Knowledge: Systems of
53. Level: Easy | Skill / Knowledge; Systems of two
two linear equations in two variables|Testing
linear equations in two variables | Testing Point:
Point; Solving a system of two linear equations
Striving a system of two linear equations
with infiniLe solutions
Key Explanation: Choice C is correct . Rewrite
Key Explanation: Choice A is correct. Systems
the linear equation - 9x + 3y = SI in slnpe-
of equations that have infinitely many solutions
inLcTeepI form. Adding 9x on both sides of the
are essentially the same line, .so the constants
equation yields 2y - 9x + 3 L . Dividing both Sides
and the coefficients must be proportional to one
another. Using the coefficients of Lhe y-values, —
of the equation by 3 yields y 3x + 27. Since the

the proportion is 1: -2. 'therefore, the coefficients



slope-inlereepl form is y mx + b and & is Lhe y —
inlereepl, b - 27.
of the x and the tunsLiinls have also Lhe same
ratio. Distractor Explanations; Choices A, B, and D
Multiplying the coefficient of x in the 1 st are incorrect and reflect errors in interpreting
equation to get r yields r = -3(-2) or r = 6, linear equations.
Dividing the constant in the 2nd equation to gel s
28

54. Level ; Easy | Skill / Knnwledge: Systems of
yields 5 = rs = 14.
-2
two linear equations in two variables | Testing
lhe solution is therefore (6, 14).
Point; Solving a system of linear equations by
elimination or substitution
DistracLur Explanations: Choice B is incorrect
and may result from swapping the values of s and
Key Explanation: Choice A is correct. To
r. Choices C and D are incorrect and may result
eliminate one variable, first, multiply the 2nd
from a conceptual or calculation error.
equation by 2 so that the coefficients of x have
opposite signs which yield -2* + 8v - 50, Adding
52. Level: Medium | Skill / Knowledge: Systems of the two equations together yields 2x - y - 2 x +
two linear equations in two variables|Testing 8y = -8 + 50 or 7 y = 42, Dividing both sides of
Point; Using a system of two linear equations the equation by 7 yieldsy = 6, Plugging y = 6 into
with perpendicular lines the 1 st equation yields 2 x - ( 6) = -8 , Adding
6 to both sides of the equation yields 2x = -2 ,

vibrdntpublisbefS- tOftl 67

Telegram: SAT_pdf
I Dividing both sides of the equation by 2 yields
ALGEBRA

equations must be the same line written


I
v- 1 Therefore, the solution to the system of differently. This means that the slope andy-
equations is (-1 , 6). intercepts of both lines are similar. Multiplying
the given equation by -2 yields - 12* + 20y = - 10
Distractor Explanalions: Choice B is incorrect which is Option A.
and may result from swapping the values of
x andy. Choice C is incorrect and may result Distractor Explanations: Choices B and D are
from swapping the signs of jr and y. Choice D is incorrect .The systems would have one solution
incorrect and may result from a conceptual or as the linear equations would have different
calculation error. slopes. ChoiceC is incorrect. The system would
have no solution as the lines would he parallel .

55. level: Medium | Skill / Knowledge: Systems of


two linear equations in two variables | Testing 57. Level: Easy | Skill / Knowledge: Systems of two
Point: Solving for linear systems simultaneously linear equations in two variables | Testing Point:
Creating and solving for a system of linear
Key Explanation: Choice D is correct. Solve the equations
system of equations using substitution.
Key Explanation: Choice B is corrctL. 'There are
Substituting the 1 st equation to the 2nd equation
a total of 22 questions m + s
yields 3*- 2 [M — - 13. Using the distributive
m + S = 22
law and grouping like terms to solve for * would 3 m + 2s = 61
yield —
3
-13, Multiplying 3 and dividing Solve for the system of equations simultaneously.
Multiply the 1st equation by 2 which would
both sides of the equation by 13 yields x = -3.
bubs Lilu Ling the value of * in the 1st equation to result in 2m + 2s =44. Solve by subtracting the
two equations which yields m = 17.Substituting
find the value ofy yields y — ——2 (—3) or y = 2 .
the value of m to the 1st equation yields s = 5.
Substituting the values of x and y to the Hence, them are 17 Multiple choice questions
expressionx - y yields -3 - 2 = -5. and 5 Student Produced Response questions.

Distractor Explanations; Choice A ts incorrect Distractor Explanations; Choice A is incorrect.


This is the value ofy. Choice B is incorrect. This This is the number of the Student Produced
is the value of *. Choice C is incorrect. This is the Responses questions. Choke C is incorrect.
product of * and y. This is the value of the total points that could be
gained from Multiple Choice Questions. Choice
56. Level; Hard |Skill / Knowledge; Systems of two D is incorrect. This is the value of the total points
linear equations in two variables|Testing Point; that could be gained from Student Produced
Solving linear systems with infinite solutions Responses.

Key Explanation: Choice A is correct. For


a system to have infinite solutions, the linear

Digital SAT Math Practice Questions ea

Telegram: SAT_pdf
I Systems of Iwo linear equations in two variables ( Answers )

58. Level: Hard | Skill / Knowledge: Systems of two -


I
yields x = 2. Substituting x to the original first
linear equations in two variables | Testing Point: - -
equation yields 9( 2 ) + y = 19. Simplifying
Comparing linear equations in a system with no -
the equation yields y = 1. Since both J: and y
real solutions values are negative, the solution lies on the third
quadrant.
Key Explanation: Uhc correct answer is 2 .
A system of equations with no solution on a Distractor Explanations: Choices A, B , and
coordinate plane is two parallel lines or lines D are incorrect and most likely result from
with the same slope. To solve frir the slope, calculation errors in solving the system of
convert each equation to slope- intercept form . equations.
Subtracting 24* to both sides of Lhe first equation

yields -6 y -2ix + 33. Dividing -6 from both
-24 33 60. Level: Easy | Skill / Knowledge: Systems of two
sides of lhe first equation yields y x+
—6 —
—6 linear equations in two variables | Testing Point:
33
or y — 4x — 6 "
. Therefore , the slope of the first Solving a system of equations with infinitely
many solutions
equation is 4. Subtracting Star to both sides of the
second equation yields -sy = - 8JC - 12. Dividing
Key Explanation: Choice A is correct . For a
-s from both sides of Lhe second equation yields
-& 12 & 12 system to haw inliiiildy many solutions, Lhe
y — x or y = - x -\ . Ihcreloie, the
—s r cuelhcienls of both x and y and the Constant
slope of the second equation is — . Since Lhe on the right side of the equation must be
proportional between Lhe two equations. Using
two equations have equal slope, then \ = - . the value of b and the coefficient ofy in the
i
second equation to calculate the ratio between
- from bolh sides of
Multiplying s and dividing 1
.
the first and scrund equation yields —4 or -2
-
,

——
*

the equation yields J . Therefore, s = 2, Therefore, the coefficients and constant in the
first equation must be multiplied by -2 to get the
59. Level: Medium | Skill / Knowledge: Systems of coefficients and constant in the second equation.
4 -2 )
two linear equations in two variables|Testing Multiplying -2 to get the value of a yield e C
Pointi Solving a system of linear equations and 8 5
determining the quadrant of the solution
Therefore, u = —— or -1.6. Multiplying -2 to get

the value of c yields 17(-2). Therefore, c = -34.


Key Explanation: Choice C is correct First, Adding the values ofa, b, and c yields a + b + c =
find the solution to the system of equations -1.6 + 2 - 3 4 = -33.6
using the elimination method. Multiplying
the first equation by 5 yields 45* + 5y = -95. DistractOT Explanations: Choice R is incorrect
because it is the value of a- Choice C is incorrect
Adding the first and the second equation yields
because it is the value of c. Choice D is incorrect

terms yields ^
45jc + 5y + x -5y = -95 +

x= ^ . Combining like

. Multiplying 4 and
dividing 137 from both sides of the equation
and may result from a conceptual or calculation
error.

itmL vibrdntpublisbefS- tOftl 69

Telegram: SAT_pdf
I
61. Level: Medium | Skill / Knowledge: Systems of
ALGEBRA

Combining like terms yields 7c = 105.


I
two linear equations in two variables | Testing Dividing both sides of the equation by 7 yields c
Point: Solving systems of linear equations in two
- 15.
variables

Key Explanation: Choice 11 is correct. In order 63. Level: Easy|Skill'Knowledge! Systems of two
for a system of equations to be true for ail real linear equations in two variables | Testing Point :
numbers, the equations must be the same line. Creating a system of equations from lest in a
The coefficients for both variables, as well as the word problem
constants, must he proportional to one another
Start by reorganizing the equations into the same Key Explanation : Choice A is correct. The
form which yields word problem describes a system of equations

dx + 7 y = 4 and with variables representing the number of 60-


Ax + 14y = e. minute panelists x and 0(1- minute panelistsy. If
Jennifer is offering speakers un the 60-minule
The coefficients ofy reveal that the equations
panel $ 50 for their lime, and the speakers on the
are proportional and have a ratio of 7:14 or 1:2.
L 20-minule panel $110 for Lheir lime* then one
Theneibre, the coefficients of x must also have
equation musL be 50* + 1 lQy - 900. The other
the same radio. Applying Lhe ratio yields —1 2
. equation represents the total number of speakers
Multiplying 4 to both Sides of the equation yields x + y, which must be equal to L 2 . Therefore', Lhe
d = 2. other equation is x + y - 12.

Distractor Explanations: Choices R, C, and D


62. Level: Easy|Skill / Knowledge: Systems of two
are incurred and may result from a conceptual
linear equations in two variables|Testing Point:
or calculation error.
Creating and solving a system of linear equations
in two variables
64. I.evel; Easy | Ski I I / Knowledge: System s of two
Key Explanation : The correct answer is 15 cup®. linear equations in two variables|Testing Point:
To determine the number of cups Jerry bought, Solving System of Linear Equations
one must write a system of equations, with
variables for the number of napkins bought ( P) Key Explanation: Choice C is correct. Plugging
and cups bought (c). .
( 25 11 ) into the first equation, yields II + 25 =
36 or 36 = 36, which is true. Plugging ( 25, 11)
Since he buys twice as many napkins as cups, the
first equation can be written as 2c = n.
into the second equation yields 2 (11) + 8( 25) =
140. Simplifying the equation yields 222 = 140,
Since he spent a total of $105 for buying napkins which is false. Therefore, Choice C is the correct
and cups, the second equation can be written as answer,
2 n + 3c = 105 .

Substituting the value of n from the first equation Distractor Explanations: Choices A and B are

to the second equation yields 2( 2c) + 3c = 105 or incorrect. These statements are false. Choice D is

4c + 3c = 105 . incorrect. There is enough information to answer


the question .
Digital SAT Math Practice Questions 70

Telegram: SAT_pdf
I Systems of Iwo linear equations in two variables ( Answers )

65. Level: Easy | Skill / Knowledge: Systems of two Adding 12 to both sides of the equation yields
I
linear equations in two variables | Testing Point: 3r = 17. Dividing both sides of the equation
17
Solve a system of linear equations —
by yields x = . Now, find y by substituting
the value for x into either of the two equations.
Key Explanation: the correct answer is 1. lhc
easiest way to solve the system of equations is
Substituting it to the first equation yields
34
to use the elimination method. Adding the two -6 -
3
6= —3 = 5.33
equations directly yields 5c = 5 . Dividing both
sides of the equation hy 5 results in e - 1 .
68, Level: Medium | Shill,''Knowledge: Systems of
two linear equations in two variables | Testing
66. Level: Medium | Skill / Knowledge: Systems of Point : Working with linear systems with no
two linear equations in two variables|Testing solutions
Point: Solving a system of linear equations
Key Explanation: Linear systems with no
Key Explanation: 14 is the correct answer. First solution are parallel and have the same gradient.
multiply Quenliris sales by Lwo in order lo set the
entire equation equal to Alices sales: Using distributive properly to simplify both
equations yields y = 3x - 15 + 10 - 2 x and y - 2 p
2(5b + 43} = 5-56 + 159.
- px + 6 x.
Using distributive properLy yields 10 b + 96 - 5.5b
Combining like terms yields y = x - 5 and y - (6
+ 159.
- p) j: + 2 p. Equating the slopes of the equations
Subtracting 96 and 5.5b from both sides of the yields L - 6 - p. Adding p and subtracting 1 from
equation yields 4.5 b = 63. both sides of the equation yields p = 5.
Dividing both sides of the equation by 4.5 yields
b = 14.

67, Level: Easy | Skill / Knowledge: Systems of two


linear equations in two variables | Testing Point:
Salving a system of two linear equations

Key Explanation; The correct answer is 5,33,


Using the first equation to solve for y yields
-
y - 6 + 2x, Substitute the value fory in
terms of x into the second equation and solve
- -
for x. This yields 2 ( 6 + 2x ) x = 5. Using
distributive property yields -12 + 4JC - JC = 5,
Combining like terms yields -12 + 3JC = 5.

wwwi vibrdntpublisbefS-tOftl 71

Telegram: SAT_pdf
I ALGEBRA
I
If x 3 which of the ini [owing statements is If - 4JT = 5. and y > ftflO then what is the the
NOT true? least integer value of JC?

A} 6

B) 4x + 2 U
h
C)

^
l
D) -2 --
Jackie is scheduling her next haircut, if her hair
:
is already I 0 inches long and she knows her hair
grows at least an inch every two months, which
statement represents the total length of her hair
( y ) after (JE ) months?

A) 0.51 +10

B) y = 0.5* + 10
lhe shaded region in the graph represents which
'

of the following inequalities? C) y 2^c + 10


D) y 0-5*

4 If x 1- 0 and y 30, then which of the following


must also he true ?

A ) x - y Z 70
B) x - y lO
^
C) x - y £ l 0
D) * + y < 10
A} y > 9j t - 4
B) y > 9x - 4
C) j
-
D ) y 9x - 4
t t e- 4

Digital SAT Math Practice Questions 72

Telegram: SAT_pdf
I Linear inequalities in one or two variables
I
If -40 x 90, and -30 y 20 , then what is x + 2 < -l
-
the greatest possible value of JE yl
x < " x -+ 2 i
s
Which of the following inequalities expresses the
domain of x-coordinates that satisfy the system
of inequalities above?

A ) x < -1
If |&t + 14| 4x + 24 then which of the following li) x > -L
is true?
Q -2 < x < -3
A) Je > 5 D > x ^ -3
—It-n
)
R) x <.

19
C) - —: < * < 5
19
D ) xS 5 orx ——
If y 1, which of the following statements is
NOT Hue ?

A ) ly 12
B) 4y + 7 > 22

^
If >/2016 < <*VH » ~ = < Vli, and a is an
V 14

integer then what is the value of a ?


C)
S
- y > 25

1
D)
v -2 > —:1

wwwt vibriintpu Wishers tortt


, 73

Telegram: SAT_pdf
I ALGEBRA
I
Jane is knitting scarves and hats for her family What is the value of t if t is an integer and
as presents. If each scarf takes 15 feet of yarn |2f - 2| < t ?
lo make and each hat takes 12 feet of yarn to
make, and she purchases 425 feet of yarn* which A ) -1
inequality represents the maximum number of
B) 0
hats ( ft ) and scarves fa) she can make for her
family? C) 1
D) 2
A ) 425 15/f +12s‘
13) 425 15s + 12fi

C) 425 15fi +12s


D) 425 I 5r + I 2fi

I f p q < 1 Of ) and q is a positive multiple of 6, what


is the greatest possible integer value of p?

A) 6
B) 16
A hardware company is selling a new kind of
picture hanging stud that can hold up to 12 C) 30
pounds. It a painting is x pounds and the frame D) %
is y pounds, which of the following inequality
correctly expresses the amount of weight that the
Studfa ) can hold ?

A} s
^ y- x
B) i Z y + x
C) s < y - x Variables r and s are real numbers. If 20 < r < 30
and 40 < s < 60, which expression represents all
D) s y+ x the possible values of the expression r - j?

A) - 40 < r - s < - 10
B) - 3 0 < r - s < - 2 0
C) - 3 0 < r - j < 2[>
D) - 20 < r-r < 30

Digital SAT Math Practice Questions 74

Telegram: SAT_pdf
I Linear inequalities in one or two variables
I
Which of the iol [owing is a solution to the Carmen wants to spend no more than $43 on
inequality below? books and pens for her next semester. A box of
pons costs $5 and a book costs $ 3. If she buys
2x + 5 +7 only one box of pens, what is the maximum
2y > - x + 4 number of books she can buy?

A ) ( 2, 4 ) A ) 12

U ) (-4, 3) B ) 13

C) (2, 1) C) 3ft
13) (-3, 3) D) 3

A bakery has a single oven for making loaves


ufbread and cakes. Using the oven, the baker
can only make 27 pastries in a day. The baker
can only bake a maximum of 4 loaves of bread
in a day. If the bakery must produce aL least one
for each type of pastry, what is the maximum
number of cakes the baker can make on a
particular day?

A ) 23
U ) 24
C ) 26
D) 27

If 12n < 49 and 7n > 22 and n is an integer, what


is the value of n ?

t'l & rdnfpijblishtfri.COrtl 75

Telegram: SAT_pdf
I
69. Level: Medium | Skill / Knowledge: Linear
ALGEBRA

71. Level: Easy| Ski 11 / Knowledge : Linear


I
inequalities in one or two variables | Testing inequalities in one or two variables|Testing
Point: Solving linear inequalities Point: Solving linear inequalities

Key Explanation: Choice C is correct. In order Key Explanation: Using the given equation 8y -
to find the inequality that is not true, simplify .
Ax = 5 find the value of y in terms of x .
each option by isolating x . The inequality that Adding 4* to both sides of the equation yields 8 y
will not t>e simplified as x 3 will be the answer. = 4x + 5.
Simplify Choice C by multiplying 5 and dividing Dividing both sides of the equation by fi yields
6 from both sides of Lhe inequality. This yields 4x + 5
y=
8
or xS . Therefore, this statement is
Subs Liiu ling the equation to the inequality y >
false.
6
^ 800 yields .

Distractor Explanations: Choice A is incorrect Multiplying to both sides of the inequality


because the inequality can be simplified tn x 3 yields Ax + 5 > 6400.
by dividing both sides by 2. Choice 13 is incorrect Subtracting 5 from both Sides of the inequality
because lhe inequality can be simplified to x yields 4x > 6395.
3 by subtracting 2 then dividing both sides by - from both sides of the equation yields
Dividing 1
A. Choice D is incorrect because lhe inequality
6395
can be simplified to x 3 by adding 2 and then x > —— or x >159(1.75. Since x must be greater
multiplying both sides by 2. than 15911.75, then the least integer value of x is
1599-
70. level: Easy|Sldlll / Kno wledge : Linear
ineqnabties in one or two variables | Testing Jl , Level: Hard | Ski II / Knowledge: Linear
Point: Determining a linear inequality equaliun inequalities in one or two variables |Testing
from its graph Point: Solving linear inequalities

Key Explanation : Choice A is correct. The solid Key Explanation: Choke A is correct , Since her
red line on the graph indicates that the inequality hair grows at least an inch every two months,
symbol used is either or sign. Si nee the then her hair grows at least half an inch every
shading on the graph is above the hue, then it month . This means that hair growth Afar x mnniter
indicates that the inequality symbol used is either 0,5x. To create the inequality for the total length
> or >, Since > appears on both criteria, then this of her hair after x months add the hair growth
is the symbol used in the inequality represented after x months to the length of the existing hair.
by the graph. This yieldsy 0,5x + 10 ,

Distractor Explanations; Choices B, C, arid D Distractor Explanations: Choice B is incorrect


are incorrect and most likely result from a lack of and may result from not considering that her
knowledge in graphing inequalities. hair can grow more than 1 inch per two months.

Digital SAT Math Practice Questions 76

Telegram: SAT_pdf
I Linear inequalities in one or Iwo variables { Answers )

Choice C is incorrect and may result from an 75. Level: Medium | Skill / Knowledge: Linear
I
error in representing the statement “at least an inequalities in one or two variables | Testing
inch every two months’" into an expression . Point: Solve linear inequalities
Choice D is incorrect and may result from
calculating only the hair growth after x months Key Explanation: Choice C is the correct option .
and not considering the existing length. Given 16* + 141 4x + 24 , the way to solve an
ahsolute value equation is to split into two linear
inequalities as follows:
73. Level: Hard | Skill / Knowledge: Linear
ftr + 14 4x + 24 or fix + 14 -{4* + 24).
inequalities in one or two variables | Testing
Thus subtracting 14 from both sides of the first
Point : Combining linear inequalities in one
inequality results in (is: 4x + 10. Subtracting Ax
variable to a linear inequality in two variables
from both sides of the first inequality yields 2 x
10. Dividing both sides by 2 results in x 5.
Key Explanation: ChoiceC is correct. Rewriting
Only Choice C has this as a possible answer.
the second inequality to have a symbol yields
30 y. Combining the two inequalities yields x +
Distractor Explanations: The error in choosing
30 40 + y Subtractingy and 31 i from both sides
.
Choice A or B occurs when one considers only
of the inequality yields x - y 40 - 30 orx - y
one value of the inequality. Choice D is the
10.
opposite of the required value for x.

DistratLor Explanations: Choice A is incorrect


and may result from a calculation error in 76. Level: Medium | Skill / Knowledge: Linear
combining the constants. Choice B is incorrect inequalities in one or two variables | Testing
and may result from using the wrong inequality' Point; Solve linear inequalities
symbol. Choice D is incorrect and may result
from using the wrong inequality symbol and sign Key Explanation: 13 is the curneel answer.
Given 2, 016 < aj14 dividing both sides of the
fory.
^ -
inequality by i/fl and putting the expression
under one square root results in
74. Level ; Hard | Skill / Knowledge; Linear
inequalities in one or two variables | Testing < a
Point; Interpreting compound inequalities
This simplifies to: i/i 44 < a or 12 < a. So a has
Key Explanation: The greatest possible value of to be greater than 12.
the expression x - y can be obtained using the
maximum value of x and the minimum value Solving L < ^JiA by multiplying both sides of
V14
ofy. From the given inequalities, the maximum
the inequality by Vl 4 results in a < VHVH *
possible value cfxis 90 and the minimum
This simplifies to a < 14.
possible value ofy is -30. Substituting the values
So 12 < a < 14
to the expression x - y yields 90 - (-30) or 120.
Since a is an integer, 4 = 13.

.
wwW vibrdntpublisbefS- tOftl 77

Telegram: SAT_pdf
I
77. Level: Medium | Skill / Knnwledge: Linear
ALGEBRA

of an incorrect combination of variables and


I
inequalities in one or two variables | Testing inequalities.
Point: Solve linear inequalities

79. Level: Easy| Skill / Knowledge : Linear


Key Explanation : Choice C is correct.
inequalities in one or two variables | Testing
Subtracting 2 from both sides of the first
Point: Solving linear inequalities
inequality produces x -3; In the second
inequality can be reduced to 2*. Subtracting Key Explanation: Choice if is oorrcct . Ihe
2.v from both sides of the second inequality inequality must be less lhan or equal to 425
yields -x < 2. Dividing both sides of the second feet of yarn, as that is the maximum value
.
inequality by negative 1 yields x > -2 ( When Jane purchased. The values 15 and 12 must
you multiply or divide an inequality by a negative be the cudhcienls of the variables (s) and ( Ji ),
number, the inequality sign must be flipped.) respectively.
Thus, to satisfy the above system of inequalities, Since ihe Lohd yarn Lo be used is represented by
the x value must he less than or equal to -3 but ihe expression 15J + L 2h , then the inequality is
greater than -2t thus giving Choice C- Note: the 425 15s + 12k .
y-coordirates are irrelevant as the question asks
tor the domain and not the range of the system of Distractor Explanations; Choices A , C, and D
inequalities. are incorrect and are likely the result of errors in
interpreting the values in the given question .
Distractor Explanations: Choices A, B , and
C are incorrect and are likely the result of
30. Level: Easy|Skill/ Knowledge: Linear
not solving both systems of inequalities and
inequalities in one or two variables|Testing
combining the domains or from arithmetic
Point: Solving linear inequalities
errors.

Key Explanation: Choice B is correct . If the


73. Level: Easy|Skill / Knowledge : Linear frame can hold up to 12 pounds, the variable
inequalities in one or two variables | Testing (s) must be greater lhan or equal lo the sum
Point: Solving linear inequalities of the other variables. Since the total weight of
the painting and the frame is x + y, then the
Key Explanation: Choice C is correct, in order .
inequality iss > x + y o r s > y + jr. Therefore
Lo find the statement that is not true, choose a Choice R is correct.
value for y which is greater than or equal to 4 and
substitute in each option. Using 5 as the value of Distractor Explanations; Choices A , C, and
8 D are incorrect and are the result of errors in
y and plugging it in Choice C yields -(5) > 25.
creating inequalities.
Simplifying the inequality yields 8 25 which is
false. Therefore, Choice C is the correct answer.

Detractor Explanations; Choices A , R ,


and f > are incorrect and are likely the result

Digital SAT Math Practice Questions 78

Telegram: SAT_pdf
I Linear inequalities in one or Iwo variables { Answers )

31. Level: Medium | Skill / Knowledge: Linear Si. Level: Hard | Skill / Knowledge: Linear
I
inequalities in one nr two variables | Testing inequalities in one or two variables | Testing
Point: Solving linear inequalities with absolute Point: Solving compound linear inequalities
value
Key Explanation: Choice A is correct. To find
Key Explanation; Choice Cis correct. Id find
' the range of the possible values of the expression
the possible values of t, make two inequalities r - s,subtract the opposite boundary limits of the
using the content of the absolute value and the two variables.
positive and negative values of the constant. To gel the minimum boundary of r - s , subtract
This yields 2f - 2 < 1 and 2f - 2 > - 1 . Solving the
ihc maximum boundary of s from the minimum
values oft yields l < ~ and t > — - Combining boundary of r which yields 20 - 60 = -40.
1 To get the maximum boundary of r - s, subtract
the two inequalities yields — < t < -3 or 0.5 < ( <
the minimum boundary of s from the maximum
1.5. Since t is an integer, then t can only he equal
boundary of r which yields 30 - +0 - -10-
to 1 .
Ihcrcfore, Lhe correct answer is -40 < r - r < -10.
Dislraclor Explanations: Choices A , B, and
Distractor Explanations; Choices B , C, and D
D are incorrect and are the result uf errors in
are incorrect and reflect errors in interpreting
evaluating absolute value inequalities .
inequalities.

32. Level: Easy | Skill / Knuw ledge: Linear


H 4. Level : Medium | Ski 11 / Knowledge: Linear
inequalities in one or two variables | Testing
inequalities in one ur two variables | Testing
Point : Solving linear inequalities
Point : Solving for systems of linear inequalities

Key Explanation: Choice K is correct - The


Key Explanation: Choice A Is correct - To
greatest possible integer value ofp car be
determine if a point is a solution , plug it into
obtained using the least value of *}- Since q is a
both inequalities and verify if the statements are
positive multiple uf 6, the least value uf q is 6.
true. Plugging in (2, 4) from choice A to the 1 st
Substituting 6 for q in the given inequality yields
inequality yields 2 (2 ) + 5 3(2 ) + 7, Simplifying
p(6) < 100. Dividing both sides of the inequality
it yields ? 13, which is true.
by 6 yields p < lti.67, Therefore, the greatest
possible integer value of p is lb . .
Plugging in ( 2 4) to the 2nd inequality yields
2(4) > - (2) + 4. Simplifying it yields 8 > 2, which
Distractor Explanations: Choice A is incorrect is also true, Since both statements are true, then
and may result from finding the least value of (2, 4) is a solution to the system of inequalities.
q. Choice C is incorrect and may result from
a conceptual or calculation error. Choice D is Distractor Explanations: Choice B is incorrect.

incorrect and may result from Ending the value Substituting (-4, 3) to the 2nd inequality yields
of pq when p = 16 and q = 6. --
2(3) > ( 4) + 4 or 6 > 8, which is false.
Choice C is incorrect. Substituting ( 2, 1) to the

.
wwW vibrdntpublisbefS- tOftl 79

Telegram: SAT_pdf
I 2nd inequality yields 2( 1 ) > - 2 + 4 or 2 > 2,
ALGEBRA

87. Level: Easy | Skill / Knowledge: Linear


I
which is false. inequalities in one or two variables|Testing
Point: Creating and solving equations for linear
Choice D is incorreel. Substituting (-3, 3) to the
inequalities
2nd inequality yields 2( 3 ) > - (-3) + 4 or 6 > 7,
which is false,
Key Explanation: Choice A is correct . Since
Carmen buys only one box of pens, the amount
AS. Level: Easy|Skill / Knowledge: Linear left that she can only spend to buy hooks is $43 -
inequalities in one or two variables | Testing $5 = $38. Let b be the number of hooks that she
Point: Solving linear inequality equations will huy. Hence, 2 b is the total amount of money
to he spent on hooks. Since she can only spend
Key Explanation : Choice C is correct. The
up to $38, then 3b 38. Dividing both sides of
baker Can bake a maximum of 4 loaves uf bread
the inequality by 3 yields b < 12.67. The next
a day. The leasl number uf loaves of bread lhaL
the baker can make is 1 . Hence, the maximum lower inLcger is 12 . Therefore, Carmen can buy a
number of cakes the baker can make is 27 - 1 = maximum of L 2 books.
26 Cakes.
Distractor Explanations: Choice E is incorrect
Distractor Explanations: Choice A is incorrect.
This is the value of the least number of cakes A hook costs $3 , however, alter buying 12 hooks
they can bake in a day, Choice B and Choice D and 1 box of pens, she is left with $2, which is not
are incurred. These options can be a resulL uf a enough tor the 13th book. Choice C is incorrect.
conceptual error. This is the amount she can spend atter buying
a bux uf pens. Choice D is incorrect This is the
cost per book.
86. Level: Easy | Skill / Knowledge: linear
inequalities in one or two variables | Testing
Point: Solving linear inequalities

Key Explanation : The correct answer is 4. Since


ao
12n < 49, n < = 4.1. Since 7 n > 22,then n >
12

—22 = 3.1. Combining the inequalities yields


3,1 < H < 4,1,
Therefore, n can only be 4.

Digital SAT Math Practice Questions 80

Telegram: SAT_pdf
Advanced Math
This chapter includes questions on the following topics:
• Equivalent expressions
• Nonlinear equations in one variable and systems of equations in two variables
• Nonlinear functions

Telegram: SAT_pdf
I ADVANCED MATH
I
Which answer correctly simplifies the expression Simplify the following expression
2
50* + 40* - ISO - 20( x?+ x - i ) J
2x l— *- l
x -3 x+2
A ) SO*1 + 60* 170 -
_ '
xl +7 x - S '
B ) 70

CJ
*
2
60JC - ! 70

3fhr + 50* - ISO


A}
--
, *? x 6 j
V + 7* - S "
D) 30x! + 20* - 130 B)
t
xz + * - 6 j

C)
xz - x - 1 ^
2*- l j

( x* + x -\ )
D)
(* + 2)(JC - 3 ; >
Simplify' the following expression: *(* - 5) + 4
U - 3)- a.
A ) (* - io) C* + aj
B) (* - 4)(* + 3)
C) (* - 5)(* + 4) The diffisrencc of -7 x' + 4* - 12 and 3*^ + l &t + 4
can he written in the form a1 + bx + c where a , fc,
D) x - x - S
’ and c are constants. What is the value of a x t ?

90

Which of the following expressions is equal to

*v ? Whai is the value of m in the equation


fa ")- m
-=. 12S if n2 - 0 and if m > (1?

A) l
x
B) *
Q y
X
D)
y

Digital SAT Math Practice Questions 82

Telegram: SAT_pdf
I Equivalent expressions
I
The equation of the acceleration of an object - - -^ - -
( a2b 3a1 + lOflfr3} ( l b lab2 3b1 )
sinking in water can be represented by
Which of the following is equivalent to the
a= s H— preceding expression?

where a is the acceleration in meters per second,


g is the gravitational constant of 9.8 meters pet
A ) 3a1b -W +W - naP
second squared, «r is the mass of the displaced B ) 3a1b - Mab!
water in grams, and mff is the mass of the object C) 3dsh - 3d2 + 3 + 12
itself in grams. ^ ^
U ) 15
^ ^ +3 + 3ft2
Which of the following equations expresses the
mass of the displaced water ( mj in terms of the
other variables?
a
A) -1
£
Tfftj > 4, which of the following is equivalent to
o
B} mw =- 1
* -i
a
a
f — j f —y +
\ m+ 4 j
C) mw - -mv K
+f
A)
1
2m + 6

^^
n> rn | R ) lm + 6
+l
s C) m1 + 6 m + 8
(m1
+ 6m + 8 )
D)
2m + 6

What is the difference of (7 - 2 x ) - (-8x + 1 OJC3) ?

A ) 10 jd - 2i + 15

B) -10Ur2 + 6 x + 7 If a hours and 3-0 minutes is equal to 4 — hours,


C) lOUd - Za - l what is the value of a in quarter hours?
D) -10Ud - 2x - \

wwwi ribrtintpu blishtfri .COrtl 83

Telegram: SAT_pdf
I ADVANCED MATH
I
Simplify the following expression: y[ y - 5} + 4 For all values of z > 0, which of the following
(v - 4) - 4. expressions Is equivalent to

A) Or + ll )5
—*1 + —1y
?
B) lly - 11 z
A) Zl£
C ) (r - 5) (y + 4) z
D) f - y-8 zy + zx
B)
xyz
xy
C)
s( x + y )

D)
" V'
Which of the following expressions is equal to
a"1#
1

A)
*
B ) a2 2X 2 +6 X - I I B
If - 2x + n+ , what is the
JC -3 x- 3
C) h 1
value of B?
3
rt
D)
F A ) -11
B) 25
C) 14
D) 36
|Q]
Which of the following is equivalent to 6x?( xyz )r.

A ) 6i’6jfjf 6 xs
B) 21 Bxyz1
C) bx } yz If 5( xy + yz ) - 2( xz - xy ) = axy + byz + cxz, what
is the value of a ?
D) 36x?yz

A) 7
B) -2
C) 5

D) 10

Digital SAT Math Practice Questions 84

Telegram: SAT_pdf
I Equivalent expressions
I
^
IT the expression a 2 - is1 is equivalent to (a)tl ,
what is the value p if b = a?

[
^ —
the value of d + b + c?

If 3 + bx + c {2r + 1 )( jt 5) - 13 then what is

Which of the following expresses the parabola


y - ( jr +- 3}(r - 1 ) in vertex farm ?

A ) y = xL + 2x - 3
B} y = ( X - i y + A
C) y = { x + * Y - A
D) y = l*- 2i + 3

A parabolic function can be written in the terms


of /tx) = ax2 + bx + c, What is the value of -c after
^
simplifying the function f [ x ) = (7 + 3JE - 18) -
(-Jr1 - 17r + 19)?

A) -31
B) -37
C) 37
D) 41

ivmt vibrdntpublisbefS- tOftl 85

Telegram: SAT_pdf
I
8fi. Level: Medium | Skill / Knowledge: Equivalent
ADVANCED MATH

Key Explanation: Choice D is correct. Start


I
expressions|Testing Point: Simplifying by simplifying the exponents of each variable.
quadratic expressions by using the distributive Subtracting the exponents of the matching bases
property and combining like terms being divided yields x-J y 1 1, Simplifying
ihe expression yields xy 1 and according to the

Key Explanation: Choice D is correct. Using inverse exponent rules, the expression can be
r
^
the distributive property yields 50 + 4flx - 1 SO
- 2(bf! - 2flx + 20. Combining like terms yields
rewritten as
r
30xi + 20x - 130. Distractor Explanations: Choices A , B, andC
are incorrect and may result from a conceptual
Distractor Explanations: Choice A is incorrect or calculation error.
and may result from an error in distributing the
negative sign inside the parentheses. Choices
01 . Level; Medium |Skill / Knowledge: Equivalent
U and C are incurred and may result from a
expressions | Tbsting Point: Combining
conceptual Or calculation error.
rational expressions through the least common
denominator
S9. Level: Medium | Skill / Know ledge: Equivalent
expressions | Testing Point: Using the Key Explanation: Chok e A is correct.
distributive properly lo simplify a linear To simplify the expression, rewrite
expression ihe fractions using ihe leasl common

denom inator whi ch i s ( x - 3M* - 2}. Th is


Key Explanation : Choice C is correct. To
simplify the expression ,, use distributive property yields
(2x - l }(* + 2) (x - lXx - 3)
( x - 3)(r + 2) (x + 2)( x - 3)
.
which yieldsx2 - 5x + 4AC - 12 - 8. Combining ( 2JC - l)( x + 2) - fx — 3)(x - 1) ,
like terms yields x3 - x - 20. Using distributive
( x - 3)( x + 2 )
The equation can he simplified once more by
properly yields
2xJ - jt + 4x - 2 - x ’ + 4x - 3
factoring which yields ( x - 5) ( x + 4 ), xi - x ~ 6
3
x +7 x - S
Combining like terms yields
Dis tractor Explanations: Choice A is incorrect x1 - x - b
and may result from an error in factoring r1 - x
Distractor Explanations; Choices R, C, and E>
- 20. Choice B is incorrect and may result from
are incorrect and may result from calculation
a calculation error which leads lo the wrong
errors in simplifying polynomials and fractions.
expression x3 - x - 12 . Choice D is incorrect
and may result from disregarding -12 in the
expression . -
92 , I eveb Medium |Skill / Knowledge; Equivalent
expressions|Testing Point; Operations with
quadratic equations
90. Level: Medium | SkiJJ / Knowledge: Equivalent
expressions | Testing Point: Using exponent
Key Explanation: The correct answer is 160.
rules to rewrite a rational expression.
Subtracting the second polynomial from the first
polynomial and combining like terms results in

Digital SAT Math Practice Questions 86

Telegram: SAT_pdf
I -7 xi + 4x - 12 - (3 416* 4 4}
Equivalent expressions ( Answers )
I
^ 1 =—. Next, subtract 1 from both
= -7x* + 4x - l 2 + (-it3) 4 (-I6x) 4 ( -4)
S
= -lOjd - 12* - 16 sides of the equation to get:
The standard form of a quadratic equation isy =
ax2 +bx + c. Therefore, a = -10, b = -12, c -16.
The product of a and c is -10 x -16 = 160.
- -
11 '
= —-_ 1 Then multiply both sides of

the equation by - mn to get:


93. Level: Hard | Skill / Knowledge: Equivalent nK - +1 which is Choice C.
Expressions | Testing Point: Use exponent rules &
and the difference of two squares Distract or Explanations: Choice A would be
selected if a math calculation mistake was made.
Key Explanation: Using exponent rules to raise
Choices B and D would he incorrectly picked
a power to a power results in the equation being
rewritten as: (2)6"** “/ - = 12&. (m 4 n )( m - n)
represents Lhc factored form uf the difference of
if a mistake was made in isolating the correct
variable.

two squares and can be wriLLcn as m1 - H 3. Thus,


2^1 = 123 i 27 = 12&> so 27 = 2^ 1 Sulw. 95- Level : Easy | Skill / Knowledge: Equivalent
n1 9, substituting 9 for n 3 in the equation and expressions | Testing Point: Using the
distributive property and combining like terms
_
solving for m results in
27 2»a 2 11a2

27 2»®a2 9
Key Explanation: Choice B is correct.

Using distributive property to simplify the


Since the bases are the same, the exponents must
expression (7 - 2x ) - ( ~8x 4 1 Obd ) yields 7 - 2x 1-
also be equal. So:
ftx - W x2 -
ms - 9 = 7. Adding 9 to both sides yields
zn! = 16. T aking the square root of both sides Combining like terms yields - lOxf * 4 6x + 7.
results in m = ± 4.
Distractor F.xplauatinns:Choices A, C, and D
Since m > 0 the answer is 4,
are incorrect and may result from a conceptual
or calculation error.
94. Level: Easy | Skill / Knowledge: Equivalent
Expressions | Testing Point: Rewrite a rational
96, Level : Easy|Skill / Knowledge; Equivalent
expression to solve for a particular variable
expressions | Testing Point; Using the
distributive property and combining like terms
Key Explanation: Given the equation

Key Explanation: Choice C is correct , Start by


.
Fjr
distributing the negative sign to the parentheses
To solve for first divide both sides of the of the second term in the expression { alb - 3n! 4
equation by g to get: ^--
lOaf ) ( 2 - 2ab1 - 3 1 ).
^
This yields a b - 3a + lOuff + 2s3i? + lab2 + 3i?:.
2 3

.
wwW vibrdntpublisbefS-tOftl 87

Telegram: SAT_pdf
I Combining like terms yields 3i i - 3n + 3i s +
2 ?
ADVANCED MATH

99. Level: Easy| Skill / Knowledge : Equivalent


I
12(1
^ . expressions|Testing Point; Using the
distributive property to simplify a quadratic
Detractor Explanations: Choices A, B , and D expression
are incorrect and reflect errors in simplifying
complex polynomials. Key Explanation: Choice C is correct. In
order to simplify the expression, distribute and
combine Like terms which yields y* - Sy + Ay -
97. Level: Hard|Skill/ Knowledge; Equivalent
16 - 4 or f - - y - 2 0.
expressions|Testing Point; Rewriting a
rational expression by finding the least common The equation can be simplified once more by
denominator factoring which yields { y - 5 )(y + 4 ) .
Therefore, the correct answer is C-
Key Explanation : Choice D is correct.
firstly, And the LCM for the denominators of m DiSiractOr Explanations: Choices A, U and
+ 2 and m + 4. The LLTM is ( m + 2)( m + 4) . D are incorrect and arc the restilL of incorrectly
distributing values, combining like terms, or
Simplifying the denominator yields
factoring polynomials.
1

:
1
or
m+4+ m+2 2tn + 6
( m + 2)0n + 4) + 6m + 8 lOO.LcveL Easy | Skill / Knowledge: Equivalent
expressions|Testing Point: Using exponent
The denominator in Lhe denominator
rules to rewrite a rational expression
will become the numerator which yields
( m2 + 6m + 8 }
Key Explanation : Choice D is correct. Start
( 2m + 6)
by simplifying the exponents of each variable.
Distractor Explanations: Choices A , E , and C
By subtracting the exponents of the matching
are incorrect and reflect errors in simplifying
bases being divided yields This can
complex fractions with different denominators. be simplified to a b . Since b has a negative
2 ~2

exponent, it will be moved to the denominator to


9«. bevel; Easy|Skill / Knowledge; Equivalent make the exponent positive. This yields '
expressions|Testing Point; Converting between
¥
units Distractor Explanations; Choices A , B, and C
are incorrect and may result from a conceptual
Key Explanation: The correct answer is 16. or calculation error.
30
Converting 30 minutes to hours yields or —
101. Level: Easy|SkiU / Knowledge: Equivalent
1
- hour. Since a H
2
L

hour
2
]
= 4 - hours, then a
2 expressions|Testing Point; Using the
= 4 hours. Converting 4 hours to quarter hours distributive property
yields 4( 4) = 16 quarter hours .

Digital SAT Math Practice Questions sa

Telegram: SAT_pdf
I Equivalent expressions ( Answers )

Key Explanation: Choice C is correct. Using re incorrect. These options may be a result of a
I
distributive property to simplify 6 x?{ xyz ) yields miscalculation or conceptual error.
'
fuc yz.

104. Level: Easy|Skill / Knowledge: Equivalent


Distractor Explanations: Choices A , B, and
expressions | Testing Point: Matching
D arc incorrect. These choices may result from
coefficients.
conceptual errors.
Key Explanation; Choice A is correct. Using the
102. Level: Hard | Skill / Knowledge: Equivalent distributive property to simplify 5{ xy + yz ) - 2( xz
expressions | Testing Point: Adding rational - xy ) yields 5 xy + 5yz - 2 xz + 2xy. Combining
expressions like terms yields 7xy + 5 yz - 2xz. Matching the
cuelllcients to axy + byz + txz yields a = 7, h = 5,
Key Explanation: Choice D is correct. and c = - 2.
Simplifying the numerator first yields
l i _ r +*_ DistractoT Explanations: Choice R is incorrect.
!
x y xy This is the value of c which is the coefficient of xz.
y+ x
Choice C is incorretL. This is the value b which is
This would rcsulL to ^z . By multiplying both the coefficient of yz. Choice D is incorretL. This
the numerator and denominator by
: we get is the value of u + b + t.
z
y+ x
, which is option D.
xyz 105 Level : Hard | Ski 11 / Knowledge: Equivalent
Distraclor Explanations: Choices A , B, and D expressions | Testing Point: Converting from
are incorrect. These options may be a result of exponents to radicals
conceptual or calculation errors.
Key Explanation: If b = a, then n 3 J? 1 = o1 J? h
Mince the bases an: the same, the exponents
103, Level: Easy | SkiU / Knuwkdge: Equivalent
should therefore be added.
expressions | Testing Point: Using the remainder
theorem.
Adding the exponents yields —2 + —3 = b
=—
6
Key Explanation; Choice B is correct - B is the
value of the remainder when 2s3 + 6x - 11 is Equating the two expressions yields « 6
divided by x - 3, The remainder would be the Rewriting the right side of the equation in
value ofyf 3). 11 11

exponential form yields = tjt’ , Matching the


Substituting 3 to x in ZJC3 + fa -11 yields 2( 3 )3 +
exponents yields p = 6,
6(3} - 11 or 25,

Distract or Explanations: Choice A is incorrect. 106. Level: Medium | Skill / Knowledge: Equivalent
This is the value of the y- intercept of the expressions | Testing Point: FOlLingout
-
equation y = 2x? + 6 x 11 . Choices C and D binomials and matching coefficients

wwW, vibrdntpublisbefS- tOftl 89

Telegram: SAT_pdf
I Key Explanation : To determine the values of
ADVANCED MATH

j 108. Level: Easy | S- kil 1 / Knowledge: Equivalent


I
a ,b,ct simplify the right side of the equation by expressions|Testing Pointi Using the
using distributive property which yields lx 10* 1
- distributive property and combining like terms
+ 1- 5 - 13 . Combining like terms yields ZJC2 -
9JC - 13, Comparing the coefficients and constant Key Explanation: 'Ihe correct answer is Choice
on the equation ax’ + hx + c = 2*1 - 9x - 18 C. Using distributive property lo simplify the
yields a = 2 , b = -9 and c = -13. Substituting the expression ( 7i^ + 3JC - 18 ) - (-3** - 17x + 19)
value to the expression a + b + c yields 2 - 9 - 18 . yields 7jd + 3x - 18 + Sx* + 17x - 19. Combining
'
therefore, the value of the expression is -25. -
like terms yields IQbd + 2t)x - 37 where c -37.
-
Therefore., -c 37.

107. level: Medium | Skill / Knowledge: Equivalent Dislractor Explanations; Choices A, and P
expressions|Testing Point: Converting from are inoorretL and may result from a calculation
one form of a parabola to another error, Choice B is incorrect and may result from
solving Lhe only value uf C.
Key Explanation : Choice C is correct. The
-
equation of a parabola in vertex form is y [ x -
+ k . To rewrite the given equation into verteT
form , expand the right side of the equation
which yields y - jd + 2x - 3. Completing Lhe
square and grouping Lhe perfect square Lrinumial
yields y = (JC' + 2JC + 1) - 4 . Simplifying Lhe
equaLion yields y -= ( x + 1)J - 4,

Distractor Explanations: Choices A , E , and D


are incorrect and may result from a conceptual
or calculation error-

Digital SAT Math Practice Questions 90

Telegram: SAT_pdf
I Nonlinear equations in one variable and systems of equations in two variables
I
+4 ( A- + 2 )
If
JE
= 2 then wlitil is ihe value of - ?
A1+ A - 90 =0
3 U if
If cis a solution to the equation above, and c > Q,
A) 2
what is the value of cf
13) 3
C) 4
D) 0

2* x 2f = 1,028
'

In the equation above, what is the value of x + y?


It' m + 4 — s' m + 10 , what is the solution set of
the equation ?

A) HU ]
*} {-6}
c) Hf
D ) {6}
If ( px + + K) — + rx + 24 for all values of
x andp + q — F>, what ate the two possible values
of p?

A ) 2 and 3
13) 6 and 1
Q Sand 2
1
y A +4 D ) - 1 and -6
y( x - 4) = x1

How many solutions are there at most to the


system of equations above?

A ) i solution
13) 2 solutions If d > 0 and cf - 16 = 0, what is the value of d ?
( 1) 3 solutions

D) 4 solutions

ivw. vibfiirttpijbHitiers.eiMi 91

Telegram: SAT_pdf
I ADVANCED MATH
I
If x = 7' j2 and lx =
^2 y , whal is the value of How many solutions does the equation below
have?

\ - 2 x + 6| - -4
A} 0

B) 1
C) 2
D) Inimite

If a 4 b p 4 rcj and q ^ 0,. ( hen f is represented
by which expression ?

A)
’]

R ) a-
If m and n are solutions to the quadratic equation
C) a + b + £ below, what is the value of m + n ?
'1
Ax? - 17* + 4 - 0
lt + b -l
D) -
A} 4
1
B)
4
17
c>
HQ 7
D) 1
If if is a positive integer and 2ir - 2k - 0, whal is
the value of fc?

If one of the solutions for the equation below is


—I , whal is the value of the other roots ?
Sx2 - bx - G 0

Digital SAT Math Practice Questians 92

Telegram: SAT_pdf
I Nonlinear equations in one variable and systems of equations in two variables
I
i
If b2 - - = 0 and the values of b are real n umbers,
4
what is the sum of all possible values of fc?

flow many solutions dues llie equation below


have?

3(JT - 2 )1 + 5 = 3x*+ 12x - 17

A) 0
B} 1
C) 2
D) Infinite

vibrdntpublisbefS-tOftl
itmt! 93

Telegram: SAT_pdf
I
109. Level: Easy| Ski 11/ Knowledge: Nonlinear
ADVANCED MATH

Subtracting m and 10 from both sides of the


I
equations in one variable and systems of equation yields
equation in two variables | Testing Point: Solving m1+ 7 m + 6 = 0 Factoring the quadratic
for a variable in an equation and finding the equation results in
correct value by substitution ( m + 6}(m + 1) = 0

Setting each m + 6 and m - 1 equal to zero


Key Explanation : Choice C is correct. Use
the equation —— =2 to get the value ofx .

Multiplying both sides of the equation by 3 yields


results in the solutions m = I and m = 6.


Therefore, m 1, -6, When sides of an
equation are squared, there can l>e extraneous
x + 4 = 6. Subtracting 4 from both sides of the .
solutions created Thus, both -6 and -1 need to
equation yields x - 2. Substituting x = 2 in the be cheeked inLo the original equation Lo sec if
(* + 2) , 2+ 2 4
there are any extraneons solutions. When -6 is
^^Tr 4
J
expression
(
substituted form in the original equation , there
Dis tractor Explanations: Choices A, B , and D is a negative number under the square root.
are incorrect and may result from a conceptual This is not a valid answer so -6 is rot a solution .
OTCALCULATION error. Therefbrej m — -l is the only solution , or Choice
C-

110, level: Easy | Skill/ Knowledge: Nonlinear


Dislraclor Explanations; Choices A, It , and 1>
equations in one variable and systems uf
are mourn'd . They am the resulL of arithmetic
equations in two variables | Testing Point : Solve
errors and not checking the answers into the
equations with square roots in one variable
original equation to eliminate the extraneous
solution .
Key Explanation : Choice C is correct. The
most efficient way to solve this problem is to just
substitute the answer choices and see which one 11 L Level: Easy|Skill/knu wlcdgc : Nonlinear
makes the equation true. Plugging in -6 intu equations in one variable and systems of
the equation results in -6 + 4 = V-4 or -2 = 2 . equations in two variables | Testing Point: Solve
which is not correct. This eliminates Choices A systems of nonlinear equations in two variables
and B. Testing Choke C results in -1 + 4 = .
OT 3 = 3. Thus Choice C is correct. Key Explanation: Choke D is correct.
Substituting the value of y from the first equation
The less efficient way to do this problem is to
into the second equation results in {JC! + 4) ( x - 4)
solve for m. Start by squaring both sides of the
equation to remove the square root:
The left side of the equation becomes: ( m + 4) J = Multiplying out the binomials on the left side of
mJ + 8m + 16. the equation yields
The right side of the equation becomes: m + 10. if -
4if + 4x -\b = if
Setting the two equations equal results in ?
Subtracting x from both sides of the equation
fti:+ StH + 16 = m + 10 results in
16 =0

Digital SAT Math Practice Questions 94

Telegram: SAT_pdf
I Nonlinear equations in one variable and systems of equations in two variables ( Answers )

-
I
The n Roots Theorem states
^ Jt)

of degree n, where rr 0, then /fx) has at most


is a polynomial Key Explanation: Choice A is correct. Using
distributive property to simplify the expression,
n zeros. Therefore, according to this theorem , ( px + 3 )( qx+ 3 ) yields ( pq ) x2 + 8 px + 3qx + 24 .
since the above polynomial is of degree 4 , there Combining like terms makes the equation (pq )
are at most 4 solutions to the above equation. x2 + ( 8p + iq )x + 24 = 6s2 + rx + 24. Matching
the coefficient of je5 on both sides of the equation
Detractor Explanations: Choices A , B, and C .
yields pq = Since p + q = 5 then the factors
are incorrect and are either the result of lack of offi that satisfy the equation must he 2 and 3.
knowledge of the n - Roots Theorem or the result Therefore, the possible values of p are 2 and 3.
of arithmetic errors.
Detractor Explanations: Choices B , C, and
D are incorrect and reflect errors in factoring
112. Level: Easy | Skill,''Knowledge: Nonlinear polynomials.
equations in one variable and systems of
equations in two variables|Testing Point: Solve
a quadratic equation 115. Level: Easy |Skill/ Knowlcdge: Nonlinear
equations in one variable and SysLcms of
Key Explanation: Ihe correct answer is 9. cqualiuns in two variables |Testing Point:
Ihe equation jri + J: - 90 - 0 can be factored as Solving a quadratic equation by ihe square root
fulluws: method
(x + 10)( x - 9) = 0 Setting both terms equal to
tccro yields Key Explanation: The correct answer is 4. Since
tF - Id = 0, then tF = 16 . Solving for d by getting
X = - I 0 or 9
the square root of both sides of the equation
Since t > 0, the value of c is 9 as it is the only
solution greater than zercc yields d = ± 4.
The question states d > 0, therefore, the answer
is 4
113. Level: Easy | Skill / Knowlcdge: Nunlinear
r

equations in one variable and systems of


equations in two variables|Testing Point: Use 116. Level: Easy | Skill / Knowlcdge: Nonlinear
rules of exponents to solve equation equations in one variable and systems of
equation in two variables|Testing Point ? Solving
Key Explanation: The value ofjc + y is 10 Using. systems of nonlinear equations in two variables
exponent rules for multiplication with the same with square roots
base, find that 2J X 2J = 2** r - l ,024 L Since, 2 =
1,024, 2* * r = 2inand thus JC + y = 10. Key Explanation: The correct answer is 196. To
find the value of y, substitute the value of x from
the 1st equation to the 2nd equation.
114. Level? Easy | Skill / Knowledge: Nonlinear
.
equations in one variable and systems of
equations in two variables | Testing Point:
Solving systems of linear and nonlinear
^
This yields 2 x 7 S = 2 y or 14

Group!ng the rad ical signs yields 14 se V2 =


equations in two variables

.
wwW vibrdntpublisbefS- tOftl 95

Telegram: SAT_pdf
I ADVANCED MATH

Dividing both sides of the equation by -J 2 yields


119. Level: Medium |.Skill / Knowledge : Nonlinear
I
equations in one variable and .systems of
i
* =& equations in two variables | Testing Point:
Squaring both sides of the equation yields 196 = binding solutions for absolute value equations
y or y = 196.
Key Explanation: Choice A is correct. Absolute
117. Level: Easy| Skill / Knowledge: Equivalent values are always positive. An absolute value
expressions|Testing Point!Solving for one equal to a negative value will have no solutions.
variable in terms ol another
Dislraetor Explanations : Choice B is incorrect.

Key Explanation; Since a + b = p + rcf, solve for r For an absolute value to have 1 solution, it
by isolating it. should be equal to 0 , Choice C is incorrect.
For an absolute value to have Lwo solulions, it
.Subtracting p from both sides of the equation should be equal to a positive number. Choice D
yields a + b - p — rq . is ineOrrecL. Equations involving absolute values
cannot have infinite solutions.
Dividing both sides of the equation by q yields
a + b - ft
<51
— — r. 120.Level: Easy | Skill / Knowledge: Nonlinear
Distractor Explanations: Choices A, B , and equations in one variable and systems of
C are incorrcoL and reflect errors in analyzing equations in Lwo variables | Testing Point:
multiple equations with the same variables . Finding the sum of the solutions in a quadra lie
equation

1IS. level: Medium | SkiU/ Knowledgc : Nonlinear Key Explanation : Choice C is correct. The
equations in one variable and systems of sum of solutions in a quadratic equation can be
equation in Lwo variables | Testing Point: Salving -b
nonlinear equation in one variable
found by the formula —a where a and b are the
coefficients of xF and x, respectively. Since a = 4
Key Explanation : The correct answer is 1 .
and b - -17 the sum of solutions is —17 = 17—
4 4
To find t, factor the given equation.
D 1stractor Explanations: Choices A and B ate
Factoring out U yields Zkik? - 1) = 0. incorrect. These are the solutions to the quadratic
Factoring the difference of two squares binomial equation . Choice L> is incorrect , This is the value
yields 2t(fr + l )(Jt - 1) = 0. of the product of the solutions which can be

Equating each factor to aero and finding the


solved using the formula — ,

value of k yields k = 0, k = -1 and k = 1. i .e. k =


{-1, 0, 1}. 121. Level ; Medium |Skill / Knowledge; Nonlinear

Since k is a positive integer, thenk - 1. equations in one variable and systems of


equations in two variables | Testing Point:
Finding the roots of a quadratic equations

Digital SAT Math Practice Questions 96

Telegram: SAT_pdf
I Nonlinear equations in one variable and systems of equations in two variables ( Answers )

Key Explanation: The product of the roots of j 123. Level: Medium | Skill / Knowledge: Nonlinear
I
a quadratic equation can be solved using the equations in one variable and systems of
formula —a where a is the coefficient of x and c is
2 equations in two variables |Testing Point:
Finding the solutions of a quadratic equation
-
the constant. Since a = 3 and c = 6 the product

of the roots is —3 = -2. If one of the roots is Key Explanation : Choice B is correct.
Expanding the left side of the equation yields 3(;d
—— , then.
3 HI s = -2. where s is the second

root of the equation. Multiplying both sides of


- 4x + 4) + 5 = 3x1 + 12x - 17.

Using distributive properties yields 3 jri - 12x +


the equation by -3 yields s = 6. 17 = 3*1 + L 2x - 17, which is equivalent to -12*
+ 17 = 12x - 17. Adding I 2x and 17 to both
sides of Lhc equation yields 34 = 24x. Dividing
122. Level: Easy | Skill / KnOwledgC: Nonlinear 17
equations in one variable and sysLems of
both sides of the equation by 24 yields —
= x or
equations in two variables | Testing Point: A - — . Since there is onlv one value of x, Choice
12
Solving a quadratic equation 13 is the correct answer.

Key Explanation: First determine the factors of DistractoT Explanations: Choice A is incorrect.
the equation to find Lhe solutions. The equation For an equation to have no solution , simplifying
y -—— features difference of two squares, the equation would result in a false statement.
4
which indicates that the equation can be factored Choice C is incorrect Linear systems cannot
into conjugates. have two solutions. Choice D is incorrect For a
system to have infinite solutions, simplifying the
Factoring the left side of the equation yields
equation would residL in a true statement.

HH) = 0 . Equating each factor to 0

yields b — — and b = —— , Adding the two factors

I+ f _ lhr 0
^ ,

.
wwW vibrdntpublisbefS- tOftl 97

Telegram: SAT_pdf
I ADVANCED MATH
I
The Law of' Universal Gravitation can he What is the value of x when

^
1
represented by the formula g = - - where is
^ - = (* - *-12)- ?
JC + 3
1 1

the acceleration due to gravity, G is a constant,


A) 3
M is mass, and R is the distance. Which of the
following gives the value of ft in terms of , G,
^
B) -3
and Al?
O 4

A ) fi D) 5

B) R=
GM

C) R - £S
G. LJ

5.-U
>
°* G If ti — —1 , then what is the value of the expression
27 "

—y ?

j[ x ) = [* - 4) ( r + 2 ) andgf *} = 4.s; - 17

if x > 0 then for what value of x is the statement


fix} - g ( jc ) = 0 true?

A) 6
B) &
C) -3

D) 3

Digital SAT Math Practice QuCStidns 98

Telegram: SAT_pdf
I Nonlinear junctions
I
Which of the graphs below most closely
represents the function y = 2X?

A)

71
U)

Q
St
/ \
T» it - -|--4t+|
-ftf| - --3H-|--2

7
Which equation below represents the graph in
the jfy-plane above?

Ihc formula for air resistance can be represented A) y = (jr1 + 5* - 6)(jf + 2 )


by the equation F = -cir*, where F is the force of R) y = (ar1 + 5* - 6)(* + 3}
air resistance, C Is the air constant, and v is the
velocity of the object , The negative sign indicates
C)
^y = ((*?= + 33* + 22H)(* +12))
that the direction of air resistance is opposite to D) =* + JC + X +
the direction of motion of the object - Which of
the following equations represents v in terms of
c and f ?

A ) v = cF 2

B) v —
C) V

D) v
7
wwui VT & rCrntpijW
^herittdri
} 99

Telegram: SAT_pdf
I ADVANCED MATH
I
A goiter hits; a golf ball at a driving range. If The formula for the break-even point of the
-
the equation h = t1 + 15f ( t = time in seconds ) manufacturing of a certain product is b
_L
represents the height of the hall ( h ) in feet, what s
is the height in feel four seconds after the ball where (it ) is the break-even point, (/) is the fixed
was hit? costs, (s) is ihe sales price, and (c) is the cost to
make each unit Which of the following correctly
expresses (s) in terms of ft,/, and &

A)
^ b+ c

f or the graph of a parabola with a positive



B) f
b+c
= —r
t
'

lead tucHicicnl, and a vertex at (2, 1) , which l -C


quadrant (s } does the graph not pass through ?

l. Quadrant TT D) *= c
ii. Quadrant TIT

iii. Quadrant TV

A} i

U ) i and ii
C) ii and iii

.
D) i ii and iii

If the x- intercepts of the graph of a quadratic


function are (-1, 0) and (5, 0), and the graph
has not been dilated or reflected from the parent
function , what is the distance between the points
on the graph intercepted by the line y = 7 ?

A) 3
B) 8
C) 9

D) 10

Digital SAT Math Practice Questions 100

Telegram: SAT_pdf
I Nonlinear junctions
I
S Which of the following Ls a value for b for which
4 b1 + 30 b + 52
the expression is undefined ?
3
b1 -169

: A) 2
i B ) 13

-5 -4 -3 -1 -1 0 1 2 4 5
C) 0
V,
A ) -2
-2
-3
4

Which of Ihe following could be Lhe equation of


the graph above? The range of the function g{ x ) is the set of real
9
numbers greater Lhan or equal Lu — . If the

A ) x2 (JC - 3 )3 - 1
zeros of the flint Lion ( rc ) are -3 and 0, which of
B} jefje +
>
3 s -1
the following could be ^ fir) ?
C) x* { x - i y
^
A ) (jt + 5) fr - 2) + 1
D)
^ (* + 3 )3 + l
B ) (i + 5)( x - 2) + 10
C) (x + 5) ( x ~ 2) + 5
D ) (x + 5) ( x - 2) + 2

1
In the equation a + 3 =^ which of the
a+3
following is a possible value for a + 3 ?

A) l

B) 3
C) -3
D) 2

wwwi vibrdntpublisbefS-tOftl 101

Telegram: SAT_pdf
I ADVANCED MATH
I
The formula for specific resistance can be If 6 X t?1-1-1 = 1296, what is the value ofs?
expressed as R = where p is a constant
A) 1
and A is the cross- sectional area. Which of the
following gives A in terms of R andp? B) 2

I Q 3
A) A = fl -
1' D) 4

C ) A pR —
1
D) A- —
ps.

mi
Jt*) - ax
1
- 22

Tor the fimtlion j:) , a is a constanL Mj{ -2 )


^
- 14, what is the value offl 3 )?
-

A ) -2
R) -4

C) 2
D) 4

mi
For the function *) = 4 x + Xs - ] 2 , what is the
^^
value of (*) when x) is at its minimum?

Digital SAT Math Practice Questions 102

Telegram: SAT_pdf
I Nonlinear junctions
I
Which of the following graphs below best If fc( n ) = n 3 - 8, what is the value of n when h( nj
represents the equation y = 3* - 2 ? = 19?
rS '

-7- A) 3

-- 5
A-
/ B ) -R

A)
i
/ C) -2
D) 9
1
1 l'

f 10

- 21
-7-
B
na
— -S -
-S-
i
The population of a town is represented by p( f ) -
3102(1.012)J, where ( is the number of years J3y
what percentage does the population of the town
.
B}
grow every f years ?
i HI
A ) 12%
B ) 3402%
C) 10 U%
—r ® '

D) 1,2%
i

4.

C)
— -i

7 !i 7 S 9 10

=s j
Which of the following shows an equation of a
parabola in vertex form ?

3
2'
A) y = 3
^ + &c + 14
B) y = 30c1 + 55)

D)
-U . I : Li
^
C) y = 3 + 55

—— -- \
- D ) y = 3( x + 2)* + 13

m
3
A

.
wivW vibrdntpublisbefS- tOftl 103

Telegram: SAT_pdf
I ADVANCED MATH
I
What is the positive value of Jt in the equation 3
helots’?
If / U) = -*+ 2 anti f { x ) = 3 , what is the value
of U) i ?
I -T + 4| = S
« 1
:
“> i
c) 3

If Or) - lx3 - 25 x 4 20 is a function in X, fur what DJ 2


^
value of x, (x) reaches its minimum value?
^
A ) 20
25 4 v'4fi5
B)
4
25
C)
-1 What is the value of 4 -Jd J 43 when .v - 0?
D) 25
A) 6
13) 0
C) 3
D) -3

Iff — 2, what is the value o f g?

If |x + 31 = —
:l
, what is the sum ut all values of x?

Digital SAT Math Practice Questians 104

Telegram: SAT_pdf
I Nonlinear junctions
I
Tf x) = X2 + x, which of the fallowing
^
expressions is equal to fix + 2)?

A ) ( x + 4) ( x + l )
B) ( x + 2 ) ( x + 3)

C) Xs + 5JC - 6
D) x* + 2x + 2

Which uf the following equations, when graphed


on the ( x, y ) plane above, expresses the following
function in vertex form ?

1 5
AA *> T -i
'
B ) / = (i + l}1 - 4 *(*) = *" +- 13^ + 10^ 4 216
gM
C) y = (* - l )(x + 3) Which of the following is equivalent to
/w
J
D) y M x + l ) + 4 x* + 12x + 3G
A)
x -6
1
B)
x -6
1
Q
x +6
What is the value of -jc ifx > 0 andjfa) - D > ( x + 6)1*

xl + X-12
when fix ) = 25 ?
( *+ 4 )

How many real solutions does y = 2a? - 3x +- 5


have?

itmt!vibrtntpublishtXS.COrtl 105

Telegram: SAT_pdf
I ADVANCED MATH
I
Every year, Hon adds twice the number of model Hnw many integer solutions does the equation
airplanes to his collection from the year before. below have ?
Which of the following type of function best
|JC + 3|< 4
describes the number of model airplanes Ron has
in his collection with respect to time? A) 0
B) -6
A} Increasing exponential
C) 7
B) Decreasing exponential
D) 2
C) Increasing linear
D) Decreasing linear

A substance has a mass of 312gand a half- life


of 120 days. iLs half-life is given by the equation
Tf a parabola represented by the equatinn _/{x} j
( lW
= -2 ( + 2)3 + 12 is translated Log( jr ) = -2x' +
jt , where d is lime in days. If the
Ax + 14, which of the following statements best
equation is rewritten using m months as a time
describes the horizontal transformation ?
interval, which of the following besi represent
the equaiiun of the half-life? ( Assume that every
A ) Tire fantfiem moves 3 units to the right.
month has 30 days )
R ) The fanction moves 3 units to the left
( 1 't 13
C) Tire function moves 4 units left - A) p( m ) = 312 f - l "
:
D) The funelion moves A units right I !

B)

C)

D)
p(m ) = 3l 2

^ j*
HI

Digital SAT Math Practice QuCitiOns 106

Telegram: SAT_pdf
I Nonlinear junctions
I
Wliat is the -intercept of the exponential Chemical H' was observed and the researcher
-^- -
equation y (6)1 1 ?
tame
J
up with the model p(d ) = 300(3) where
p( d ) is the mass of chemical w alter d days.
ii
Which statement would best describe (3)1 ?

A ) The mass of chemical w triples every 5 days ,

li) The mass of chemical w triples every d days


A phone company creates a model that shows C) The mass of chemical w is five limes its
the number of phones they have sold in the past initial mass after 3 days.
If ) years. The number of phones sold in the first
year was 1,600,000, The number of phones then D ) The initial mass of chemical w.
increased by 72% every year after thaL. Which of
the following best represents the model equation
uf the number of phones Sold by the cumpany
where L is the number of years?

A) p ( / ) = 1,600,000( 0,72)'
A botanist observes the growth of a tree fur 25
B) p ( / ) = 1,600,000(1,72)'
years and comes up with the model in meters.
C) p ( t ) = 1 ,600,00« l( 0-2g )‘
Ihe function is /i(f ) = 2, 3(1,12)J, where t is
D } p (f ) = 1 ,600000(1 -72 the number of years. Which of the following
sLaLcmcnls isnol true aboul Lhe model?

A ) The initial height of the tnse is 230cm .

11) The height uf the tree increases by 12%


annually,

The revenue of a consulting firm is described C) The height of the tree after the first year of
by the model ( x ) shown below, where x is the observation is 2.576m
^
number of years and 0 x 9- Which of the D) The height of the tree increases by 112%

^
following best describes } = 40,140,8?
fix ) = 32,000 (1.12)*
annually.

A ) The revenue increases by $ 40,140 ,8 after 2


years.
B) The revenue of the consulting firm was
$40,140.8 at the beginning of year 2,

C) The revenue of the consulting firm is


$40,140.8 at the end of year 2,
D) The revenue of the consulting firm decreases
to $40,140, 8 after 2 years.

wwW, vibrdntpublisbefS-tOftl 107

Telegram: SAT_pdf
I
124. Level: Medium |Skill / Knowledge: Nonlinear
ADVANCED MATH

Key Explanation: Choice D is correct. Making


I
functions|Testing Point: Solving for one the right side of the equation have positive
variable in terms of another 1 1
. Cross
Key Explanation: Choice A is correct .
" ' "' T- JV *- U)
multiplying to eliminate the fractions yields ( x1
Multiplying both sides of the equation by R 2 - x - 12) = x + 3. factoring the left side of the
yields = GM. Dividing both sides of the equation yields ( x - 4) ( x + 3) = ( x + 2 ). Dividing
1 both sides of the equation hy ( x + 3 ) yields x - 4
equation bygyields -K ' = . Getting the
= 1 . Adding 4 to both sides of the equation yields
square root of hoth sides of the equation yields x - 4 + 4 = 1 + 4. Therefore, x = 5.

Dislractor Explanations: Choices A and U are


incorrect and may rcsidl from multiplying -1
Dislractor Explanations: Choices R, C, and
by the trinomial . Choice C is incorrect and may
D are incorrect as the result of incorrectly
rcsidL from solving x from the equation (x - 4) =
multiplying and dividing variables.
0-

125. level: Medium|Skill/ Knowledge : Nonlinear


127.Lcvd Hard|Skill / Knowledge: Nonlinear
functions|Testing Point: Performing operations
funetions | Testing Point : Evaluating equations
with functions
with exponents
Key Explanation : Choice D is correct. To find
Key Explanation : To evaluate the given
the value of x that makes fix ) - - II, plug
expression , make the base the same. Converting
each equation uf / and ", This yields ( x - 4) ( x +
2) - ( -lx - 17 ) = 0. Using the distributive property 27* yields (3J )* or 3311- Simplify the expression
yields JC1 - 2x - 3 - ix 4 17 = 0. Combining like 33" by subtracting the exponents which yield
terms yields x3 - Cut + 9 = 0. Factoring the perfect
3*” or 3*\ Substituting the value of a yields 3![“ >
square trinomial yields (JC - 3)J , Equating the
or 31. Therefore, the expression is equal lo 3.
factor to zero yields x - 3 = 0. Therefore, x 3.

Distractor Explanations; Choice A i.s incorrect 128. Level: Easy | Ski 11 / Knowledge: Nonlinear
and may result from adding the values of x from functions | Testing Puint : Determining a graph
the two factors. Choice B is incorrect and may of an exponential function given its equation
result from a conceptual or calculation error.
Choice C is incorrect and may result from nut Key Explanation : Choice R is correct . The
equating the factor to zero in solving for x. equation y = 21 is an exponential function , and
the graph in Choice B is the only graph that
features exponential growth .
126. Level; Hard | Skill / Knowledge; Nonlinear
functions|Testing Point: Solving rational
Distractor Explanations: Choice A is incorrect
equations
because it features a graph of a linear function.
Choice C is incorrect because it features a

Digital SAT Math Practice Questions 108

Telegram: SAT_pdf
I Nonlinear functions ( Answers)

graph of a cubic function. Choice D is incorrect 131. Level: Easy|SkiJJ / Knowledge: Nonlinear
I
because it features a graph of a quadratic functions | Testing Point: Using a quadratic
function. function

Key Explanation : 44 is correct . 'Ihe height of the


129. Level: Easy | Skill / Knowledge: Nonlinear
ball after (0 seconds can be found by substituting
functions | Testing Paint: Solving for one
4 seconds for ft ) in the equation:
variable in terms of another
h = -[4)1 + 15( 4}
h = -16 + 60
Key Explanation: Choice C is correct. In order
h = 44 ft
to rewrite the equation in terms of v, one must
isolate that variable on one side of the equation :
F = -cv1 L32. Level: Easy | Skill / Knowledge: Nonlinear
functions | Testing Point: Creating a quadratic
and interpreting its Location oil a graph

Key Explanation: Choice C is correct . A


parabola with a positive lead codTitienl opens
Detractor Explanations: Choices A , B, and D
upward. Since Lhe vertex of the given parabola is
are incorrect because when they are rearranged

to isolate F, they do not equal. F -cv3 .
.
at ( 2 1) Lhcn Lhc lowesL value of the graph is at y
= 1. Ihis means that the graph does not go below
y V . 'Iherefore, Lhe parabola does noL pass

130. Level: Easy | Ski 11 / Knowledge: Nonlinear through the quadrants b e l o w y = 0 or the x-axis
functions | Testing Point: Making connections which are Quadrants TIT and TV
between algebraic equation representations and
a graph DLstractur Explanations: Choices A, B, and
D arc incorrect and may res nil from an error
Key Explanation: Choice A is correct. To in interpreting the data in the equation of a
determine the equation of the polynomial in the parabola.
graph , recognize that all answers are written in
some version of intercept form . Then identify
133, Level ; Hard | Skill / Knowledge: Nonlinear
the x-intereepts of the graph, which lie at (-6, 0),
functions | Testing Point: Creating quadratic
(-2, 0), and ( 1 0), The equation of the graph can
h
equation and using transformations
-
then be written as y - { x + 6)(* l )(x + 2) . This
answer can be simplified as y = (x1 + - 6)(JE + Key Explanation; Choice R is correct. To find
2) by multiplying the first two factors together.
where the function intercepts the function y -
7 , first write the quadratic equation using the
Distractor Explanations: Choices B, C, and D
given x-i intercepts. Using the intercept form, the
are incorrect and may result from using wrong
_r- intercepts. equation b e c o m e s y = ( x + ] ) ( x - 5) , Substituting
the value of y yields 7 = (x + ])(*- 5). Using
the distributive method on the right side of the

wwW, vibrdntpublisbefS- tOftl 109

Telegram: SAT_pdf
I - -
equation yields 7 = x1 4x 5. Subtracting 7
ADVANCED MATH

Key Explanation: Choice A is correct. The


I
- -
from both sides yields 0 = jr1 Ax 1 2. Factoring graph crosses the x axis in 4 places and thus
the equation to find the values of x when y = 1 has 4 solutions. Tt is then a quartic equation of
yields 0 = (x + 2} ( x - 6). liquating both factors to highest power x1. This eliminates Choice U as
zero yields x = -2 and x = 6 . The two points that when this expression is multiplied out it yields a
intercept the other function are then (-2.7) and highest power of 3. The graph crosses the x axis
(6, 7), so the distance between the two points is at approximately -0.5, 0.5, 2 , 5, and 3, 5 and thus
6 - ( - 2) = 8 . has zeros at these points. This eliminates Choice
C as it has zeros at x = 0 and x - 3, The function
Distractnr Explanations: Choices A , C, and
D are incorrect and reflect errors in finding
-^
y x x - $ ¥ -1, or Choice A, has a vertical shirt
down 1 from the graph nfjdix - 3 )*. That would
solutions and intersections between two move the graph 1 unit below the x axis from the
functions. parent function whose lowest point would he at
thex-axis. This matches the graph . Choice D

134. Level: Easy| Ski I I / Knowledge: Nonlinear would move the graph one unit up which is not

functions|Testing Point: Solve tor one variable represented in the diagram.


in an equation
Dislraclor Explanations: Chokes fl, C, and D

Key Explanation: Choice A is correct. To solve are incorrect and reflect errors in interpreting the

for s, add c Lo bolh sides: graph as stated above.

b
^ -
s L36. Level: Easy | Shill / Knowledge: Nonlinear
functions | Testing Point: Solve a quadratic
b+ c= L
s equation by square root method

Then divide both sides of the equation by b + c;


Key Explanation: The first Step in solving
( & + <;)
* =/ the equation a + 3 = ——
a +3
is through cross

multiplication . This yields (a + 3 )3 = 1 . Taking


b+ c
the square root of both sides of the equation
This gives Choice A as the correct answer,
results in (CM- 3 ) = ± 1
Distracior Explanations; Choices B, C, and Only -1 is in the answer choices. Thus, the
D are incorrect and are the result of errors in correct answer is Choice A,
making math mistakes in rearranging equations
Distracior Explanations: Choices B, C, and
with multiple variables.
D are incorrect and are the result of arithmetic
errors.
135. Level: Medium | Skill / Knowledge; Nonlinear
functions|Testing Point; Make connections
between graphs and their algebraic
representations

Digital SAT Math Practice Questions 110

Telegram: SAT_pdf
I Nonlinear functions ( Answers)

137. Level: Medium | Skill / Knowledge: Nonlinear 139. Level: Easy | Skill / Knowledge: Nonlinear
I
functions | Testing Point: Use difference of two functions | Testing Point: Solving for one
squares and knowledge of rational expressions variable in terms of an other

Key Explanation: Choice J) is correct. A Key Explanation : Choice B is correct. In order


rational expression is undefined where its to express the equation in terms of A, isolate
denominator is equal to ztro. thus, in this variable A on the left side of the equation.
problem where b1 - 169 = 0. Using the difference
Multiplying both sides of the equation by A
of two squares fd - 169 is equivalent to ( b - 13)
yields RA = p.
( b + 13). Selling ( b - 13) and ( f > + 13) each equal
to 7ern results in b = -13, +13. Only b = 13 is Dividing both sides of the equation by R yields
an answer choice. Thus, Choice B is the correct
i -:
answer.
Therefore, Choice B is the correct answer.
Dislrae Lor Explanations: Choices A, C and
DLsLrac lor Explanations: Choices A, C, and D
D are incorrect and are the result of arithmetic
are incorrect because when they are rearranged
errors in selling Lhe denominator equal to lero.

138. Level; Hard | Skill / Knowledge: Nonlinear


to isolate R, they are not equal to R —p — ,

140. Level : Easy | Skill / Knowledge: Nonlinear


functions | Testing Point: Determine the
functions | Testing Point: Solving for a variable
function from the zeros and Lhe range
in a quadratic function
Key Explanation: Choice B ts correct. Test the
answer choices by expanding out each quadratic Key Explanation: Choree B is correct. Start by
function in the answers to see which yields 7eros determining lhe value for a by replaeing x with
of -3 and 0. Expanding out Choice B yields (x + _
-2 and ftx) with -14.
5)(JC - 2) which expands to jd + 3x - 10. Adding This yields -14 = a( -2 )2 - 22,
the 1 (J to this expanded polynomial results in x3
Simplifying and adding 22 to both sides of the
+ 3x. Factoring out an x results in X(JE +- 3) and
setting equal to zero yields the solution , or zeros* equation yields 8 = a ( 4).
ofx = 0 and x = -3, or Choice B. This answer Dividing both sides of the equation by 4 yields 2
choice also results in a vertex of the parabola = a or a = I .

——
{-3 -s']
of | , I , Since the parabola opens up, its Replace a with 2 and x with 3 to find the value of
.
fl 5 ) which yields
minimum value is —
-9
which makes the range of
3) = 2(3)* - 22 = 2 x 9 - 22 = 18 - 22 = -4.
4
gfx) greater than or equal to
— 4 ^
4 DLstractor Explanations: Choices A and D
Detractor Explanations; Choices A, C, and are incorrect and reflect errors in arithmetic
D are incorrect and are the result of errors or solving algebraic equations. Choice C is the
in factoring polynomials to find solutions or value of a and reflects a misunderstanding of the
knowledge of parabolas and their equation forms. question .

.
wwW vibrdntpublisbefS-tOftl 111

Telegram: SAT_pdf
I
141. Level: Hard|Skill / Knowledge: Nonlinear
ADVANCED MATH

Detractor Explanations: Choice A is incorrect.


I
functions|Testing Point: Working with different The graph of option A is linear. Choice C is
forms of parabola equation and vertex incorrect. They - intercept of the graph is 2.-
Choice D is incorrect. This is a decreasing
Key Explanation: Choice A is correct. When exponential graph, however, the given equation
graphed , the vertex of the parabola is dearly the requires an increasing exponential graph .
minimumjfx ) value. To determine the vertex,

^
rewrite the equation in vertex form x ) = ( x -
hy + k where [h , k ) is the vertex. Completing
144. Level: Easy|Skill / Knowledge: Nonlinear
functions|Testing Point: Solving a cubic
the squares and grouping the perfect square
function
trinomial y i e l d s = (jr* + Ax + 4) - 1 ft.
.Simplifying the equation yieldsj{ x ) - { x + 2 f -
Key Explanation: Choice A is correct. To Ond
16. Hence, Lhc vertex is (-2, -16). Therefore, =
the value of H, equate the equations to each other
-2 when /fa) is at its minimum.
which yields 19 = w3 - fi . Adding ft to both sides
of the equation yields 27 = n 5 or n 3 - 27 . Getting
142. Level: Easy|Skill / Knowledge: Nonlinear the cube root of both sides of the equation yields
functions|Testing Point: Using exponent rules n —3
.

to solve nonlinear function


Disiraclor Explanations; Choice E is incorrect.
Kcy Explanation: Choice D is turned. Using the This is the y-inlerccpL of the function h ( n ).
exponent rules for multiplication, the left side Choice C is incurred. This is the eube rout of
of the equation can be simplified by adding the the y-intercept. Choice D is incorrect . This is the
exponents. This yields 6 x 6* L = 1
-6s- 12%. value of H .
"

Tt follows that s = 4 because 61 = 1296-


145.Level: Medium|SkiJJ / Knuwledgt: Nonlinear
Distraclur Explanations: Choices A, B , and
functions|Testing Point ; Finding the growth
C are incorrect and art the result of errors in
rale and growth factor
solving equations and exponents.

Key Explanation : Choice D is correct - The


143. Level: Hard |Skill / Knowledge; Non linear growth factor is 1.012 from the given equation .
functions|Testing Point : Matching a graph to its The growth factor is increasing since it is greater
exponential equation than 1 . To find the growth rate, calculate the
value of T in 1 + r = 1.012. Subtracting 1 from
Key Explanation ; Choice R is correct . The both sides of the equation yields r = 0, 012 or r =
equation y - 3“ - 2 , is an exponential growth 1.2% in percentage. This would be the percentage
function. Hence, the possible answer is either increase in the population of the town every f
choice R or C, To determine which one is years.
correct, find the y- intercept of the graph by
substituting 0 with x . This yieldsy = 3s - 2 ory = Distractor Explanations: Choice A is incorrect
- 1 . Since the graph in Choice E Intersects the y - and may result from an error in converting the
axis at (0, 1), it is the correct answer.

Digital SAT Math Practice Questions 112

Telegram: SAT_pdf
I Nonlinear functions ( Answers)

decimal to a fraction . Choice B is incorrect. The (-25) 25


I
yields = . Therefore, JC =
value 3,042 is the initial population . Choice C is 2( 2 ) ^
incorrect. This is the growth factor in percentage
Distractnr Explanations: Choice A is incorrect
form .
and may result from getting the value ofg when
x - 0. Choice B is incorrect and may result from
146. Level!Easy | Skill.' Knowledge: Nonlinear getting the values o f x when = 0. Choice D is
functions | Testing Point: Recognizing the vertex
^
incorrect and may result from a conceptual or
form of a parabola calculation error.

Key Explanation: ChoiceD is correct. The


149. Level: Easy |Skill / Knowledge : Nonlinear
equation of a parabola in vertex form isy = a{ x -
functions | Testing Point: Solving square root
hY + w h o r e (h , It ) is the vertex. Option D is in
funclions
the vertex form . The vertex of this line y = 3( jr +
2 + 13 is [-2, 13 ).
^ Key Explanation: The correct answer is

17. Substituting f 2 to the equation yields
DislracLor Explanations: Choices A , B, and C
^ jg + 3 = 2 \/5
are incorrect. These equations of a parabola are
( ) - Squaring both sides of the
equation to eliminate the radicals yieldsg +
written in standard form.

3 4(5). Subtractings from both sides of the
equation yields g - 4( 5) - 3 or g - 17.
147. Level! Easy | Skill / Knnwledge: Nonlinear
functions | Testing Point: Solving absolute value
150.Level : Medium | Ski 11 / Knowledge: Nonlinear
equations
functions | Testing Point; Solving an absolute
Key Explana lion: To solve for the positive value value equation
ifx, equate the contents of the absolute value
(
Key Explanation: The absolute value indicates
to the positive value of the constant. This yields
that then: are two solutions to the equation,
x + 4 = 8. Subtracting 4 from both sides of the
equation yields x = 4, which can be found by creating two equations
as follows x + 3 = —5 and x + 3 = - —5 . By
148. Level: Hard | Skill / Knowledge: Nonlinear subtracting 3 from both sides of the two

^
functions | Testing Point: Solving quadratic
equations
equations yields x = - - 3 and x = ——; - 3
4
Simplifying the two equations yields *= - -
Key Explanation: Choice C is correct. To find
26
the minimum value of x in a quadratic equation, and x = - — Adding the two values of x yields
,

use the formula x . , Comparing the


4 26 .' I '
2a —+ or
5 5
-
"

given equation gfx) = l x1 25x + 20 to the


general form equation ax1 + in; + c = 0 yields a =
Therefore, the sum of all the values of x is 6, -
2 and b = -25, Substituting the values of a and b

.
rnvW vibrdntpublisbefS- tOftl 113

Telegram: SAT_pdf
I
] 51. Level: Hard | 5kill / Knowledge: Nonlinear
ADVANCED MATH

Key Explanation: Choice B is correct . The graph


I
functions|Testing Pointi Evaluating a linear shown represents a quadratic function . To create
function for a value and then determining the the quadratic equation , use the vertex form y k -
value of a rational function - (x - fd ) where (ft, k ) is the vertex of the graph ,
'therefore, when (-1, -A) is substituted into the
Key Explanation: Choice A is correct. First, use vertex form equation, it yields y + 4 = ( x + l 1),
the information provided to determine the value 'this is equivalent to y = (JC + l )2 - 4.
ofx, before finding the value of (*}
Combining the two equations yields +2 =3 Distractor Explanations: Choice A is incorrect
because it represents a linear equation , not a
Subtracting 2 from both sides of the equation
quadratic equation. Choice C is incorrect as it is
yields - r = l . Multiplying 2 and dividing 3 from in intercept form, not vertex form. Choice D is

both sides of the equation yields x= — . Hence, incorrect because iL includes the opposite sign for
the y-value of the vertex.
ur' = -x 42 .
154. Level: Hard|5kill / Knowledge: Nonlinear
Distractor Explanations: Choice B is incorrect
ftinctions|Testing Pninti Solving a rational
because ii is Lhe value of x, Choices C and D are
incorietL and may result from a conceptual or
equation

calculation error.
Key Explanation: Substituting the value of
-n f
'
^
152. Level: Easy|51cill / Knowledge: Nonlinear j[ x ) to equation j{ x ) = x + x yields
( x + 4)
hinctions|Testing Pnint: Evaluating a quadratic
x’ + x -llV
expression given an inpuL value 25 = . Getting the square
U+ D
Key Explanation: Choice A is correct. Begin root of both sides of the equation yields

— 1 into the expression


by substituting x (

+ 5 w hich yields (o + s/J) + 3 ,


V25 =
v
x1 + x -12
X+4 ^ or ±5 =
3
x + x -12
( x + 4)
Factoring the right side of the equation yields
Simplifying the expression yields >/1^ + 3 or 3 +
( x + 4)(* - 3)
3, Therefore, the value of the expression is 6. ±5 = or ±5 = x — 3 . Equaling the
X+4
right side of the equation to both values of the
Distractor Explanations: Choices B, C, and D
left side yields 5 = x - 3 and - 5 = x - 3, Solving
are incorrect and may result from a conceptual
for x in both equations yields x = S and x = -2.
or calculation error.
Since x 5 0, then x = S, Therefore, the value of -x
is -3,
153. Level: Easy|Ski I I / Knowledge; Nonlinear
functions|Testing Point: Creating quadratic
155, Level; Easy|Skill / Knowtedger Nonlinear
equation from its graph
functions|Testing Point : Using the discriminant
of a quadratic equation to find the number and
type of solutions

Digital SAT Math Practice Questions 114

Telegram: SAT_pdf
I Nonlinear functions ( Answers)

Key Explanation: The answer is 0. To find the ( a + ft ) (a


I
- i ) where a = x and b = 6 resulting in
?

number of solutions a function has, use the fix ) = ( x + 6 ) ( x 6).-


-
discriminant formula b2 4 ac. In this function a
The numerator is difficult to factor as it can’t be
= 2, ft = -3, c = 5. The discriminant therefore is factored by grouping. The easiest way to factor
(-3)1 - 4( 2 )(5) = 9 - 4 = -31 . it is to assume that far + fi ) might he a factor of
If the discriminant is a negative number, this it since it is a factor of the denominator. Since
indicates that the function has no real solutions. all terms in the numerator are positive, - (5 )
cannot he a factor of the numerator. Using long
division or synthetic division, divide [x + 6) into
156, Level; Easy | Skill / Knowledge: Nonlinear
the numerator. This results in JT +12x + 36. This
funclions | Testing Point: Evaluating a nonlinear
expression is easily factored into (* + 6) (* + 6) ,
function giver an algebraic binomial input value
Cancelling out like terms between the
Key Explanation: Choice E is correct. Begin by numerator and denominator results in
substituting (JC + 2) for xin the expression which _ (* + $)* _ (* + 6) J (* + &y
yields ( x + 2)2 + [ x + 2 ). - 36 (*+ 6)U -6) x -6
Using distributive property to simplify the jf +U x + l f i
or Choice A.
expression yieldsx? + 4x + 4 + x + 2. x -6

Combining like terms yields ad + 5JC + 6. OistractoT Explanations: Choices B, C, and D


factoring the expression yields (JC + 2)(x + 3). are incorrect and are due to errors in fat Luring or
simplifying complex funclions.
Di si tract or Explanations: Choice A is incorrect
and may result from only substituting far + 2 ) to
x2 . Choice C is incorrect and may result from an 15#. Level : Easy I Skill / Knowledge: Nonlinear
error in simplifying the expression. Choice D is functions | Testing Point: Recognize nonlinear
incorrect and may result from adding JT 4- x to x equations
+ 2.
Key Explanation: Choice A is correct. Ron is
adding planes to his collection at a doubling rate
157- Level: Medium | Ski 11 / Knowledge: Nonlinear each year, Since his collection does not increase
functions | Testing Point: Simplify rational at a constant rate each year, this is considered an
expressions exponential increase,

Key Explanation: Choice A is correct. First DLslractor Explanations: Choices B, C, and


substitute the algebraic expressions for in D are incorrect and are the result of errors in

^
the numerator and fbr x) in the denominator
of the fraction. Factor both numerator and
interpreting functions.

denominator as follows;
159, Level : Medium | Skill / Knowledge: Nonlinear
The denominator can be factoring using the functions | Testing Point: Translating quadratic
difference of two squares a1 - b1 = equations

mrW, vibrdntpublisbefS-tOftl 115

Telegram: SAT_pdf
I Key Explanation : Choke A is correct. The
ADVANCED MATH

Key Explanation: Choice D is correct. Every


I
-
vertex offix ) is ( 2, 12) . The equation nfg( jt) = four months or 120 days the mass of the
-2*1 + 4JC + 14 in vertex form would he substance is halved. Choice D is the only
-2 (JC - l )1 + 16 . Hence, the vertex would be
, equation that would result in a half-life of 4
(1, 16). lhe x coordinate translation from -2 to 1 months.
is 3 units to the right.
Distractor Explanations: Choice A is incorrect.
Distractor Explanations: Choice B is incorrect This option suggests that the substance has a
and may result from an error in interpreting half - life of 120 months.
the direction of translation . Choices C and D
Choice li is incorrect. Uhis option suggests that
are incorrect. These options represent vertical
transformation.
the substance has a half - life of 1 day or —
months
Choice C is incorrect, this opLiun suggests that
160. Level: Hard |Sic ill / Knowl edge: Nonlinear
the Substance has a half-life of 30 months
functions|Testing Point: Finding the solutions
of an absolute value inequality
162. Level: Medium |.Sitill / Knowledge: Nonlinear
Key Explanation: Choice C is correct. To functions|Testing Puinl: Finding Lhey-
solve for Lhe absolute value linearly, use two inleroepL
inequalities for the positive and negative values .
Key Explanation : They-intercept of a line is
#
|x + 3|< + 4 |JC + 3|< ^ 4 given when x - fl. Substituting 0 for x in the
JC +3<4 -x - 3 < 4 —
given equation yieldsy = ( 6 )“ - 1. Simplifying
-x < 7 the equation yields -2.
JC <1
JC >-7
163.Level: Easy|Shill/ Knowledge: Nonlinear
Thus, -7 < JC < 1. Since the value of JC is between functions|Testing Point; Creating an
.
-7 and 1 the solutions to Lhe above inequality exponential function from data
are 0, - 1, -2 . -3 -4 -5. -6. Therefore there are 7
, , ,
solutions to the inequality above. Key Explanation: Choke B is correct , the
equation is an exponential growth function .
Distractor Explanations; Choices A , E , and If The equation isy = a (1 + r)' where a is the
are incorrect and may result from a conceptual initial amount , r is the growth rate and x is the
or calculation error time interval. Since the model is increasing by
72% then r = 72%. The growth factor can be
161 , Level: Hard | SLill / Knowledge; Nonlinear written as ( 1 + r) = ( ] + 72%) = L72. Since the
functions | Testing Point: Changing units of rime initial number of sales is 1,600,000 then ft =
in an equation 1,600,000, Since the rime interval is in years , then
x = f . Therefore, the equation of the model is
p[ t )=1,600,000(1,72)',

Digital SAT Math Practice Questions 116

Telegram: SAT_pdf
I Nonlinear functions ( Answers)

Distractor Explanations: Choke A is incorrect. Distractor Explanations: Choice B is incorrect.


I
This model represents a function that is The mass does not triple every d days because the
decreasing by 2fi%. Choice C is incorrect. This time interval d is divided by 5 in the equation.
model represents a function that is decreasing Choice C is incorrect . Since the growth factor is
by 72%. Choke D is incorrect. fIhc function is 3, the mass triples per lime interval. Choice D is
conceptually wrong because the time lac Lor used incorrect. This initial mass of chemical w is 300.
does not align with the situation ,
16*. Level : Easy | Skill / Knowledge: Nonlinear
1 4. Level: Rasy | Skill / Knowledge: Nonlinear
* functions | Testing Point: Determining the
functions | Testing Point : Interpreting value of meaning in context of terms in nonlinear
nonlinear function function

Key Explanation: Choice C is correct. The Keyr Explanation ; Choice D is correct. Since the
model ( x} describes the revenue of the
^ -
growth factor is 1.12,the growth rate r 1.12 - 1
consulting firm after f years. Since f - 2 in f [ 2 )t = 0.12. Hence, the height of the tree inc reases by
$40,14 tl.fi represents the revenue of the firm at 12% annually and not 112%. Therefore, Choice
the end of year 2. D is false.

Distractor Explanations: Chokes A , En and D Distractor Explanations: Chokes A, B, and C


arc incorrect. These statements are false. are incorrect. 'Ihese are true statements about the
model given.
165. Level: Medium | Skill,'Knowledge: Nonlinear
functions | Testing Point: Evaluating meaning of
term in nonlinear funcliun

Key Explanation: Choice A is correct - Using


the given function, the mass of chemical ic after

5 days will be P( d ) = 300(3)" = 300( 3) = 900-


This new mass is thrice the initial mass of 300.
Therefore, the function indicates that the mass of
chemical w triples every 5 days.

.
wwW vibrdntpublisbefS- tOftl 117

Telegram: SAT_pdf
This page is intentionally left blank

Telegram: SAT_pdf
Chapter 5

Problem-Solving and
Data Analysis
This chapter includes questions on the following topics:
Ratios, rates, proportional relationships, and units
* Percentages
* One - variable data: distributions and measures of center and spread
* Two-variable data: models and scatterplcts
* Probability and conditional probability
* Inference from sample statistics and margin of error
* Evaluating statistical claims: observational studies and experiments

Telegram: SAT_pdf
I PROBLEM - SOLVING AND DATA ANALYSIS
I
Doug's car holds 50 gallons of fuel. If he drives At Smart Pets, 60% of the 20 dogs have blue eyes.
150,000 miles at 35 miles per gallon, how many The fraction oi hamsters at the store with brown
full tanks of gas will he use through his journey if eyes is equal to the fraction of dogs at the store
he starts with a full tank ? with brown eyes. What is the ratio of hamsters
that have brown eyes if the dogs only have hrown
A ) 10 and blue eyes?
R ) 25
C) 72
D) 136

L 2 Carpenters lake 3 days to build 36 desks for


a local school . How many days will it take tor 9
U2 carpenters to complete 63 desks if all Carpenters
work at the same raLef
Tf two pounds of grapes cost $3.06, how much
will g pounds of grapes cost ?
A) 1

A ) 3.9% 13) 5

f 3.96 A C) 6
ITJ*
C) 3-96 + g
D) 7

3.96
tf

A square field has a side length of 2.5 feel. If a


fence, which is 36 inches away from either side
of the field is created, what is the perimeter of the
There are 36 students at a statistics camp. If two- fence created in inches? ( L fool = 12 inches)
thirds of the students are giris and three-fourths
of the girls are under 5,5 feet, how many girls are
under 5,5 feet tall?

Gina's age is 3 times her daughter 's age. Five years


from now, the ratio of Gina’s age to her daughter's
age would be 7:3. How old is the daughter now?

Digital SAT Math Practice Questions 120

Telegram: SAT_pdf
I Ratios, fates, proportional relationships, and units
I
The ratio of students to teachers in a school is Martha is a math tutor who helps students
20.S :ic. If there are 164 students in the school and prepare for tests. She charges $20 an hour for
8 teachers, what is the value of At tutoring students one-on-one, but she charges
$25 an hour for pairs of students studying with
A) 1 her in a small group. If Martha has 3 hours to
tutor on the night before a test, what would he
li ) 2
the difference in her profits if she were to tutor
Q 4 three pairs of students tor an hour each vs. three
individual students for an hour each (in dollars) ?
D) 8

fhe price of a school trip per student is inversely


'

proportional Lo Lhe number of students going If —-idf - 4L . whal is the value of c


on the trip. If 28 students go on the trip, each
student pays $p. if two students do nut attend the A) 1
trip, what is the new price per student ?
Tf ) 16
1
A)
^
36

14
C) 16

D) -
1
7T />
13

13
Q -p
14

D) 26 p

If —n = 3, whal is the value of —m ? ?


'/!

A) 0
B) 1
C) 2

D > 6

wwwi vibriintpublisbefS .tOftl 121

Telegram: SAT_pdf
I PROBLEM - SOLVING AND DATA ANALYSIS
I
A marine biologist found that the ratio of plant Cookie Dough Vanilla
life to animal life in a particular lake is 2:5. If the
estimated animal life in the lake is 1,680,000,
which of the following best approximates the
population of plant life in the lake?

A ) 672,000
R ) 4,200,000 Strawberry Chocolate
C) 480,000
The pie chart above shows the results of a
D) 1,200,000
school survey. In the survey, people were
asked to choose their favorite ice cream from
among a group of four choices. Which of the
following was chosen as the favorite ioe cream by
approximately 35% of the students surveyed ?

A) Vanilla
Mt. Carmel School conducted a study that
showed fur every three girls in the school, one R ) Chocolate
does noL play any spurL. 40% of Lhe school’s 1,500 C) Strawberry
population are girls. What is the approximate
D) Cookie Dough
number of girls that dn not play ary sport in the
school?

A biologist models a function tor the population


offish in a pond using the equation =
1,320(2)', where t is the number of years. If the
time interval of the function is converted into w
months as /fm ) = 1320(2)*", what is the value of
it ?

Digital SAT Math Practice Questions 122

Telegram: SAT_pdf
I Ratios , rates, proportional relationships, and units ( Answers )

167. Level: Hard | Skill / Knowledge: Ratios, rates, error or lack of understanding of ratios and unit
I
proportional relationships, and units | Testing costs.
Point : Settingup a proportion and solving
problems involving derived units
169. Level: Easy | Skill / Knowledge: Ratios, rates,
proportional relationships, and units | Testing
Key Explanation; Choice I> is correct. Create a
Point: Using proportions to solve word problems
proportion to express miles per gallon in terms
of the total gallons required which yields
Key Explanation : Choice It is correct. Since
Semites 15fl, 00 t) mi /ei
[ here are 36 kids in the camp and iwo-lhirds are
1 gallon x gallons
girls, there are a total of — x 36 = 24 girls at the
Cross multiply to simplify yields 35x - 150,000.
Statistics camp. Of the 24 girls, three- fourths are
Dividing 35 from both sides of the equation 3
yields under 5.5 feet , This means lhaL - * 24 = 18 girls
4
x = 4,285.71 gallons are under 5.5 teet.

Divide the Lotid required gallon of fuel by Lhc 170. Level : E.asy | Skill / Knowledge: Ratios, rates,
CapaciLy of the Lank. to determine how many total
proportional relationships, and units | Testing
tanks are needed. This yields
Point: Solving problems involving proportions
4,285.71
or 85.71 tanks
511
Key Explanation: rlhe currecl answer is 0.40. 'lb
Rounding to the nearest whole Lank yields 86
, find uuL the ratio of hamsLers that haw brown
tanks. eyes, Erst determine the fracLion of dogs with
brown eyes. Since 60% of the 20 dogs have blue
Distractnr Explanations: Choices A , B, and C eyes and dogs only have brown or blue eyes, then
are incorrect and are most likely the result of 4(1% have brown eyes. Therefore, the ratio of
rognding errors or errors in creating and solving
proportions,
hamsters is 40:100 which is 0.40 or —.
171, Level: Hard | Skill / Knowledge: Ratios, rales,
168- Level; Easy | SkillfKnowledge: Ratios, rates,
proportional relationships, and units | Testing
proportional relationships, and units | Testing
Point : Working with inverse proportions
Point; Solve problems involving derived units
Key Explanation; Choice D is correct.
Key Explanation: Choice B is correct. If 2
The number of carpenters varies inversely
pounds of grapes cost $3,96, then one pound of
proportional to the number of days to complete
3.96
grapes costs — - Multiplying by g results in the the work. Therefore, 12 x 3 = 9 x k , k = 4. This
cost of # pounds of grapes, or Choice B. means that it takes 4 days for 9 carpenters to

D istractor Explanations: Choices A , C, and D


complete 36 desks. 4 * — = 7 days.
are incorrect and are the result of an arithmetic

wwW, vibrantpublisbefS-tOftl 123

Telegram: SAT_pdf
I PROBLEM - SOLVING AND DATA ANALYSIS

Distract*) r Explanations: Choices A , B , and C 174. Level: Easy| Ski 11 / Knowledge : Ratios, rates,
I
are incorrect. These options are incorrect and proportional relationships, and units|Testing
maybe due to conceptual or calculation errors. Point: Working with ratios

Key Explanation: Choice A is correct . The ratio


172. Level: Easy|Skill / Knowledge: Ratios, rates,
ol students to teachers is 164:8. Dividing the ratio
proportional relationships, and units|Testing
by 8 yields 20.5:1. Therefore, k = 1.
Point: Converting units

Distractor Explanations: Choices B, C , and


Key Explanation: The perimeter of a square held
D arc incorrect. These options may be due to
is found by 4 (length of one side) . First, convert
conceptual or calculation errors.
all units to inches. The length of one side of the
square held is 2.5 which would be (2.5 sc12) 30 - 175. Level: Easy| Skill / Knowledge : Ratios, rates,
inches. This is the value of one side of the square
field. If there is a margin of 36 inches around the proportional relationships, and units|Testing
field, the length of one side of the fence would he Point: Working with inverse proportions
30 + 36 inches- 65 inches. The perimeter would
Key Explanation: Choice B is correcL. The Lula!
therefore be 4( 56} - 264 inches .
price of Lhc trip can be represented by $p * 28
- $2Kp. Since two students will not attend the
173- Level: Medium | Skill / Knowledge: Ratios, nates, trip, 26 students will need to split the total cost
proportional relationships, and units|Testing equally among themselves. Therefore, the price
Point: Working with ratios that each of the 26 students needs to pay would

Key Explanation: Assuming that ihe daughter’s he


25
which is equivalent to —1 p.
.’
current age is x and {Tinas current age is 3x
Dis trade r Explanations: Choice A is incurred .
It assumes that p is the total price of the trip.
Now Five years
from nuw Choices C and D are incorrect and may be due
Gina’s age 3x 3x + 5 to conceptual or calculation errors.
Daughter’s age JC x+5

176, Level: Easy|Skill / Knowledge: Ratios, rates,


Using the ratio of their ages 5 years from
proportional relationships, and units|Testing
3x + 5 7 _ _
now yields
X +5

- . Performing cross Point: Calculating profit from rate data

multiplication on yields 9x + 15 = lx + 35.


Key Explanation ; If Martha tutors three
Subtracting 7 x and 15 from both sides of the
individual students for one hour each at the rate
equation yields x = 10 . Therefore, Gina's daughter
of $20 an hour, she will make 3 x $20 = $60,
is 10 years old.
Alternatively, if Martha tutors three pairs of
students for one hour each at the rate of S25 ail
hour, she will make 3 x $25 = $75. Therefore, the
difference in the two profits is $75 - $60 = $15.

Digital SAT Math Practice Questions 124

Telegram: SAT_pdf
I Ratios , rates, proportional relationships, and units ( Answers )

177. Level: Medium | Skill / Knowledge: Ratios, rates, Plant life 2


I
proportional relationships, and units | Testing yields
Animal life 5 —
Substituting 1,680,000 to
Point : Rewrite a rational expression Plant life 2
the equation yields - _
Multiplying '

1, 680, 000 5
Key Explanation: Choice it is correct. Using both sides of the equation by 1,680,000 yields
cross multiplication, the equation can be 2 ^ 1, 680,000
rewritten as: 4c x 4 = d x 1
Plant life -
5
= 672, 000 .
This simplifies to:
Distraclor Explanations: Choice B is incorrect
16c = d
and may result due to a calculation or conceptual
Dividing both sides of the equation by c results in
error. Choice C is incorrect , 'lhis is the value
—c = 16 of - of the animal life in the lake. Choice D is
7
Distraclor Explanations: Choices A , C, and D incorrect, lhis is the value of —^7 of the animal fife
are incorrccl and rdlcct errors in simplification. in the lake.

17ft I.eveli Medium | SkillyKnowledge: Ratios, rates, 180. Level : Rasy|Skill / Knowledge: Ratios, rates,
proportional relationships, and units | lasting proportional relationships, and units | Testing
Point!Working with equivalent proportions Point: Using ratios to find a data value

Key Explanation: Choice C is correct Key Explanation: The correct answer is 2t>0,
Since 40% of Lhe school’s 1,500 population are
1 1
If — = 3, it follows —m.3
Since
m 3'
— girls, there arc a total of (40%)(1.500 ) = 600 girls.
multiplying 6 to both sides of Lhe equation yields Since 1 ouL of 3 girls docs not play any sports, Lhe

^ = 200 girls.
bn 6
— - - or bn
—m - % Therefore, the answer is 2. curncct answer is xftQO
m 3

Distract nr Explanations: Choice A is incorrect


181 . Level : Medium | Ski 11 / Knowledge: Rates, ratios,
-m
and may result from solving the value of proportional relationships, and units | Testing
3m
Choice B is incorrect and may result from Point : Modifying an exponential function for a

solving the value of — . Choice D is incorrect units change

18n
and may nesulL from solving the value of
HI
Key Explanation; The correct answer is —1 -
179, Level: Easy | Skill / Knowledge: Ratios, rates,
Since 1 year = 12 months, t = —
12
. Hence, the

proportional relationships, and units | Testing function becomes / (m) = li 320(2)11 . Therefore,
Point; Using ratios to find population values
k= —.
L
12
Key Explanation: Choice A is correct. Use the
given ratio of 2:5 to create the equation that

wwW, vibrdntpublisbefS- tOftl 125

Telegram: SAT_pdf
I PROBLEM- SOLVING AND DATA ANALYSIS

1 £2. Level: Easy | Ski 11 / Knowledge: Ratios, rates,


I
proportional relationships, and units | Testing
Point: Using ratios and percentages to interpret
pic chart data results

Key Explanation: Choice 11 is correct .


To find the one that is approximately
35%, determine the ratio of the number
of students that chose each flavor using
number of students that chose the flavor
total number surveyed
Adding up all the numbers to find the total
number of students who participated in the
survey yields 63 + ft4 + 126 + 147 - 420. Solving
the percentage of students who chose chocolate
147
Yields x 100% — 35%, Therefore, chocolate is
420
the corrccl answer

Distractor Explanations: Choices A , C, and


D are incorrect and are the result of errors in
creating proportions from a pie chart.

Digital SAT Math Practice Questions 126

Telegram: SAT_pdf
I Percentages
I
There was a sale an sofas at a furniture store Number of Questions Correct on Math Exam
for 60% off the original price of any sofa, with
an additional 40% off y our entire purchase if Number Correct Frequency
you buy a matching armchair for a fixed price 31 40 3
of $450. Which expression represents the total 41-50 6
amount you need to pay if yon buy a sofa that is 51 -60 7
originally $s and a matching armchair ?
61-70 9
71 -SO 3
A ) 0,4(0,6s +- 450)
HI 90 4
b ) 0,6(0,41 +- 450) 91-100 2
C ) 0.4s + 0.6( 450)
What percentage of students scored between 0
D } 0.6s + 0.4( 450)
and 70?

A ) 26.5%
b) 73.5%

C) 25%
D ) 75%
A school is renovated so fhaL it is 15% larger than
before. Tfthe school currently has a maximum
capacity of 744 students after the renovation,
what is ihc imiimim capacity (in number
of students) it can hold before the renovation
( round it to the nearest number )?

Opera
Movies
14.2%
21.1 %

Sports
23.6%
Concerts
25.9%
Plays
16.0%

Using the pie chart above, approximately what


percent of people did not like sports?

A ) 76,4
B) 23.6
C) 75

D ) 14.2

wwW, vibrdntpublisbefS- tOftl 127

Telegram: SAT_pdf
I PROBLEM - SOLVING AND DATA ANALYSIS
I
A school has a total population of 620 students. The price of fuel in country X fluctuates. The
The school also sells school supplies to students price decreases by 10% between 2014 and 2016
before the beginning of the year. If 70% of the and then increases by 20% between 2017 and
students purchase school supplies for $25 each, .
2019. If the price of fuel in 2019 is $17.28 what
how much does the school make in total ? was the price of fuel in 2013?

A ) $9,486 A ) $18.66
B) $10,850 B) $19.00
C) $12,543 C) $15.55
D) $ 15,575 D) $16.00

The price rtf' a dress is $24. Tf the price is The price of a chocolate bar increases from $0.60
increased by 120%, what is the new price uf the to $2.40. What is ihe percentage increase in ihe
dress? price of Lhe chocolate bar?

A) 400%
fl ) 300 %

C) 26%
D) 3333%

The price of a dress in a retail store is $200.


During the End of Year Hale at the store, the dress
can be bought at a 30% discount. Amanda buys
the dress during the F.nd of Year sale and uses a
12% off Coupon . How much does Amanda buy
the dress for?
5 gallons of a 60% salt solution is mixed with x
gallons of a 50% salt solution to get a 56,25% salt
solution . What is the value of x?

Digital SAT Math Practice Questions 128

Telegram: SAT_pdf
I Percentages
I
The price of a winter coat is $180. There's a 15% A study shows that 1.5% of the houses in the
sales tax imposed on the retail price. Nia bought city are commercial houses and the rest are
the coat using a 20% off coupon . How much did residential houses. If the number of residential
she pay for the winter coal ? houses in thedlyis 12 ,000. What is the number
of commercial houses in the city? ( round off to
A ) $144.00 the nearest integer)
B } $207.00
A ) 180
C) $ 165.60
li ) 133
D } $ 171.00
C) 60
D) 150

20% of a positive integer is equal to 60% of 200.


What is the value of the positive integer ?

A ) 600
B ) 24
C) 120
D) 96

Opera
Movies
14.2%
21,1%

Sports
23.6%
Concerts
25,0%
Nays
16.0%

In 2017 , approximately 5,842 people were


surveyed about preferred weekend activities.
How many people preferred opera or sports?

wwwt vibrdntpublisbefS- tOftl 129

Telegram: SAT_pdf
I PROBLEM - SOLVING AND DATA ANALYSIS

183. Level : Easy | Skill / Knowledge: Percentages


I
Testing Pointt Using percentages to snlve
a proportion and solver —3425 = 73.52941176%,
problems which rounds to 73.5%.

Key Explanation: Choice U is correct . In order Distracto r Explanations; Choices A, C, and


to correctly represent the situation, match the D arc incorrect and are the result of errors in
correct variable with the correct percentage. The interpreting frequency charts nr calculating
sofa s is 60% off, so the total price is in fact 40% percentages.
of the original price or 0.4s. The armchair has
a fixed price of $450. The sum of 0.4s and $450 136.Level: Easy | Skill,' Knowledge: Percentages
1

is then discounted again by 40%. So, the total Testing Point: Using percentages to solve
amount to pay is 60% of the sum of 0.4s and problems
$450.' lhcieforc, the correct expression is 0.6(0.4J
+ 450). Key Explanation : Choice A is correct Since
23.6% of the 100% of people surveyed liked
Distractor Explanations: Choices A, C, and D sports, Lhe percentage of people lhaL do noL like
are incorrect because they represent errors in
spurts is Lhc difference between the LoLal and
writing discounts as percentages and errors in those Lhal did like sporls. 'Ihis yields 100% -
matching variables and prices with the correct 23.6% = 76.4% .
discount.
Distractor Explanations: Choice B is incorrect
and may result from finding the percentage of
184. Level: Easy | fski I I / Knowledge; Percentages
Testing Point; Using percentages to solve people that like sports. Choice C is incorrect and
problems may result from finding the percentage of people
that do not like concerts. Choice D is incorrect
Key Explanation: If the sixe of the school after and may resulL from finding the percentage of
ils renovation is L 5% larger than before, then it is people that like operas.
now 115% larger. To get the original maximum
capacity of the school, divide the current 187. level; Easy | Ski I If Knowledge; Percentages
744
maximum capacity by 115% This yields
, or Testing Point; Using percentages to solve
646.95, Rounding to the nearest number yields problems
647 students ,
Key Explanation: Choice B is correct . First,
determine what 70% of the student population is
185. level; Easy|Sit ill / Knowl edge: Percentages which yields (0,7)620 = 434. Then , multiply this
Testing Point; Use percentages to solve problems value by the price per pack of school supplies
Which yields 434 X $25 = $10,850.
Key Explanation: Choice B is correct . Adding
up the numbers in the frequency column of .
Distractor Explanations: Choices A C, and
the table, there are 34 total values. There are 25 D are incorrect and are likely the result of
out of the 34 scores between 0 and 70, Set up

Digital SAT Math Practice Questions 130

Telegram: SAT_pdf
I Percentages ( Answers )

arith metic errors in calculating the percentage of change in price is a 10% increase and assuming
I
the student population . that $ 17.2ft is the price in 2013. Choice D is
incorrect. This would be the price assuming the
overall change in price is a 10% increase and
188. Level: Easy | Skill / Knowledge: Percentage
417.28 is the price in 2019.
Testing point : Calculating the percentage
increase
191 . Level : Easy | Skill / Knowledge: Percentages
Key Explanation: The price increases by 120%. Testing Point: Working with percentage increase
lienee, the new price will be 220% of the original
price. Multiplying 2.2 to the original price yields Key Explanation: Choice B is correct.
2.2 x 24 or 52.80. The price of the new dress will The percentage increase is found by
be 452.80. Increase in price
xL 0O%. Tn this case,
Original price
the price of a chocolate bar moves
189. Level: Medium | Skill / Knowledge: Percentages
from $0.60 to $2.40. This would yield
Testing Point : Working with percentage decrease
2.411 - 0-60 1.80
and discount
0.60
* 100% GGu
*100% - 300%.

Key Explanation: During the End of Year Sale, DistractnT Explanations: Choices A, C, and
the price of the dress is 30%off which means that D are incorrect These options may he due to
it is 70% off its original price. 70% of 4200= $140. calculation or conceptual errors.
Amanda has a further 12% ull coupon. This
would make Lhc price of ihe dress to be 38% of 192. Level : Easy I Skill / Knowledge: Percentages
$140 = $123.20, Testing Point: Calculating percentages in
mixtures
190. Level: Hard | Skill / Knowledge: Percentages
Key Explanation: The curreel answer is 3. To
Testing Point: Using percentage increase and
solve this, first , calculate the total amount of the
decrease
resulting mixture by adding the amounts of the
Key Explanation; Choice D is correct. The price two solutions which yield 5 + x. Then, multiply
of fuel in 2013 would be the price of fuel before the mass with the corresponding percent solution
the decrease in 2014, The price in 2016 can be and add them together which yields 5(0,6) +

found like this:


17.28
= $14.40. The price in *(0,5) . Equating this to the resulting mixture
1.2 yields 5(0,6 ) + JC(0,5) = (5 + x}(0,5625), Using
2013 would be = 416 which is the price distributive property yields 3 + 0,5* = 2,8125 +
before the decrease or the price in 2013. 0,5625*, Subtracting 0,5* and 2,8125 from both
sides of the equation yields 0,1875 = 0.0625*.
D istract or Explanations; Choices A is incorrect. Dividing both sides of the equation by 0.0625
Ibis would be the price in 2019 assuming that yields 3 = x or x = 3. Therefore, 3 gallons of the
$17,28 is the price in 2013, Choice R is incorrect . 50% salt solution is used in the mixture.
This would be the price assuming the overall

wwW, ribriintpublisbefS-COftl 131

Telegram: SAT_pdf
I PROBLEM- SOLVING AND DATA ANALYSIS

193. Level: Medium | Skill / Knowledge: Percentages 195. Level: Easy| Ski 11 ,' Knowledge : Percentages
I
Testing Point: Using discounts and sales tax Testing Point: Using percentages to solve
percentages problems

Key Explanation : Choice C is correct. A sales Key Explanation: 2,209 Is correct . kind the
lax of 15% means lhat the new price is 115% of
number of people that prefer opera or sports by
the retail price. This makes the new price to be multiplying the total percent by the total number
1.15 x 1ft!) = $207. This is the price after the sales of people. This yields (14.2% + 23.6%}(5,ft42) =
tax. Nia used a 20% off coupon which would 37-8% ( 5,342 ) = 0.37ft ( 5,ft 42 ) = 2,209.
mean that she pays only 80% of the price which
would yield O.ft x 2(17 = $165.6. 196. Level: Medium|Skill/ Knowledge: Percentages
Testing Point: Using percentages to calculate a
Distractor Explanations: Choice A is incorrect. data value
This is the price withouL imposing a sales Lax and
only applying the discount Coupon. Choice B is Key Explanation : Choice B is correct , The
incorrect This is the price after the sales tax is percentage of residential houses in the city is
imposed and not applying the discount coupon. ( 100 - 1.5) % = 98, 5%. 93.5% was represented
Choice D is incorrect. This option may he due to by L 2,000. Therefore, 1.5% of the houses arc
misconception or calcuiation errors. 12, 000 K 1.5%
98.5%
—tft 2.7, which when rounded off

to the nearest integer is lft 3 .


194, Level: Medium | Skill / Knuwledgc: Percentages
Testing Point: Solving equations involving Instructor Explanations: Choice A is incorrect.
percentages This is the value of 1.5% of 12 ,000. Choices C
and D are incorrect. These options may be due to
Key Explanation: Choice A is correct. Assuming
conceptual or calculation errors.
k is the positive integer, the equation will be 20%
of = 60% of 200.
*
Simplifying the equation yields 0-2fc = 12(],
Dividing both sides of the equation by 0,2 yields
* = 600.
Distradur Explanations: Choice B is incorrect.
This is the value of 20% of 60% of 200. Choice
C is incorrect. This is the value of 60% of 200.
Choice D is incorrect. This is «0% of 60% of 200.

Digital SAT Math Practice Questions 132

Telegram: SAT_pdf
IL 97
One - variable data: distributions and measures of center and spread

L 99
I
Student Population at Greendale School Number of Questions Corrector! Math Exam
Grade Number of Students
Number Correct Frequency
2^ 74 31 40 3
3' j 73 41-50 6
41 n 81 51 60 7
5"' 90 61-70 9
6 th 75 71 80 3
81 90 4
7 <h 82
91-100 2
Rased on the data in the tahie above, what grade
would Lite median number of students be inaL Which measures of central tendency in this set of
Green dalle School be ? data have the greater values?

A ) Median and Mean


R ) Median and Mode
C) Mode and Mean

198 D ) It cannot be determined

TIi e table below repre.se nts the number of species


uf butterfly found in parks over two different
seasons

200
Spring Summer
Park A 32 22 A flock of geese flies south for the winter. If their
ParkB 16 10 journey is 2, 365 miles and they Lake 3.75 days
Park C 17 20 to complete the journey, what was their average
Park D 40 32 speed in miles per hour ? Round your answer to
Park E 23 17 the nearest tenth.

What is Lhe dillcnence in the average number of A ) 26.3 tnph


butterflies in the parks per season?
B) 260.2 mph

A ) 5.4 Q 315, 3 mph

B) 15.1 Dj 630, 7 rtiph

C) 20.2
D } 25.1

ivw. fUjrrrrttpuWhrfirrj.cnrtJ 133

Telegram: SAT_pdf
I201
PROBLEM-SOWING AND DATA ANALYSIS

204
I
- 4, 12, y, 108, -324 A research assistant collected 11 rocks and.
weighed them and rounded off their masses to
A sequence of 5 integers ( which follow a pattern ) the nearest whole number, 'lhe masses of the 11
is shownabove, wherey represents the median rocks in kilograms are shown below.
value. What is the average of the first 4 integers ?
23 , 34, 54, 22, 25, 43 , 36, 29.14, 51 , 21

If a rock that weighs. ] 1 kgs is added to the study,


which of the following statements is true about
the study ?
202
A ) The mean and the range will decrease after
Wlial is the mean for the data set below? the mass is added.

13, 12, 18,10, 11, 9, 15, 18, 11 13) Ihc mean and the range will increase afLer
the mass is added.
A} 9 C) Ihc mean and ihc range will not change after
the mass is added
13) 13
D) The mean will decrease but the range will
C) 12
increase after the mass is added .
D) 18

295
293
What is the difference between the mean and the
'
Ihc average score of 30 students on a lest is 78% . median ol the data scl below?
If Lhe score ot Lhe student who scored lowest
is removed , what would happen to the average
*
score of the remain i ng students?

A ) The average score of the remaining students ••


remains die same. • * * + *
E- ) lhe average score of the remaining students
"

decreases.
m
13 14 15 16 ]g 19 20
l7

C) The average score of the remaining students


increases.

D) I here isn’t enough inlormalion to make a


"

conclusion on the change in the average


score.

Digital SAT Math Practice Questions 134

Telegram: SAT_pdf
I206
One - variable data: distributions and measures of center and spread

208
I
1
Data set A A math teacher increased his students stones by
12 . Which oi the following would increase after
this change?

I. Mean
*
•• IT . Median
ITT . Range
• t t • 9
TV Standard deviation
I I I I
J3 14 IS J 6 ,7 is 19 20 A ) land TIT
R ) TandTf
Data set R
C ) None of the above
D} A 11 of the above

• * 209
I I I I
1.3 14 IS 16 IS 19 20
17
CLASS BUCK CLASS MOW
Which of the following statements Is true about 100 - 80 ] 4
the two data seLs above? KO - 60 8 9
60 - 40 6 2
A ) The standard deviation of data set A is 40 - 2(1 I ]

greater than that of data set B.


The data above represents how Class Buck and
B ) The standard deviation of data set R is
Class Mow performed in their midterm results.
greater than that of da la set A . Which of the following cannot be true about the
C) The standard deviation of data set A is equal two elasses represented in the table above?
to the standard deviation of data set B,
D) There isn't enough information to conclude A ) The median of Class Buck and Class Mow is
73.5%.
about the standard deviation of the two data
sets . B) The mean of Class Mow is greater than the
mean of Class Buck -
C ) The standard deviation of Class Mow is
higher than the standard deviation of Class
Buck.
207
D } The median of Class Buck is 57.6%.
What is the mean of the ungrouped data below';
32 , 43, 54, 56 , 36 ,59 , 56

ivw. itffcrcrrttpuWbrflCfJ.eOrtJ 135

Telegram: SAT_pdf
I PROBLEM - SOLVING AND DATA ANALYSIS

197. Level: Medium |Skill / Knowledge: One variable - 199. Level: Hard | Skill / Knowledge: One variable
I
data: distributions and measures of center and data: distributions and measure of center and
spread|Testing Point; Determining the median spread | Testing Point: Compare and interpret
value of a set of data measures of central tendency

Key Explanation: Choice C is correct. To find Key Explanation : Choice D is correct. Although
the median value of the data set, start by finding from the frequency table ihe median of the data
scl can be found and one can estimate the mean,
the total number of students in the school, which
the mode cannot be determined based on a
yields 74 + 73 + 31 + 90 + 75 + 82 = 475. Since
frequency chart.
the total number of students is odd, then the
expression to get the median is where H is Distractor Explanations; Choices A , B, and
the total population. Substituting the data yields C arc incurred and arc the result of errors in
interpreting frequency charts.
Median - = 233 . 'Ihaeforc, ihe 233"'
student from the lowest tn the highest grade is
200.Level: Easy|Skill/ Knowledge: One- variable
the median . Add the number of students per
data: distributions and measures of center and
level to accumulate 233. The composition of spread | Testing Puint : Finding the average speed
students will he 74 from 2 nd
gradej 73 from 3rd from data
grade, 31 from 41*1 grade, and 10 from Ei 01 grade.
Therefore, the 238"' student is in the 5"' grade. Key Explanatiun: Choice A is correct . Begin by
calculating the total time traveled by the geese in
hours:
198. Level; Medium | Skill / Knowledge; One-variable
data: distributions and measures of center and Converting days to hours yields
24 hours
spread|Testing Point Calculating and compar 3-75 fluyyx
1 day

9< i hours.
ing the means of a set of data
Divide the total distance in miles by
Key Explanation: Chuiee A is correct. the total time in hours to calculate the
To find the difference between the average spued which yields Average speed =
averages, calculate first the average for
2, 365 Mites
each season. This yields Average = 90 hours
= 26,2777 mpK Rounding off lo the

32 + 16 + 17 + 40 + 23 123 ^ ni]g

= 25.6 aud nearest tenth , yields 26.3 mph-


5 5
22 + 19 + 29 + 32 + 17 191 Distractor Explanations: Choices R, C, and D
Average — = 29- 2 .
5 5 are incorrect and may result from a conceptual
Then , get the difference which yields 25.6 - 20.2 or calculation error.
= 5 4.
,

291 - Level; Medium | Skill / Knowledge: One-variable


Distractor Explanations: Choices B C, and D . data: distributions and measures of center and
are incorrect and may result from a conceptual
spread | Testing Point: Compare and interpret
or calculation error.
mean (average ) median range and standard
deviation
Digital SAT Math Practice Questions 136

Telegram: SAT_pdf
I Ore - variable data: distributions and measures of center and spread ( Answers )

Key Explanation: The correct answer is would in turn lead to a change in the average
I
20. Looking at the sequence, each integer is score. Choice B is incorrect . The average score
multiplied by 3 to get the following integer To . would decrease if a lower data set is added or if
find /, multiply 12 by - 3 which yields 12 ic — 3 = the outlier on the higher side is removed. Choice
-36 . Therefore, the median is - 36 . Use this value 1) is incorrect ; there is enough information to
to find the average of the first four integers which answer the question.
-
yields ( 4) + 12 + (-36) + 108 80
4 4
204. Level: Easy | Skill / Knowledge: One variable
202 . Levelt F.asy | Skill / Knowledge: One - variable data: distributions and measures of center and
data : distributions and measures of center and spread | Testing Point: finding the mean and the
spread | Testing Point: Finding the mean of range of data

ungrouped data
Key Explanation: Choice D is correct Adding a
Key Explanation: Choice B is correct . The mean data value that would be an outlier on the lower

of ungrouped data Can be found by gelling the side of the data set would increase the range . The

average. This means that the sum of the data will old range was ( 54 14)- 40 while the nets’ range

be divided by the number of data . The sum of the would be (54 - 11 } - 43.
data is 13 + 12 + 11 + 1 0 + 1 1 + 9 + 1 5 + 1 K + I K The old mean wotdd be 32 and the new mean
- 117. Since there are 9 numbers, the mean would would he 30.25. 'lhc mean has decreased.
11 7
he - 13 ,
i ,' D is, tract or Explanations: Choices A , R , and
DislracLor Explanations: Choice A is incorrect C are incorrect. These statements are false and
'
Ibis is Lhc least data value in the set . Choice C therefore incorrect .
is incorrect , 'Ihis is the median of the data seL ,
Choice D is incorrect. This is the greaLesLdala 2(15. Level : F.asy | Skill / Knowledge: One - variable
value in Lhc set. data: distributions and measures of center and
spread | Testing Point: Interpreting measures of
203 . Level: Easy | Sid. il / Knowled.ge: One- variable central tendency from a dot-plot histogram
data : distributions and measures of center and 2
spread | Testing Point: Changing average score Key Explanation: The correct answer — ,

measure of central tendency The mean of the data set is given by


sum of the values
. The sum of the 12 values is
Key Explanation; Choice C is correct . Removing w, of the values
the Lowest score or the outlier on the lower end ( 13 x 1 ) + ( 1 4 x 2) + ( 15 x 5) +( 17 x 2) + ( 18 x
of the test vcould Increase the average swre - 1) + (20 x l ) = 188. The mean would therefore
be = 15 - or 15.667 . The median of 12 data
DistraclorExplanations: Choice A is incorrect . L2 3
The average score would not remain the same is given by the average of the 6'1' and the 7 th data ,
7

because removing a data set would lead to a which would be 15 . The difference between the
change in the sum of the scores in the test which

ivw. idfcmrttpuWfi/irfj.enrti 137

Telegram: SAT_pdf
I PROBLEM-SOWING AND DATA ANALYSIS
I
near and the median would be I 15 - 15
1 3 — ; =
Key Explanation: Choice B is. correct . It' each
data point is increased by 12, the mean and the
median would both increase.
3
Detractor Explanations : Choice A is incorrect,
266. Level: Medium | Skiii / Ku owl edge; One variable 'the range of the data set would remain the
data: distributions and measures of center and same because all the data points arc increased
spread |Testing Point; Finding the standard by the same amount , this would not change tile
deviation of grouped data difference between the maximum and minimum
data. Choice C is incorreci . " the mean and range
Key Explanation : Choice A is correct. will increase and thus not all options are false.
A standard deviation is a measure of Choice D is incorrect. The range and standard
how dispersed the daia is with respect deviation remain the same and therefore rot all
to the mean . Tire mean in Data Set A is
options can be correct.
13 + 14 + 14 - 15 + 15 +15 + 15 + 15 + 17 + 17 + 18 + 2 U
12
- 15.67 . ’lhe mean in Data Set B is 209.Level: Hard | Skill/ Know Ledge: One- variable
14 + 14 + 15 + 15 + 15 + 15 + 15 +17 + 17 + 17 +18 data; disiribulions and measures of cenLer and
11 spread|Testing Point: Calcinating measures of
center and spread
- 15.63. Looking al the charts, the data in SeL A
is more dispersed from its mean compared to
Key Explanation : Choice D is correct This is
the daia in SeL 13. Therefore, SeL A has a greater
the only option that is false. Since there are 16
standard deviation than Set B.
scores in Class Buck , the median is the average
of the Klh and Vth scores. The Hth and 9th scores
Distractor Explanations: Choices A, C, and D
of Class Buck, lie between 60 and 80. Therefore;
are incorrect The statements are false.
the median cannot be lower than 60 and greater
than 80.
2117. l evel: Easy | Skill / Knowledge: One -variable
data: distributions and measures of center and Dis tractor Explanations : Choice A is incorrect.
spread|Testing Point: Finding lhe mean of a A median of 73,5 is possible since it is between

group of data and 80. Choice B is incorrect - Tire mean of


class Mow is between 60 and 80 and tire mean
Key Explanation : Tire correct answer is 48- The of Class Buck is between 51 ,25 and 71 ,25 .
Hence, it is possible that the mean of class Mow
mean of the above data is given by tire sum of the
is greater than the mean of class Ruck. Choice
data set divided by the number ol data. The sum
C is incorrect. It is possible that fire standard
of the data is 32 + 43 + 54 + 56 + 36 + 59 + 56 =
deviation of Class Mow is greater than the
336
336 , Therefore, the mean would be —7 = 48- standard deviation of Class Buck.

208 l evel ; Hard |Skill / Knowledge: One-variable


data: distributions and measures of center and
spread |Testing Point; Calculating measures of
center and spread

Digital SAT Math Practice Questions 138

Telegram: SAT_pdf
I Two-variable data: models and scatlerplols
I
Cell Growth
1J “ J

L X
I
£ oa
' t) 6
j
^ X
X
—— School A
School fl

£ 02 ,

6 c 1 2 3 4 5 6 7 B 9
Year since 2010

The graph above represents the approximate cost


of textbooks in college fur two different schools.

licLwc&l which years was the Cost of textbooks


equal at both schools?

A ) 2013 - 2014
Time ( hours )
11) 2014 - 2015
Ajccunling to the data in ihe graph uf /f Jf ) above,
how many hours have passed when jc) = 1? C) 2015 - 2016
^ D ) 2016 - 2017

wwwi whrdnfpijWishtfrS.fOrt} 139

Telegram: SAT_pdf
I PROBLEM - SOLVING AND DATA ANALYSIS
I
200 jjj 3 0
25
190
X g
> I
J:i


15
E ISO
E m
£
Li

3? 170
&>
Q
_
5
- I“ 0 1
160 i 0 2 4 6 8 JO 12 J4 16
38
X
Time ( in minutes)

i .' ii
The scatterplot above represents the distance
llial Janel has walked going Lo her friend’s house
140 150 160 170 180 190 200 from her home. According lo the graph, at
approximately whar time Interval does Janet walk
Arm Span (cm )
the fastest ?
Lhe scalier plot above shows llic da la collected
on the arm span and height of' students in a cl ass. A ) li - 2 minutes
A line of besl Qt with the equation 13) 4 - 6 minutes
y - 0.81M: + 25.68 where x is arm span in cm and
(2) 8 - 1 0 minutes
y is height in cmt is calculated tor the data. Based
on Lhe line of best fit , if the arm span of a student D) 12 - 14 minuLcs
is 1(5(5 centimeters, what is the predie led height
in centimeters?

A) ifiy.2i

R) 166.78

C) 163-64
D) 166-91

Digital SAT Math Practice Questians 140

Telegram: SAT_pdf
I Two-variable data: models and scatlerplols
I
90
S5
——
J I L The temperatures y in a town on jr flays were
recorded ° C and plotted on the graph below.
Using the line of best fit as the predicted value,
30 by how many“C is the predicted value on day 4
S 75 greater than the actual value recorded ?
y II
S 70
-
65 /
==
II 5-

60 ::
55 / ==
II

EE
85 05 10t ) 105
Temperature IT ) 0 10

The scatterplot above shows the relationship


between the outdoor temperature and the wuLer
Consumption for a Camper un a 10-day tamping
trip. Ore day is not included in the scatter plot
where the temperature was 97" F. based on Lhe
line of best hi, which uf Lhe following is elusesL to
the amount of water consumed by the camper on
that day?

A ) tiK ounces

b ) 73 ounces

C) 75 ounces
D) 82 ounces

itmL vibrdntpublisbefS- tOftl 141

Telegram: SAT_pdf
I PROBLEM - SOLVING AND DATA ANALYSIS
I
The temperature in town A is recorded for ten Which of the following best describes the
days and represented in the scatter plot below. equation ot' the line of best fit ?

/
E X
/
/

X
x Flint in i m+- y
X

X
X 11 In
X
-5-
*
\ X-S-

A) y = -3JC + 2.8
For how many days actual temperatures does the
line of best fit predict a higher temperature? B) y - \.lJt - lfS

-
C) y l A x - l.A
D) y - -1.2* - 2.8

Digital SAT Math Practice Questions 142

Telegram: SAT_pdf
I218
Two- variable data: models and scatterplots

220
I
Which of the ini lowing is the equation that best " Uu
represents the graph below? tsoo
J 500
-1 U 0

IiLJLJ
7
Q 200 7
*
100
10'
* u
Hour I EJouri Hour } -
llour i Iiuur 5
*
*
Libii LinJ Vivian hike a trail one afternoon. The
graph above shows the distance they traveled
o to overtime. At which interval do Lisa -and Vivian
hike at the slowest speed?

A ) Between hours 1-2

A)
B)
r - 3(1.5)'

y ^ O. SSU.Z ) 11
B) Between hours 2-3
-
C) Between hours 3 1 —
D } Between hours 4-5
C) y ^ 0.85{0.2) 11

D) - 1.2 (0.85)^
^

219

The graph below represents Lhe model /fx ) . What

^
is the value of 3,5) that is predicted by the line
of best fit ?

N
10'

www. vibranlpubtishers.cain 143

Telegram: SAT_pdf
I PROBLEM - SOLVING AND DATA ANALYSIS
I
Number 10 20 30 40 30 60 70 80 90 100 160,000
of Points 140,000
120,000
Jerome 2 1 0 ] 3 0 0 1 1 0 100,000
Matilda 4 0 0 3 1 0 0 1 0 0 80,000
60,000
40,000
Jerome and Matilda art playing darts, 'there 20,000
are 10 different point sections on the board and 0
Jerome and Matilda have already played 9 of 0 0.5 1 1.5 2 2.5 3
Lhcir 10 darLs in this game. 'Ihe information is
represented in the table above. If Jerome hits Ihe graph above represents Lhc average salary
the 70-point spot on his final dart, what must as iL corresponds to Lhe distance frum idle CiLy
Matilda store to beat Jerome? center. According to the graph, which value is
closest to the average salary at 1.75 miles from
A ) 100 the city Center?

U ) 90
A) S60,000
C) 80
B) S70,000
D) NoL possible given her currcnL score
C ) S90,000
D) $120,00(1

Digital SAT Math Practice Questions 144

Telegram: SAT_pdf
I Two- variable data: models and scatterplots
I
Year Drivers Licenses
2000 141 ,000
2002 137,000
2005 133,000
2006 127,000
2011 102,000
2014 91 >800

Using the table above, a regression analysis


between the number of drivers Licenses issued
in Humble, Texas and the year resulted in the
following equation: y - - 3,725s: + 7, 596,500.

According to tbe regression equation :

A ) There is a decrease in die number of drivers


licenses issued since The year 2000 .

There is an increase in the number of drivers


licenses issued since ihe year 2000 .
C) There is no evidence of an increase or
decrease in the number of drivers licenses
issued since tbe year 2000.

UJ There is no decrease or increase in the


number of drivers licenses issued since Lhe
year 2000,

WWW. vibranlpubtishers.cain 145

Telegram: SAT_pdf
I PROBLEM - SOLVING AND DATA ANALYSIS

-
210. LeneL: Easy| Ski I / Knowledge: Two variable -
213. Level: Medium |SkiJl / Knowledge: Two variable
I
data: models and scatterplotx | Testing Point: data: models and scatterplots|Testing Point:
Solving problems involving two variable graphs Reading a scatterplot graph

Key Explanation: Ihe graph represents an Key Explanation: Choice C is correct. Speed can
exponential function Looking at the graph, y = 1 yards
, be calculated as j which on this graph
minute
at the start or when x = 0- Therefore, the number
of cells is initially 1 and zero hours have passed . is —x or the slope. The fastest speed is therefore
where Lhc slope is the largest or steepest. And
21L Level: Easy|Skill/ Knowledge: Two-variable the steepest line among the choices is between
data: models and scaLLcrplols | Testing Point: minute ft and ] 0. The answer is, therefore,
Interpreting data on two-variable graphs Choice C.

Key Explanation ; Choice D is correct. The point Distractor Explanations: Choices A , B, and D
where the two graphs intersect is where the cost are incorrect because the speed at those time

of lhc textbooks is the same, lhc intersection intervals is less than the speed at ft - 10 minutes.

point lies between the 6th and 7th year after


2010. Therefore, the answer is between 2016 and 214.Level Easy|Skill/ Know ledge: Two-variable
2017. data: models and seallerplots|Testing Point:
Solving problems involving line of best fit
Distractor Explanations: Choices A , E , and C
are incorrect and may result from an error in Key Explanation : Choice C is correct.
interpreting the data on the graph. Identify 97° F on the x-axis, Then identify Lhc
intersection on the line of best Cl which is at y =

212. Level: Easy|Skill / Knowledge: Two variable 75-

data models and scatterplots | Testing Point:


Distractor Explanations: Choices A , B, and
Solve problems using line of best fit on a
D are incorrect and are the result of errors in
scatterplot
interpreting lines of best lit.

Key Explanation: Choice B is correct . To find


the predicted height, substitute 166 for x in the 215.1evel; Medium |.Skill / Knowledge; Two-variable
line of best fit equation as follows : data: models and scatterplots|Testing Point :
y = 0,85x + 25,68 Finding the equation of the line of best fit
y = 0.85(166) + 25.68 = 166.78
Key Explanation ; The actual temperatures
Detractor Explanations; Choices A , C, and D recorded on day 4 were 0 and 3.1.
are incorrect and reflect errors in simplification
Looking at the graph, the JT and y intercepts
or misinterpretation of the line of best fit. of the line of best fit are (10, 0) and (0, 2) ,
respectively. Solving the slope of the line yields

——
-2 - 0 1
H e n c e, the equation of the line

Digital SAT Math Practice Questions 146

Telegram: SAT_pdf
I Two- variable data: models and scalterplots ( Answers )

-
x i ntercept i n the graph, y = l . ljr - 2.8 is the
I
of best fit is y = 1 x + 2 - The predicted value of
— equation of the line of best fit.
1 4
4thday would be y = - ( 4} + 2 or y = - - + ± ,

Distractor Explanations: Choices A and D are


Simplifying the equation yields y =

^ ory = 1.2.
The value that is Less than 1.2 is 0. Therefore* the
difference is 1.2*.
incorrect because both equations have a negative
value of the slope. Choice C is incorrect because
the ^-intercept of the line is not matched with
the graph .

216. Level: Easy | Ski] I / Knowledge: Two-variable


218. Level: Hard |Skill / Knowkdgc: Two-variable
data : models and scalterplots | Testing Point :
data: models and scalterplots | Testing Point:
Interpreting scalterplots and line of best fit
Determining the exponential equation from

Key Explanation: The correct answer is fi. To graph


solve this, count the number of data helowthe
Key Explanation: Choice B is correct. The
line of best fit There are 6 points below the line.
graph is an increasing exponential graph. An
exponential growth function has an equation ofy
217, Level: Easy | ShiU / KjiowJedge: Two- variable - d( l + r) where a is they-inlerccpt of the graph
1

data : models and scatLerplols | Testing Point : and (1 + r) is Lhe growth factor. The y-intercept
Determining the equation of line of best fit from of the graph is less than 1. ffence, the answer
graph can only be Choice B or Choice C. Since the
growth factor of Choice B is greater than 1 , y =
Key Explanation: Choice JJ is correct. Since 0.85( 1.2)^ is the equation of the graph .
the graph rises from left to right, then the slope
of the line of best fit is positive. Looking at the Distractur Explanations: Choices A and
choices, only Choices B and C have a positive D are incorrect because the y-intercepls in
slope indicated by the positive value of the co- the equations are greater than 1. Choice C is
efficient of x. Hence, one of them is the correct incorrect because a growth factor of less than 1
answer. implies that the equation is an exponential decay
To determine which of the two choices is function .
correct, solve the x-intereept of each equation by
substituting 0 to y. 219. Level: Easy | Skill / Knowledge: Two-variable

u
yields —
x = = 2,55,
1.1
_
Hence, Choice B becomes 0 = 1, 1* - 2.8, Adding
2.8 and dividing both sides of the equation by l . l
2.8 „
data: models and scatterplots | Testing Point:
Finding the predicted value from the line of best
fit

Getting the x -i ntercept of Choice C yields 0 = Key Explanation; To find the equation of the
2.1 x - 2.4. Adding 2.4 and dividing both sides of line of the best fit , first find the slope . Using the
2, 4
the equ ation by 2.1 yields x = — = 1.14. points (0, 12 ) and (8, 0) to find the slope yields
2. L
Since the *- intercept of Choice B matches the
12-0
0-8
12 -3 . . , . ..
— = — . The equation of the line

.
wwW vibrdntpublisbefS- tOftl 147

Telegram: SAT_pdf
I PROBLEM- SOLVING AND DATA ANALYSIS

-
of best fit can be written in the slope intercept for Matilda to beat Jerome with only one more
I
form y = tns + c, where c is the -intercept and throw.
^
m is the slope. Hence, the equation of the line of
best fit isy - -1.5« + 12. Substituting 3.5 to * and Dislraetor Explanations: Choices A, B, and C
_
(3.5) toy in the equation yields /(3.5} = -1.5(3.5) are incorrect because every value added to 290 is
^
+ 12 = 6.75. still less than 470 points.

220. Level: Easy|Skill / Knowledge: Two - variahle 222.Lcvel: Easy|Skill/ Knowledge: L’wo- variable
data: models and scatterplots|Testing Point: data: models and seallerplots|Testing Point:
Interpreting data values on a graph and the Interpreting a value tom a graph
distance between them
Key Explanation: Choice Cis correct. To
Key Explanation : Choice B is correct. To solve determine the average salary (y), when ( x ) is
1.75, use the line of best fit. 'Ihc line of best fit
this problem, analyze the data on the graph and
includes the ordered pair ( 1-75,90,000), so the
compare the distances that they traveled between
average income is approximately $90,000.
each time interval. Between hours 1 - 2, they
traveled 400 - 2t »0 - 2 ttfl feet. Between hours
Distraclor Explanations : Choices A, B , and D
-
2-3, they traveled 450 - 400 50 feet. Between
are incorrect and are the result of incorrectly
hours 3-4, they traveled 550 - 450 - 100 feet
interpreting the data on the graph.
Between hours 4-5, they traveled 650 - 550 ^
100 fed. Since they covered the leasL distance
between hours 2-3, they were traveling at the 223- Jjevel: Easy|Skill / Knowledge: Two-variable
slowest speed at this time interval. data: models and scatterplots I Testing Point:
Calculate trends in table and work with algebraic
Di&tractor Explanations: Choices A, C, and D equation
are all incorrect and most likely result in errors
in understanding and interpreting the data on a Key Explanation: Choice A is correct.
graph . Luolung at the data on the graph as the year
increases, there is a trend showing that the
number of drivers licenses issued is decreasing.
221 Level: Easy|Skill / Knowtedgei Two-variable Additionally, Lhe regression equation is written in
data: models and scatterplots|Testing Point; the form of y = mx + b where m is the slope and
b is the -intercept. In the regression equation
Evaluating data models ^
y = -3,725x + 7,596,500 and thus the slope is
Key Explanation: Choice D is correct. After -3,725 which is negative and thus indicates that
9 darts, Jerome has a total of 400 points and
as the years increase, the number of drivers
licenses decrease.
Matilda has a total of 290 points. If Jerome
throws a 70-point throw, his score increases to
Dislraetor Explanations: Choices B, C, and D
470 points. Because the difference between their
are incorrect and are the result of interpreting
two scores Ls over 100 points, which is the largest trends in table data incorrectly or the slope of a
single point value, it Ls therefore impossible linear function incorrectly.

Digital SAT Math Practice Questions 148

Telegram: SAT_pdf
I Probability and conditional probabilily
I
Mary has a jar of different flavored candies. She 0-5 &-10 11-15 Total
has 47 total pieces, including 16 chocolates and hours hours hours hours
17 pepperminl-ilavored candies. If she also has a Grade 10 21 16 25 62
third type of candy, which are caramel flavored,
Grade 11 15 17 35 67
what is the probability that Mary will NOT
Total 36 33 60
choose a caramel candy from the jar ?
Students
16
A)
47 According to the data in the table above, if one
was to choose a student at random from grade
17
B} 1 Oj, what is the probability that the student
47
studied at least 6 hours a night ?
14
C)
47
33
U)
^
17

Declared Non- Total


Fne-Med Majors
Majors
Took a Did not take Total Passed 204 150 354
Bapyear a gap year
Biology 101
Preshman 37 75 112
Did Not Pass 32 55 87
Sophomore 21 62 33 Biology 101
Junior 40 60 100 Total 236 205 441
Senior 15 82 97
Total 113 279 392 The table above shows the pass / fail results of a
class of Biology 101 students, which includes
The table above includes the results of a survey of Pre- Med majors and some Non - Major students.
students at a school who took a gap year
If a single student is chosen from the full class,
If a Student is randomly chosen from the what is the probability that they are a Non-Major
freshman and sophomore, whal is ihe probability student who passed the exam ?
that the student took a gap year ( round it to the
,

150
nearest tenth decimal ) ? A)
441
204
B)
441
32
Q
441
87
D) T77
441

wwwi vibrdntpublisbefS- tOftl 149

Telegram: SAT_pdf
I PROBLEM - SOLVING AND DATA ANALYSIS
I
In a carnival game, there are a total of n hean A restaurant has 102 items on the menu. The
bags in a sack. The bean bags are either red or menu contains either dairy-free or gluten - free
yellow. If the probability of choosing a ret! bean items and these items either contain nuts or not.
bag al random from llic sack is- 60%, which of the
following represents the number of red hean bags Gluten Daily Total
in the sack bags?
, free free

With nuts n so
A) Without nuts
10
4ft 102
R ) 6«
HJn WhaL is the probability that a customer chooses a
°T
D) 6t) n
dairy free meal without nuts?
li
A)
26
11
B)
51
26
C)
51

The chance that it would tain in the first Two 16


D)
weeks of December is 50%. Tf it rains on the 51
first 3 days oi December, wlial would be the
-
probability that it would also rain on the lLh day
of December ?

3
Al 14
7 Sever halls numbered 1 to 7 are inside a host. Tf a
ft ) ball is randomly picked from the box, what is the
14
probability that it is a iaeior of 6?
C) -
14
12
D)
14

Digital SAT Math Practice Questions 150

Telegram: SAT_pdf
I Probability and conditional probabilily
I
A chocolate box has 3 flavors: Mango, White, and A bag hasp blue- only marbles , g red- only
Dark chocolate. There are 12 pieces of each flavor marbles, and / marbles that are both red and
inside the bcoc Marie lakes a mango-flavored blue. What is the probability that Amber picks a
chocolate. What is the probability that she would marble that has the color red?
pick a mango - flavored chocolate a second time?
g+ f
A)
1 g+ f +P
A >i
li )
P+ f
11
h} g+ f + P
36
i C)
P
C) --
£+ f +P
11

i: g
m - D)
g+ f + p

A fair coin is flipped 9 times. It lands on tails The probability that Goldie carries an umbrella
5 limes anil on heads 4 times. What is the during a certain week is 0- 45 . During that week,
probability of flipping a head on the 1011' Lime? Lhe probability that it WOLIIJ not rain was 0,2 ,
What is the probability lhaL Goldie carries an
umbrella on a rainy day?

A ) 0.45
R ) 0,8
C) 0.36

D) 0, 55

wwwt vibrtntpu MishtfrS.COrtt 151

Telegram: SAT_pdf
I PROBLEM - SOLVING AND DATA ANALYSIS

224. Level: Hard| Skill / Knowledge: Probability and studied anywhere between 6 and 15 hours, so the
I
conditional probability | Testing Point: Using the 25 +16 41
probabilitycan be expressed
as
concept of probability to solve problems 62 62

Key Explanation: Choice D is correct. Ihc 227. Level: Easy | Skill / Knowledge: Probability and
probability that Mary will not choose a caramcl- conditional probability | Testing Point: Using the
flavnred candy can also he expressed as the concept of probability to solve problems
probability that Mary will choose a chocolate

16 17 33
+
__
OTa peppermint , which can he expressed as


Key Explanation : Choice A is correct. The
probability is a fraction of the total class, so the
denominator of the fraction must he 441. Next,
47 47 47
identify which value on the table represents
Distractor Explanations: Choices A. H and Non-Majors who passed the exam, which is 150 ,
150
C arc incorrect because they represent the The probability is therefore
44 ]
probability that Mary will ehoose a chocolate, a
peppermint, or a caramel candy, respectively. Distractor Explanations: Choices R is incorrect
and may result from finding the probability to
pick a Pre- Major student who passed. Choice
225. Level: Medium | SkilJ / Knuwlcdge: Probability
C is incorrect and may result from finding the
and conditional probability | Testing Point:
probability to pick a Pre-Major student who did
Using Lbe concept of probability Lo solve
not pass. Choice D is incorrect and may result
problems
from finding the probability Lo pick a Pre-Major
or non-Major student who did not pass.
Key Explanation : The total number of students
in freshman and sophomore that could be
chosen is 112 + 85 = 195. 'Ihe number of students 228- I.eve]] Medium |,Skill / Knowledge: Probability
in freshman and sophomore who Look a gap and conditional probability | Testing Paint:
year is 37 + 21 = 58, Therefore, the probability of Using the concept of probability to solve
problems
choosing a student who took a gap year is ^-
195
.0.297 = 0.3 ( rounded Lo the nearest tenth). Key Explanation: Choice A is correct. Since

226. Level: Medium |Skill/ Knowledge: Probability


and conditional probability|Testing Point : Use
60% = 100 10
, then
10 —
of the total bean bags

will be red. Therefore, multiply the total number

definition of probability to sulve problem of bean bags n by —Hi6 which results in 10

Key Explanation ; —tU41 or approximately 0 66 J


.
, Distractor Explanations: Choices B, C, and D
are incorrect and may result from a conceptual
is correct.
If you are choosing a student at
or calculation error.
random from Grade 10, the total number of
students to choose from is 62. If a student
has studied at least 6 hours, then he or she has

Digital SAT Math Practice Questions 152

Telegram: SAT_pdf
I Probability ard conditional probability (Answers )

229. Level: Easy | Skill / Knowledge: Probability and 231. Level: Medium | Skill / Knowledge: Probability
I
conditional probability|Testing Point: Finding and conditional probability|Testing Point:
probability of an event Calculating probabilities

Key Explanation: ChoiceB is correct. The Key Explanation: ihc correct answer is —4 .

probability that it would rain in the first two Ihe factors of 6 are 1 , 2 , 3. and 6 , "therefore, the
weeks of December is 50% which means the probability that the numbered ball is a factor of
probability that it would rain on the fourth day is 6 Ls
also 50% , 'this is given by the fraction —.
14

Detractor Explanations: Choices A , C, and D 232. Level: Hard | Skill / Knowlodge: Probability
are incorrect. These options would be the result and conditionalprobability'|Testing Point:
of a conceptual or calculation error. Calculating conditional probabilities

Key Explanation; Choice D is corrctL, After


230, Level: Easy | Skill / Knowlcdgc: Probability picking Lhe first thoculaLe which was mango-
and conditional probability|Testing Point: davoieti there were If mango flavors left.
Calculating probabilities Consequently, [here were 36 - 1 - 35 chucolaLes
left in the box. Ihc probability that Marie would
Key Explanation: Choice B is correct.
pick the mango flavor for the second Lime would
Probability is found by favorable outcomes
sample space , First complete the table:
^ ,
be —II15 ,

D is tractor Explanations: Choke A is incorrect.


Glulen- Dairy- Total
ffee free Ihis is the probability of picking any flavor
With nuts IS 32 511
for the first time. Choice B is incorrect and
Without nuts 30 22 52 may result from not considering the number
4S 54 102 of chocolates left in the box after the ft rst pick.
Choice C is incorrect and may result from a
The number of dairy-free products without nuts conceptual or calculation error,
is 22. Therefore, the probability that a customer
would choose a dairy-free meal without nuts 233, Level: Easy | Skill / Knowledge: Probability
mu 22 11
would be — = —51 .
102
and conditional probability|Tfesting Point;
Calculating probability of independent events
Discractor Explanations; Choice A is incorrect.
This is the probability of choosing a dairy-free Key Explanation: The correct answer is 0.5.
meal from products without nuts. Choice C is The probability of landing heads in a fair coin is
incorrect. This is the probability of randomly always 0.5 regardless of the number of times the
choosing an item without nuts. Choice D is coin was flipped.
incorrect. This is the probability of randomly
choosing a dairy-free item with nuts.

.
wwW vibrdntpublisbefS- tOftl 153

Telegram: SAT_pdf
I PROBLEM- SOLVING AND DATA ANALYSIS

234. Level: Medium | SkiJJ / Knowledge: Probability 235. Level: Easy | Skil 1 / Knowledge: Probability
I
and conditional probability | Testing Point: and conditional probability | Testing Point:
Calculating probabilities Calculating complement probabilities

Key Explanation: Choice A is correct. lhe total Key Explanation: Choice C is correct , lhe
number of marbles that have red is £ + / lhe probability that it would rain can be calculated as
total number of marbles in the bag is g + f + p- (1 -
0.2) = fl.#. The probability that Goldie carries
Therefore, the probability of pi eking a marble an umbrella ou a nainy day is 0.45 x 0.fi = 0.36.
with red color is g + f
g+ f +P Distractor Explanations: Choice A is incorrect,

Distractor Explanations: Choice B is incorrect ,


lhis is the probability that Goldie carries an
lhis is the probability of randomly picking
' umbrella , Choice B is incorrect , lhis is the
a marble that has the color blue. Choice C is probability that it would rain that week. Choice
irtCOrretL. lhis is the probability of randomly D Is incorrect as Lhis is Lhe probability lhaL
picking a marble lhaL only has Lhe colur blue. Goldie does not Carry an umbrella.
Choice D is incorrect, lhis is Lhe probability
of randomly picking a marble that only has the
color red.

Digital SAT Math Practice Questions 154

Telegram: SAT_pdf
I Inference from sample statistics and margin ol error
I
An TT company selected 1 ,000 employees A botanist found that the average height of a
randomly lip = 0.9, or 90% were in favor of
, Moringa tree is 34.6 feet. The estimated margin
4-day- weeks. The standard error ofp ( the of error in the study was 1.2 feel . Which of
sample proportion ) rounded to the nearest the following statements best represents the
thousandth is given by? information given ?

A ) 0,009 A ) The trees have an average height of 35.611


feet.
B ) 0.081
13) All Moringa trees have a height between 33.4
C) 0.064
foci and 35.8 feet
13) 0.049
C) The average height of a Moringa tree lies
between 33.4 feet and 35.8 feet.
D) All Moringa trees in Lhe Study have a height
between 33.1 feel and 35.8 feet.

Tn a random sample of 300 widgets, 4 are


defective. Using this data , how many of Lhe
12,000 widgets wuidd one estimate lobe
defective?
The average weight of high school students in a
class is 147.3 its, The maximum average weight
of the high school students was 160 Nw and the
minimum average weight is 135,6 ife. What is
the margin ol error associated with the average
weight of high school students?

A random sample of 10>000 employees was


asked whether they had lhe habil of smoking.
Fourteen percent of the 10,000 employees said
they had . Which cme of the following statements
abuut the number 0.14 is correct ?

A ) It is a sample proportion.
B) It is a population proportion.
C) It is a margin ol error.

D) It is a randomly chosen number.

vibrdntpublisbefS- tOftl 155

Telegram: SAT_pdf
I PROBLEM - SOLVING AND DATA ANALYSIS
I
A study was conducted on 20 students at an SAT A study was conducted on how many
prep center to find the average SAT score. Which teachers in an elementary school in city A
of the following statements would reduce the had a postgraduate diploma , lhc teachers that
margin of error? participated in lhc study were chosen randomly.
The study showed that 2ft out of the 35 teachers
A ) Increasing the sample of st udents from the have a postgraduate diploma. Which of the
SAT test prep center following can best be concluded from the given
information ?
R ) Increasing the sample of students from
students at a nearby SAT prep center
A ) 80% of teachers in city A have a postgraduate
C) Decreasing the sample siiC of students from diploma.
the SAT test prep center
J3) 80% of teachers in cities ha vc a pos Lgrad uaic
D) Conducting the study with the top ten diploma.
students at Lhc SAT LesL prep center
C) 80% of teachers in elementary schoul have a
postgraduate diploma.
D) Nothing can be concluded from the study.

A representative sample of households in City X


are surveyed. The survey shows that 17.8% of the
households in Lhe sample have cars. If City X has
a total of 21 ,(H)Q households, approximately how 1 (10 residents in town X were surveyed - 50% of
many of them have cars? the respondents voted yes to building a park in
the middle of Lhe town. The town’s population
A} 2,100 is <50,000. Which of Lhe following is Lhe best
conclusion drawn from the given information ?
R ) 1 ,869
C) 3.738 A ) An estimated 26,400 of the population of
D) 7.496 town X would vote yes to building a park in
the middle of the town.
B) An estimated 33,600 of the population of
town X would vote yes to building a park in
the middle of the town.
C) An estimated 33,600 of the population of
town X would vote no to building a park in
the middle of the town,
.
D) Of the surveyed 56 voted no to building a
park in the middle of the town .

Digital SAT Math Practice Questions 156

Telegram: SAT_pdf
I Inference from sample statistics and margin of error ( Answers )

236. Level: Easy | Skill / Knowledge: Inference from Key Explanation: Choice A is correct since it is
I
sample statistics and margin oi error|Testing the proportion of the sample of 10,000 people.
Point : Calculating the sample standard of error
Dislractor Explanations: Choices B, C, and

Key Explanation: Choice C is correct . Here D ire incorrect and are the result of errors in
p is the proportion, The standard error of p is understanding the concepts of sample and
population proportions.
given by MW ) , where n is the number of
H

employees interviewed. 239, Level: Hard | Skill / Knowledge: Inference from


Standard Error =
sample statistics and margin of error | Testing
0.9(1- 0.9) If (0.9 * 0. L)
- 0.0094 * 0.009. Point ; Margin of error
1000 Vl . 100(1
Key Explanation: Choice C is correct . This
Distractor Explanations; Choices B, C, and
statement is true since the margin of error is
D are incorrect and mostly result from an error
1.2 feet. Therefore the average height would lie
calculating the standard error or applying the between 34.6 ±1.2. This would mean that the
wrong formula for standard error
-
average height lies bcLween 33.1 feel lu 35 ,3 feel.

237. Level: Easy | Ski 11 / Knowledge: Inference from DLstracLur Explanations: Choices A, B, and D
sample statistics and margin of error | Testing are incorrect. These statements can he true but
Point: Extrapolate from sample to population not certain.

Key Explanation: To solve the problem, seL up a


210, Level: Medium | Skill / Knowledge: Inferenee
proportion as follows:
from sample statistics and margin of CTior
( defective sample widgets ) ( defeetive widgets ) Testing Point: Calculating the margin of error
{ total sample widgets) ( tom / widgets )

4 x Key Explanation: The correct answer is 12.2, Use


300 12, 000 Maximum limit - Minimum limit
the formula
2
Cross multiplying yields to find the margin of error. Substituting
300x = 4 * 12,000 the given data yields a Margin of error
160 -135-6 24, 4
Dividing both sides of the equation by 300 results -12.2.
2 J
in
4SS, 000 241. Level: Easy | Skill / Know!edge: Inference from
JC = = 160
300 sample statistics and margin of error | Testing
Point ; Making statistical inferences
238, Level: Easy | Skill / Know!edge: Inference from
sample statistics and margin of error |Testing Key Explanation; Choice A is correct. Increasing
Point: Understanding basic study design and the sample size of the study would improve
interpreting sample proportion

wwvt.vibriintpublisbefS- tOftl 157

Telegram: SAT_pdf
I PROBLEM- SOLVING AND DATA ANALYSIS

the accuracy of their study and thus reduce the Detractor Explanations: Choices A and E are
I
margin of error. incorrect. These statements did not consider the
education level. Choice D is incorrect because
Detractor Explanations: Choices U and C are substantial information is given to make a
incorrect . These options would increase the conclusion.
margin of error, Choice D is incorrect. This
would make the study biased.
244. Level: Easy| Ski 11 / Knowledge : Inference from
sample statistics and margin of error|Testing
242. Level: Medium | Skill / Knowlodgc: Inference Point: Making inferences from sample data
from sample statistics and margin of error
Testing Point : Extrapolate from sample data to a Key Explanation: Choice B is correct . From the
population .
survey 56% of the sample group of residents in
city X voted yes to building the park. This would
Key Explanation ; Choice C is correct. The approximate to 56% of the towns population
sample proportion of I 7.fi% is the percentage which is 60,000( 56%) - 35,6110. This is the
of households in Lhc representative sample with estimated number of residents in city X that
cars. Multiplying Lhis percentage to the total would vote for building the park.
number of households in City X yields 0.17S X
21,000 - 3,738 , Dis tractor Explanations; Choice A is incorrect.
This statement is false because 26,400 is the
Distractor Explanations: Choice A is incorrect estimated number of people whu are againsL
and may result from multiplying 10% instead of building a park in the middle of the town.
17.S%. Choice R is incorrect and may result from Choice C is incorrect - This statement is false
calculating half of the actual estimate Choice D because 56% of the sample group voted yes
is incorrect and may result from calculating twice Choice D is incorrect - This statement is false
Lhc actual estimate. because is not the number of residents hut the
percentage of residents that voted yes.
243. Level: Medium | Shill / Knowledge: Inference
from sample statistics and margin of error
Testing Point; Making inferences from sample
data

Key Explanation: Choice C is correct. The study


cart only be generalized to elementary school
teachers.

Digital SAT Math Practice Questions 158

Telegram: SAT_pdf
I Evaluating statistical claims: observational studies and experiments
I
A study took a group of students and randomly A random survey took a sample of 1,800 patients
divided them into two groups. The first group who were going ior surgery and advised 1,000
contained 1,000 students and the second 600 of them to take a particular medicine for a week
students. One group was told to wake up early before the surgery day. For the rest 800, the
in the morning and study, while the other group medicine was not given. They also collected
was told not to wake up early in the morning, the data about the quick healing ( healing before
Researchers then compared how each group gut three days) of patients whu took the medicine
grades on the class test. Out of 1 ,000 students for a week. Out of the 1,000 patients who took
in the first group, 852 students improved their the medicine, quick healing was observed in 808
grades and in the second group, 48 improved patients, and out of the 800 patients who did not
their grades. What conclusion can he drawn take the medicine, quick healing was observed
from this experiment ? only in 72 patients. What conclusion can he
drawn from this experiment ?
( I ) Most students who did not wake up early
1. Most patients whu did not take the medicine
improved their grades. were healed quickly.

(11) Most students who did not wake up early 2. Most patients who did not take the medicine
have not improved their grades were not healed quickly.
3. Most patients who Look the medicine were
(111) Most students who wake up early have healed quickly.
improved their grades.
A ) T alone is true
A ) T alone is true
B ) U alone is true
R ) I and TT are true
Cl ) I and 111 axe true
C) I and 111 are true
D) II and 111 are true
D) 11 and U1 are true

wwW, vibrantpublisbefi-COftl 159

Telegram: SAT_pdf
I PROBLEM- SOLVING AND DATA ANALYSIS
I
A scientist wants to study how the amount A random survey took a sample of 1,600 patients
of water humans drink affects their diets
, who suffer from a particular disease. 900 of
'therefore, Lhc scientist studies a group of them are to lake a particular medicine for two
people’s daily water drinking usage. Which of the months. For the rest 700 the medicine was not
following is true: given. They also collected the data after two
months about the curing from disease of patients
A ) This is an example of an observational study who look the medicine. Out of the 900 patients
only. who took the medicine, curing from the disease
was observed in 8 Hi patients and out of the
B) 'Ibis is an example of an experimental study
700 patients who did not Lake the medicine the
only.
curing was observed only in 56 patients. What
C) This is an example of both an observational conclusion can be drawn from this experiment ?
study and an experimental study
1 , Most patients who did not take the medici ne
D) This is neither an example uf an were cured.
ubscrvalional study nor an example uf an
2, Musi patients who did noL Lake the medicine
expert mental study.
were not cured.
3, Musi paLienls who look the medicine were
cured

A) 1 alone is true
B) 11 alone is Lrue
C) I and HI are true
D) IT and ITT are true

Digital SAT Math Practice Questions 160

Telegram: SAT_pdf
I Evaluating statistical claims: observational studies and experiments
I
A fitness company has a running drink, which
is consumed by 10 runners in the race. 7 of the
10 runners performed significantly better in
their races compared to other runners who did
not consume the drink. Which of the following
statements can bestbe concluded about the
running drink?

A ) 'ihe drink helps all runners perform belter in


their races.
B ) To perform better in races, runners should
lake the running drink *

C) rlhe running drink cannot be linked to the


runner’s pcrfurmance *

D ) The running drink improves the running


performance of some of the runners.

wwwi vibrdntpublisbefS- tOftl 161

Telegram: SAT_pdf
I PROBLEM- SOLVING AND DATA ANALYSIS

245. Level: Easy | Ski 11 / Knowledge: Evaluating Detractor Explanations: Choices B, C, and
I
statistical claims: observational studies and D are not correct , In experimental studies,
experiments | Testing Point: Evaluating researchers introduce an intervention and study
experimental studies the effects.

Key Explanation: Choice D is correct. II is true 248.Level: Easy | Skill / Knowledge: Evaluating
as only or 11% of the students who did not statistical claims: observational studies and
' 600
352
wake up early have improved. Ill is true as experiments | Testing Point: Interpreting sample
1.000
data results
OT 35.2 % of the students improved their grades.

Key Explanation: Choice D is correct , 11 is true


Distractor Explanations: I is not true since only
43 as only or 3% of the patients who did not
— or 8% improved their grades. 700
600 lake the medicine got cured, rest 92% not cured.
310
Ill is true as or 90% of Lhe patients who look
246. Level: Easy | Skill / Knowledge: Evaluating 900
statistical claims: observational studies and the medicine got cured .
experiments | Testing Point: Making inferences
from experiment data Instructor Explanations: I is not true since only
72
or 9% whn did not take the medicine got
Key Explanation: Choice D is correct. II is Lruc cured.
72
as only or 9% of the patients who did not
Lake the medicine have healed quickly . Ill is true 249, Lcvel: Hard | Skill/ Know ledge: Evaluating
HAS
as or 80, 3% of the patients have healed Statistical claims: observational studies and
i QQQ
quickly. experiments | Testing Pnint: Evaluating
experimental study data
Distractor Explanations: 7 is not true since only
72 Key Explanation: Choice D is correct , Since
or 9% healed quickly.
300 only 7 out of 10 runners performed better after
consuming the drink, the drink did nut enhance
247, Level: Easy | Skill / Knowledgei Evaluating the performance of all of them . Therefore, some
statistical claims, observational studies, and but not all of the runners noticed a significant
experiments | Testing Point: Determine the improvement in their races after consuming the
difference between observational studies and running drink .
experiments
Detractor Explanations: Choice A is
Key Explanation: Choice A is the correct incorrect . Not all runners performed better
answer. In an observational study, researchers after consuming the running drink . Choice B
study how participants perform certain behaviors is incorrect . This implies a cause-and-eflect
or activities without telling them what methods relationship between the running drink and the
runner’s performance. Choice C is incorrect .
or behaviors to choose.
The result on the 7 runners concludes that the
running drink affects the runner’s performance.

Digital SAT Math Practice Questions 162

Telegram: SAT_pdf
Chapter 6
Geometry and
Trigonometry
This chapter includes questions on the following topics:
• Area and volume
• Lines, angles , and triangles
• Right triangles and trigonometry
• Circles

Telegram: SAT_pdf
I GEOMETHY AND TRIGONOMETRY
I
Jen is choosing new triangular tiles for her Eliza is moving to a new house. Tf her living
bathroom floor. She has two options as shown room has an area of 175 square feet and she has a
below. The two tiles, triangle A3C and triangle couch and table that measure 10 x 7 feet in total ,
rtitc;, are similar triangles. If Jen needs enough how much empty space will she have left in her
tiles to cover 540 square inches of the floor in living room once she arranges the couch and
her bathroom, how many mote tiles of triangle table?
din; would she need to purchase if she chose that
option over triangle ABC ? A ) 70fii

A B) 105/?

C) 150/?

D) 175/?
12 inches X inches

B & inches C

4 inches 5 inches

b 7. inches c
4
( Note: Figure is not drawn to scale )
Note: Figure not drawn to scale

What is the total area of the figure above?

A) »
B) 12
C) 6
Cindy is choosing a pinata for a birthday party
The pinata she wants to buy is a sphere with a D) 14
radius of 7 inches. If she wants to fill the pinata
with 1 ,400 if? of candy, how much empty space
will there be inside the pinata, in Round to
the nearest integer, (Take n = )
22

Digital SAT Math Practice Questions 164

Telegram: SAT_pdf
I Area and volume
I
The width of a rectangular bulletin beard being
decorated for a football game is (a ) meters, and
the length ( JO is 3 meters shorter than twite
the width. Which of the following expressions
represent the area of the bulletin board ?

A ) a( 2a + 3 )

-
B } a( 2a 3 )
In the figure above, the circle is inscribed within
C ) 2a1 - !
the square. If the diameter uf the drde is 12,
13) 4d + 3 what is the area of Lhc shaded region ?

A ) 144
13) 36 rr

C) 144 - 36JT

D) 36 - 9w
John wants to put a new carpet in his movie
room, 'Ihe movie room is 6.25 yards long and
4.37 yards wide. The carpel costs $9.75 per
square foot. What will John pay to put the new
carpet in his movie room ?

A ) $266.30
The volume of a particular cube is 27 cubic
13) $2396.67 inches. What is the area of the base of the cube in
C ) $27.35 square inches?

D) $245.81

If the circumference of a circular region equals


exactly 6 JT inches, which is the area of the region?

A ) 3JT
B) 36ST
C) 12ST

D) 9*

.
mrW vibrtntpublishtfri.COrtl 165

Telegram: SAT_pdf
I GEOMETHY AND TRIGONOMETRY
I
M N The radium of a sphere is equal to the length of
a cube. Tf the cube has a surface area of 54 cm1,
what is the volume of the sphere in terms of n?

A ) 9n
fl ) 36n
C) 27n
D) lfln

I. O

Tn the figure above, LMNO is a square, and


triangle bFO is an isosceles triangle. Tf LP 6,
ivha l is the area of the square?
-
A ) 36
13) 72
C) 24
D) 4H

C D E

Tn the figure above, the area of the circle with


center O is 31JI square units. WhaL is the area of
equilateral triangle ABC?
If a 440m track encompasses a circular socceT
field, what is the area of half the soccer field ? ( use /
A ) 142 i 3 square units
:: )

IT as '

^
B) 116 3 square units

A ) 140m

R ) 7,700m
C) 124
^
> /J
square units
D) lt Sv square units

C) lMOOfti
D) 4 ,9mm

Digital SAT Math Practice Questions 166

Telegram: SAT_pdf
I 263
Area and volume
I
Paul has a ball with a volume of 36m Marc has a
higger ball of the same material. If the radius of
Marc's ball is twice the radius of Paul ' s ball, what
is the volume of Marc's ball?

A ) 64 rr
B)
Q l 2ftjr

D ) I 44n

264

A large circular frisbee has a diameter of 2 feel


and a height of 3 inches. Tf Jennifer slacks 15
frisbfiCS on top of eaeh other Lo- prepare fur Lhe
field day, wlial is die volume of lhe slack in cubic
feet ?

153T
A)
1
B ) 45TT

c>
4
ri > 6n

ivww. I’
'

ibrtmipijWjsht'rs.cem 167

Telegram: SAT_pdf
I GEOMETHY AND TRIGONOMETRY

250. Level: Medium | Skill / Knowledge: Area and


I
(7 X7 X 7 )
volume | Testing Point: Finding areas of similar
triangles using scale factor
= 1,437.33.
Key Explanation : To find the difference between Solving the volume of the empty space inside the
the number of tiles needed to cover 540 square pinata yields 1,437.33 - 1 ,400 = 37.33. Rounding
inches, first find the area of each triangular the answer to the nearest integer yields 37.
tile. The area of a triangle can be found by the

2
^
formula A = bh where It - base and h - 252. Level: Easy| Skill / Knowledge : Area and volume
height. Solving Lhc area of triangle AiJC yields Testing Point: Using definitions, properties, and
i theorems relating to area and volume to solve
-( 12}(9) or 54 square inches. To find the area of
problems
triangle ahe, first, determine the value ofz. Using
the properties of similar triangles to determine Key Explanation: Choice 13 is correct . To find
the value nf ? yields the ratio of the sides the space lefL in the room, find the difference
between the area of the living room and the area
—4 = —Z
. Simplifying the equation yields 3 = —
X that will be occupied by the furniture. The area

Multiplying z and dividing 3 from both sides of


that will be occupied by the furniture is 10 x 7 =
9 Itift1. Then , subtract this value from the area nf

Lhc equation yields Z - or z - 3. Solving for the the entire living room which yields 175 - 70 =

^ ^
area of triangle abc yields - ( 4)(3) or 6 square 105 -

inches. To gcL the number of tiles required fur Distractor Explanations: Choice A is incorrect
each option , divide the area of the floor by the and may result from calculating only the area
area of the corresponding tile. Solving for the that will be occupied by the furniture. Choices
540
number of Lrianglc ABC tiles needed yields ——
51
C and D are incorrect and may result from a
or 10 tiles. Solving for the number of triangle abc conceptual or calculation error.
55 )
Liles needed yields ——
6
or 90 tiles. Calculating
253.Level: Medium | SkilJ / Knuwledge : Area and
the difference between the number of tiles
volume|Testing Point: Finding the area of
needed yields 90-10 = SO. composite polygons

251 Level: Easy|Skill / Knowiedge: Area and volume Key Explanation ; Choice D is correct. To soke
Testing Points Finding the volume of a sphere for the area of the whole triangle, determine the
areas of the left and right triangles individually
Key Explanation: The correct answer is and then add them together. Since the triangle
37. The formula to find the volume of a on the left has two equal interior angles and a

sphere is V = JIT 3 , Substituting the given right angle, then it is a 45-45-90 angle. This
data to find the volume of the pinata yields means that the length of the legs is congruent.
So, the altitude is also equal to 4. Solving for the

Digital SAT Math Practice Questions 168

Telegram: SAT_pdf
I I
Area and volume ( Answers )

of the rectangle. To do this, multiply each


I
area of the left triangle yields - ( 4}( 4) = fl. Since
dimension by 3 length = 6.25yds X 3 = 18.75 ft,
the other triangle has a hypotenuse of 5 and a
^
width = 4.37 yds x 3 = 13.11 ft. Find the area of
.
leg of 4 then it is a 3-4-5 triangle. So, the length the rectangle by using the formula A - length x
of the base must be 3. The area of this triangle is width, or 18.75 x 13.11 = 245.8125 ft1 . find the
i ( 4)(3] = 6. 'Ihc combined area is 6 + 8 - 14. cost to pul in the carpet by multiplying the area
by the cost per square foot, 245, 8125 ft 2 x $9.75
D1straclurExplanations: Choice A is incorrect - $2396.671875 . Ihis value rounds to $2, 396.67
and may result from solving only the area of or Choice B,
the left triangle. Choice B is incorrect and may
result from assuming that both left and right Distractnr Explanations: Choices A, C, and

triangles are a 3-4- 5 special triangle. Choice C D are incorrect and are likely the result of
is incorrect and may result from solving only the arithmetic errors in converting units.
area of the right triangle.
256. Level : Easy | Skill / Knowledge: Area and volume
251. Level: Easy | Skill / Knowledge: Area and volume Testing Point: Using circumference of a circle to
Testing Point : Convert English into Algebra to find its area
solve area problem
Key Explanation; Choice 13 is correct. If the
Key Explanation: Choice B Is correct. The circumference is exactly inches, find the
formula to find Lhe area of a rectangle is length X radius using ihe formula of circumference which
width. Since a - width and b - length., the area is C - 2 nr where C is the circumference and r is
of the rectangle is a Limes b. However, since the the radius. Substituting 6JT to the formula yields
length Ms 3 meters shorter than twice the width, 6sr = 2nr. Dividing both sides of the equation by

i? can be expressed as 2a - 3 . Substituting 2a - 3 2JT yields 3 = r or r = 3- To find the Area, use the

for ii in the area equation yields formula A - nr2. Substituting the radius yields A
Area = A x t = fl x (2fl - 3 ) = ii( 2*i - 3} or Choice = nOy = 9n .
R.
DLstracLor Explanations: Chokes A, B, and C
Distractor Explanations: Chokes A , C, and are incorrect and reflect errors in calculating the

D are incorrect and are the result of errors circumference and area of a circle.
in converting the English into Algebra or
calculating wrong the area of a rectangle, 257. Level: Hard | Skill / Knowledge: Area and volume
Testing Point: Finding the area of shaded region
255. Level ; Easy | Skill / Knowledge: Area and volume with inscribed shape
Testing Point: Find area rectangle and units
conversion
Key Explanation; Choice C is correct . Find
the area of the circle by using the formula A =
Key Explanation: Choice E is correct. The first nr2. Since the diameter is 12 , then the radius is
step is to convert yards to feet for the length ~ = 6 - Substituting the radius to the formula

wwW .vitirCrntpijWisherittUrtt 169

Telegram: SAT_pdf
I GEOMETHY AND TRIGONOMETRY

yields A - n (6s) = 36n _ Newt, find the area of Detractor Explanations: Choices A, C, and
I
the square by using the formula / = s1 . Since the D are incorrect resulting from conceptual and
circle is inscribed in the square, the diameter of arithmetic errors.
the circle is equal to the side length of the square.
This means that s = 12. Substituting the length of
260. Level: Hard | Skill / Know ] edge : Area and volume
the side to the formula yields A = 121 = 144 . Now
Testing Point: Fbiding the area of a circle given
subtract the area of the circle from the area of the
its circumference
square which yields 144 - 36n.
Key Explanation: Choice B is correct . The
Distractnr Explanations: Choice A is incorrect
length of the track represents the circumference
and may result from solving the whole area of
of the soccer held, Getting the diameter of idle
the square. Choice B is incorrect and may result
from solving Lhc area of the circle. Choice D
circular field yields 440m - JID or D 440. — —
Multiplying ? and dividing 22 hm Koth
sides of
is incorrect and may result from using 6 as the
the equation yields D - 140. Hence, the radius
diameter of the Circle and Lhe side length of the 140
of the soccer field is m or 70 m. ' Ihe area
square. :
of half of the circular field would therefore be

25fJ. Level: Easy|Skill / Knowledge: Area and volume


2 7
Testing Fuinl: Finding ihe area of a square prism
Distractnr Explanations: Choice A is incorrect

Key Explanation: The correct answer is 9, To This is the diameter of the soccer field.

find the area of the base, find the length of one Choice C is incurred. This is the area of the
of the edges of the cube. Since the volume is j1 = whole circle.
27, it follows that j = $7 = 3 , Since the length
Choice D is incorrect - This option is a result of a
of one of the edges of the cube is 3, we use the
calculation or conceptual error.
formula for the area of a square. Therefore, the
area of the base of the cube is A = s* = (3) 1 = 9.
261.Level: Easy | Skill,' Knowledge: Area and volume
Testing Point: Finding the volume of cube given
259.Level; Easy|Skill / Knowledge; Area and volume
surface area
Testing Point: Using theorems relating to
squares and isosceles triangles to solve problems Key Explanation: Choice B is correct. The
surface area of a cube is given by the formula
Key Explanation: Chuice B is correct . In the
6 ns. given that a is one side of the cube. Since the
figure, LMNO is a square, and triangle LPO is
surface area of the cube is 54, 6n! = 54 , Dividing
an isosceles. If LP = 6, then PO = 6. Using the
both sides of the equation by 6 yields n* = 9.
Pythagorean theorem to find the length of LO Getting the square of both sides of the equation
yields 6: + 52 = LO? , Simplifying the equation
yields a = 3, The radius of the sphere will be 5cm.
yields 72 = LG?, Getting the square root of both
The volume of a sphere is given by the formula
V
sides yields 72 = LO. Since LO is a side of the
square LMNO,the area of the square is 72 * or —43 3
jrr which results to —4
(3)3 = 36ir.
72. ^ AT

Digital SAT Math Practice Questions 170

Telegram: SAT_pdf
I Distractor Explanations: Choices A , C, and D
Area and volume ( Answers )

263. Level : Medium | Skill / Knowledge: Area and


I
are incorrect. These options maybe a result of a volume | Testing Point: Working with similar
conceptual error or miscalculation . geometric three dimensional shapes

Key Explanation: Choice U is correct. Ihe ratio


262 . Level: Rasy | Skill .' Knowledge: Area and volume
of the radius of Paul’s ball to Marc’s ball is 1:2 ,
Testing Point: Using area of circle and concepts
Therefore, the ratio oftheir volumes would he
ni equilateral triangles
1 !:2 J or 1 :ft. Since Pauls hall has a volume of 36ST,
Volume of Marc' s ball g.
Key Explanation: Choice D is Correct . the ratio will he
36 A 1
Area of the circle - nr*
Multiplying both sides of the equation by 36ir
If r is the radius, nr1 - 81 JT which yields r 9 — yields Volume of Marc's hall - 2H8TT.
Diameter AD = 18
Triangle ARC is equilateral. Distraclur Explanations: Choice A is incorrect.
Angle ADR - 9< 1D Ihis is the value when you use the raLio of the
Hence AD becomes altitude of ABC
radius to End the volume of Marcs ball. Choices
If a is the side of the equilateral triangle then C and D are incorrect and may result from a
J3 conceptual or calculation error,
altitude - ~
ffA [formula ]
& a - 18
^o ;
2 264. Level : Easy|Skill / Knowledge: Area and volume
36 Testing Point: Rinding the volume of a cylinder
“ '

ft
2 1296 Key Explanation: Choice A Is correct. First, find
tf ~
- 432
3 the volume of one frishee hy using the formula
By formula, area of the equilateral triangle is Volume - m^ h . The diameter is 2 ft , therefore, r
- 1 ft. Tire height is 3 inches; converting it to feet
— Q
1
square units.
3 in 1
_ 4 32 >/—3 - lOWi square uniLs
,
yields h — ft orh ——ft . Hence, the volume of
So area ,
jrl^l
Hence Choice D is correct ,
one frisboe is
4
— n4 . therefore, the volume of

13 frisbees is
Distractor Informations: Choices A, B arid C 1
are incoiTect because when we calculate the area Distractor Explanations: Choice B is incorrect
they do not equal to lOSVs. and may result from solving the volume ol 180
frisbees . Choice C is incorrect and may result
from solving ihe volume of 1 irisbee. Choice D is
incorrect and may result from solving the volume
ol 24 frisbees.

11« vibfiiotptjbUstiers.ciMi 171

Telegram: SAT_pdf
I GEOMETRY AND TRIGONOMETRY
I

( Note: Image is not drawn lo scale.)

— —
If ZA 62“ and Z.Q 51", what is Lhc value of
Zt ?
In lhc above figure, if x + y - 1081’ then whal is
ihe value of z + w (indegrees) ?

A) 67“
A) 108
B) 113°
B) 72
C) 118°
C) 252
D) Cannot he determined
D) 282

Digital SAT Math Practice Questions 172

Telegram: SAT_pdf
I Lines, angles, and triangles
I
c*

z A

Id the figure above, lines A and B arc parallel, c =


35, and d = 55. Which best describes the shaded
triangle type?
{Note: Figure nol drawn lo scale)

A ) Isosceles Right Triangle


li) Scalene Right Triangle
C) Equilateral Triangle
D ) Scalene Obtuse Triangle
Note: Figure is not drawn to sthale

In Lhe figure above, whaL is Lhe value ufy?

A ) 103"
a) 7T
H
C) 13"
P
D ) iir

D
c
In the figure above, assuming Et C, and D an:
colli near, what is the sum of * + y ( in degrees)?

According to the triangle above, what is the value


of side Z?

rnvW, vibrdntpublisbefS-tOftl 173

Telegram: SAT_pdf
I GEOMETHY AND TRIGONOMETRY
I

Tn the figure above, the length of KL = the Length


of ML. What is the value crt i in degrees ? In the figure above, Irlanglc XYZ is inscribed in
the circle with center W and diameter XT. Tf YZ
A} is = YW, how many degrees is angle X\VY?
B) 76
C ) 70
D) 104

/ /
If LMNO is a parallelogram, what is the
difference between ZAflVG and ZLMN ?

A ) 7'
B) 44"
C) 68"
In the above figure AB and CD are parallel Lines,
fTFand GH are two transversals. D) 70"
What is b + d in terms of a and c

A} 130 + a + c
B) 180 - a - c
C) 180 - fl + c

D) 180 + 2( n + c )

Digital SAT Math Practice Questions 174

Telegram: SAT_pdf
I Lines , angles , and triangles
I
The measure of angle P is —
IS
n radians. What is
the measure of angle Pin degrees?

If line m 11 line n, what is Lhc value of angle z?

m
c/ b

d z
n
y/

Point A has coordinates of (6, 4) and Point B has


coordinates of (0, -8 ). Point C is the midpoint
of segment AB. Which one of the following lines
does Point C lie on?

A ) y = JC + 4
B ) / = JC + 3
C) jv =x-5
D) y = x - 2

www.Yibrnntpublisheri.com 175

Telegram: SAT_pdf
I GEOMETHY AND TRIGONOMETRY

265 . Level: Easy| Ski 11/ Knowledge: Lines , angles, Substituting the values of ZGIE , ZEGI and
I
and triangles|Testing Point; Using concepts of -
ZGEI yields 72° + 180° - % + 1 SO" w = 180°
supplementary angles and angle sum property of
Combining like terms and simplifying the
triangles to solve problems
equation yields 252“ = z + w .

Key Explanation: Choice 11 is correct . The


Distractor Explanations: Choice A is incorrect
exterior angle theorem states that the measure
and may result from solving for the sum of
of each exterior angle of a triangle is equal to the
ZEGI and ZGEI . Choice B is incorrect
sum of the opposite and nnn-adjacent interior
and may result from striving for Lhc value of
angles. Sci, the value of Zx is the sum of £A
ZGIE . Choice D is incorrect and may rcsidt
and ZC - Therefore, the value of Zx is SI “ 4- 62a
from a conceptual or calculation error
= 113°.
Distractor Explanations: Choice A is incorrect 267. Level; Easy|Skill / Knowledge: Lines, angles,
and may result from calculating the value of and triangles|Testing Point: Using concepts of
ZB . Choice C is incorrect and may result from supplementary angles and angle sum properly of
calculating the sum of ZB and ZC . Choice D triangles Lu solve problems
is incorrect because all the needed data can be
obtained from the problem . Key Explanation : Choice A is correct. The
sum of the interior angles of a triangle is 180°.
Therefore, the sum of the interior angles in
266 , Level: Easy | Skill/ Knowledge: Lines, angles,
the figure can be represented byx + 411" + 63°
and triangles|Testing Point: Using concepts of
supplementary angles and angle sum property of
= 180. Subtracting 40 and 62 to both sides of
the equation yields x = 77- Angles x and y are
triangles to solve problems
supplementary angles, which also have a sum of
180" Therefore, y = 13CF - 77" = 103°,
Key Explanation: Choice C is correct. Using
the given equation JC + y = 103° And the value of
Distraetor Explanations: Choice B is incorrect
ZFIH . Applying the Angle sum property of a
because angles x and y cannot be equal. Choice C
triangle in AE1H yields x + y + ZFIH = 180°
is incorrect and may result from calculating the
Substituting the value of x + y yields U)8°+ complementary angle of x - Choice I> is incorrect
ZFIH = 180°. Subtracting 108 from both sides and may result from calculating the sum of x and
of lhc equation yields ZFIH =180° - 108° = 72° 40 °,
Since ZFIH and ZGIE are vertically opposite
angles then ZFIH = ZGIE ,
268. Level; Easy | Skill / Knowtedgei Lines, angles,
Therefore, ZGIE -72° . and triangles | Testing Point: Using properties of
Solving for the angles in AGE1 yields ZEGI = equilateral triangles to solve problems
130° - x and ZGEI =180° - w.
Key Explanation: The correct answer is 3,
Using the Angle sum property of a triangle in
Determine the type of the triangle by solving
AGEI yields ZGIE + ZEGI + ZGEI = 180°.
for the values of the interior angles. Applying

Digital SAT Math Practice Questions 176

Telegram: SAT_pdf
I Lines , angles , and triangles ( Answers )

the Angle Sum Theorem yields 60° + x + x =


I
271. Level: Medium | Skill / Knowledge: Lines, angles,
ISO". Combining like terms and subtracting 60° and triangles | Testing Point: Using concepts of
from both sides of the equation yields 2x = 120°. supplementary angles, isosceles triangles, and
Dividing 2 from both sides of the equation yields angle sum property of triangles to solve problems
x = 60fl. Since all the interior angles are the same,
the triangle is an equilateral triangle. Since an Key Explanation; Choice ft is correct. First, find
equilateral triangle also has equal sides, Z = 3, the values nf ZAf & ZfC - Since AKLM is an
isosceles triangle, ZK Sc ZM are congruent
to each other. Since the sum of the inside
260. Level; Medium | Skill / Knowledge: Lines, angles,
angles in a triangle is 180", then 130 -110 =
and triangles | Testing Point : Use properties
ZK + ZM . Simplityingthe equation yields
of triangles to sulve triangles and determine
70 = ZK + ZM .Since hoth angles are equal,
triangle type
ZK — ZM = 70 2 = 35. lienee, ZK & ZM
arc both 35*. To hud ZNOM , find the sum of
Key Explanation: Choice E is correct. If the
ZM and ZN and subtract the sum from 130,
mZc - 35" and the mZd ~ 55" then the
This yields l &G - ( 41 + 35) = 1041’. Since ZJC is
measure of the angle on the bottom right of the
Lhe supplement of ZNflAf , then Zx - 180 - 104
big triangle is the same as the measure of angle c,
or 35 degrees since lines A and B are parallel and = 76.
these two angles arc corresponding angles and
Distractnr Explanations: Choice A is incorrect
thus are equal Since a triangle has ISO degrees,
and may result from solving the value of ZK
the measure of angle e is (130 - 35 - 55 ) or 90".
and ZM . Choice C is incorrect and may result
Since Ze measures 90 degrees, it is a right angle,
.
from solving the value of ZNLK Choice D is
Since all of the angles have different values, the
ineorrect and may result from solving the value
triangle is a scalene right triangle.
of ZNOM .

Distractnr Explanations: Choices A, C, and D


are incorrect - Choice A assumes incorrectly that 272. Level : Medium | Skill / Knowledge : Lines,
two sides of the triangle are congruent. Chokes angles, and triangles | Testing Point: Using
C and D do not take into account the right angle. concepts of supplementary angles and parallel
lines to solve problems

270. Level; Easy | Skill / Knowledge; T ines, angles,


Key Explanation; Given that A R \\ CD and EF is
and triangles | Testing Point; Use concepts of
a transversal , then a + b = 130“ si roe a and b are
supplementary angles and linear pair
co-interior angles.
Hence, b =180 - a.
Key Explanation: The correct answer is 42. A
Given that GH is a transversal for AB and CD,
straight line has i 80 degrees. Therefore:
then d = c since c and d are alternate interior
63 + x + 75 + y = ]00
angles.
x +138 = 130
Therefore, b + d = 180 - a + c.
x + y = 42 ,

wwW, vibriintpublisbefS .tOftl 177

Telegram: SAT_pdf
I GEOMETHY AND TRIGONOMETRY

Distractor Explanations: Choices A , B , and D Distractor Explanations: Choices A, C, and D


I
are incorrect and most likely result from errors in are incorrect and reflect errors in interpreting
calculating the value of b and d. angle relationships.

273. Level: Medium | Skill / Knowledge: Lines, angles, 275. Level : Easy | Ski 11 ,' Know ] edge : Lines, angles,
and triangles|Testing Point: Using concepts of and triangles|Testing point i Converting from
supplementary angles, equilateral triangles, and radians to degrees
angle sum property of triangles to solve problems
Key Explanation: To convert radians to degrees,
Key Explanation: Choice D is correct Since the use the formula x 0 , Substituting the given
figure above shows that triangle X YZ is inscribed
130 7
in the circle with center W and diameter XZ , angle yields n: or 70' , 1

K IS

then YW 7.W because they are both radii. Since
Y7. - YW - ZW, then triangle WTZ is equilat - 27d. Level : Easy | Skill / Knowledge : Lines, angles,
eral which means all angles are congruent. Since and triangles|Testing point i Finding the
the sum of the inside angles of a triangle is ISO0, measure of angles in parallel lines
each angle is 6tl“- Hence, ZZWY -60°. Since
/ X WY is the supplement of ZZWY, then Key Explanation: Angles on a straight line add
ZXWY ~ 130 - 60 = 120". up to 180". Iherdbre . - 180 - 125 - 55°.
Since Zc and Zz arc alternate interior angles,

274. Level: Easy | Skill / Knowledge: Lines, angles, they are equal. Therefore, z 55°- -
and triangles|Testing Point ; Using parallel line
theorems to solve problems 277, Level: Medium|Skill/ Knuwlcdge: Lines,
angles, and triangles | Testing Point: Calculate
Key Explanation: Choice B is correct. Since midpoint of a line segment and knowledge of
LMNO is a parallelogram, we know that there are graphs
two sets of parallel lines being cut by two parallel
transversals. Based on the Corresponding Angles Key Explanation : The coordinates of the
theorem , ZMNO = 16*. Since ZION and midpoint of a line segment is calculated by taking
ZLMN are opposite angles of a parallelogram , the average of the x coordinates and the average
then ZLON = ZLMN = 9 x + 5. Since ZMNO is of the y coordinates of the two points on the line
a supplement to ZLMN > then 16x + 9* +5 = 180 segment. Thus, the midpoint of segment AB is
or 25 x + 5 = ISO- Subtracting 5 from both sides
of the equation yields 25x = 175. Dividing both
given by ~ ~
2 )° ^
r
^etenll'ne
t

sides of the equation by 25 yields x = 7. It follows the correct equation the midpoint coordinates
that ZMNO = 16x = 16(7 ) = 112 and ZLMN lie on, substitute (3, -2) into each of the answer
= 9x +5 = 9(7) + 5 = 63 + 5 = 68. Therefore the choices for the variables x and y and determine
difference between the two is 112 - 63 = 44. which answer choice makes the equation true.
The only answer choice that makes the equation
true is Choice C as -2 = 3 - 5 or -2 = -2.

Digital SAT Math Practice Questions 178

Telegram: SAT_pdf
I Lines , angles , and triangles ( Answers )

Therefore, the midpoint lies on the line = jr - 5 .


I
^
Distractof Explanations: Choices A, It, and
D are incorrect and are the result of errors in
calculating the midpoint of a line segment or in
substituting points into equations.

itmt!vibrdntpublisbefS- tOftl 179

Telegram: SAT_pdf
I GEOMETHY AND TRIGONOMETRY
I
ID right triangle ABC, if Z.B - 90° and In a right triangle, if Zx + Z.y = 90° and

:—
Cos A = then what is Sin C? COSJf

= 12 , what is the value of sin y*.
A} I
1
E
’ 72 I
c> 5

D,— Which uf the following is NOT equal to sin — ?

A)

~
B1 cos —
ij

llJT
C ) cos —6
A B
D , *- LT

10m

K
ISO"
Tn a right triangle, one angle measures x° where

D C tan J(“ = 1, What is sin ( 90 - x°) to the nearest


thousandth?
Tn the above figure, quadrilateral AflCDis a
parallelogram. AE is perpendicular to CD and
A£ = 10 meters is the length of BC?

A ) 10m

B) 10 V3m
C) 20m
D) 20V3m

Digital SAT Math Practice Questions 180

Telegram: SAT_pdf
I Right triangles and trigonometry
I
-2 -
If sin a = 0.6 and sin ( 90 - a ) = 0, 3, what is the
value of tan u ?
-7 -6 —5 1 2 3

AP is a line tangent to the circle helow. If AO =


60!? and the length of arc length AS is 2n, what is

Lhc length of OP iri CtMl

In the graph above, a right triangle is formed


with one leg tangent to thejg-aiis. What is
the length of n which is the hypotenuse of the
triangle?

A) 2 2 V
U) 4

C) 1 J 2 A ) 12 cm

D} 2 B ) 13 cm

^
C) 6 3 cm
D > ^
6 2 cm

If sin « = otw 26, what is lhe value of «?

A ) 64
a) 334

C) 134
D) 206

www!vibriintpublisbefS .tOftl 181

Telegram: SAT_pdf
I GEOMETHY AND TRIGONOMETRY
I
Triangle PQE and triangle DEFare right- angled Triangle DSE has angles r , and 90°. What is
triangles. If DE corresponds to PQ, which of the value of cos f - sin p* ?
the following statements would make the two
triangles similar? A) 0
B) 1
A ) The length of line DE is twite the length of
line PR C) 'll
B) The Angle FDJi = 60° D) &
C) The sin of angle QflP is equal to the sin of
angle EFD
D) The length of the line PR is Lwice Lhc length
of the line DF

In the diagram shown below, f ^ilT and PQR are


right angles and ST is parallel to QE . What is the
value of tns fl?

IJ

f,

A) -

« !
3
c) ;
D >;

Digital SAT Math Practice Questions 182

Telegram: SAT_pdf
I Right triangles and trigonometry ( Answers )

278. Level: Easy | Skill / Knowledge: Right triangles Therefore, BC = 20 m.


I
and trigonometry | Testing Point: Using
concepts and properties of trigonometric ratios Distract or Explanations: Choice A is incorrect
and identities to solve problems because it is the length of AE. Choice B is
incorrect because it is the length of DE. Choice
Key Explanation; Choice C is correct. Since D is incorrect and may result from a conceptual
Cos A - - , then A - 60". or calculation error.

That means C = 30° [since B = 90°).


1 280. Level: Medium | Skill / Knowledge: Right
Therefore, Sin C = Sin 30“ .
: triangles and trigonometry | Testing Point :
Using definition of trigonometric identities Lo
Distractor Explanations: Choice A is incorrect solve problems
and may result from solving the value of Sin
R. Choice B is incorrect and may result from Key Explanation: If two angles are
solving the value nf Cos B. Choice D is incorrect complementary, the cosine of one is equal to
and may result from solving the value of Sin A nr the sine of the other. So, sin y - cos x, 'therefore,
5
Cos C siny = :—:
279, Level: Hard|Skill / Knowledge: Right triangles 281 . Level : Easy | Skill / Knowledge: Right triangles
and trigonometry|Testing Point: Using and trigonometry|Testing Point!Use
concepts of supplementary angles, angle sum trigonometric ratios with unit circle to solve
property, and specia triangles to solve problems problems

Key Explanation: ChoiceC is correct. Key Explanation: Choice D is correct. Using


Since adjacent angles of a parallelogram are the unit circle or a calculator ( in radians)
supplementary, then ZD = ISO11 - ZC .
Substituting the value uf ZC yields ZD = ISO” - —3 —2
sin = or 0,8660-
ISO1' = 30*. Since A£ 1CD , then ZAED = 90*.
Applying the angle sum property' of a triangle Since sin —3 = ->/3/ 2* —2 , Choice D is
to find the value of ZPAEyields ZDAE = ISO" correct.
- 30* - 90“ = 60“ Therefore, AAED is a special
right triangle ( 0 - 60 - 90), Distractor Explanations: Choices A, B, and

We know that the ratio of the sides will be


C are incorrect and are the result of errors in
interpreting trigonometric ratios on the unit
circle.
Given the length of AE - 10 meters, then the
length of AD = 2 times the length of AE which is
2 x 10 = 20 . 282. Level: Medium | Skill / Knowledge: Right
triangles and trigonometry | Testing Point:
Since the length of opposite sides of a
Using properties of trigonometric ratios and
parallelogram are equal, then AD = BC.
identities to solve problems

.
wwW vibriintpublisbefi .COftl 183

Telegram: SAT_pdf
I GEOMETHY AND TRIGONOMETRY

Key Explanation : Since tan Jt" = 1 , then x° = 285.Level: Easy| Skil I / Knowledge : Right triangles
I
tan 1 (1). and trigonometry | Testing Point: Using
trigonometric identities to solve problems
Hence, if = 43°.
sin a
Therefore, sin ( 90 - if ) = sfo (90° - 45°) = sin 45° Key Explanation: Tan a is equivalent to
ana
'

1 yjl 1.414
— —2 =
= f= = = 0.707 ( nearest thousand L It ) Using the identity sin a = cos(90 - d), sin(90 - a )
^ 2
is tns a. Therefore, tan a =
^
'T .i
= 2.
233. Level: Easy| iihill / Knupledge : Right triangles
and trigonometry | Testing Point: Using
.
233- Level: Medium | Sk ill ' Knowledge: Right
concepts of special right triangles lo solve
triangles and trigonometry|Testing Point;
problems
Using circle proportionality theorems and special
triangles
Key Explanation : Choice C is correct. Eased
or the given graph, the legs of the triangle both
Key Explanation: Choice A is correct. hirst ,
measure 4 units. Since the legs are congruent,
find angle AOP using arc length of AiJ and the
this is Lin isustdes right triangle, Ihis means that
radius of the circle. Arc length is found using the
it is a 45v - 45“ - 90” triangle whose sides have a
raLio of JC : x : Ws , Since Lhe lengths of the kgs formula ^
330
TTD, where D is the diameter of the
arv 4 units, n - 4 * jl .
circle. Since AO is the radius of the circle and AO
= 6, thenD = 6( 2 ) = 12 . Substituting thediamcLcr
Distractor Explanations: Choices A E and D
ZAOB
lo the formula yields ?r ( l 2) — 2ir .
are incorrect and reflect errors in interpreting 33( 1
angle and side relationship for geometric shapes Multiplying to both sides of the equation
or a graph . 1 2 ,T
yidds ZAOB = 60 ", Triangle A OP is a right-

234. Level: Easy|Skill / Knowledge: Right triangles


angled triangle and one of the angle AQP 60° —
making the triangle a special 90-60 30 triangle.
and trigonometry | Testing Point: Using
Looking at the triangle, OP is the hypotenuse.
trigonometric identities
Using the special triangle identity, OP = 2AO or
OP = 2(Gcm} = 12cm,
Key Explanation ; Choice A is correct. Eased on
the identity sin x = cos ( 90 - x ) , the angles a and Distractor Explanations: Choices B and D are
26“ are complementary. This means that a - 90 -
incorrect and these may be due to conceptual or
26. Therefore, a = 64°. calculation errors. Choice C is incorrect. This is
the value ofAP,
Distractor Explanations: Choice B is incorrect
and may result from determining the angle
287, Level; Easy | Skill / Knowledge; Right triangles
conjugate to 26“. Choice C is incorrect and may
result from determining the angle supplement to
and trigonometry | Testing Point: Working with
26°. Choice D is incorrect. This option would be
similar right triangles

a result of conceptual or calculation errors.

Digital SAT Math Practice Questions 184

Telegram: SAT_pdf
I Right triangles and trigonometry ( Answers )

Key Explanation: Choice C is correct. In similar


I
289. Level: Easy | Skill / Knowledge: Right triangles
trianglesj the matching angles are equal. If the sin and trigonometry|Testing Point; Using
of angle (JfiP is equal to the sin of angle EFD, the trigonometric identities to solve problems
angle QRP and angle EFD are equal.
Key Explanation: Choice A is correct. Using ihe
Distraelor Explanations: Choices A , B, and D —
trigonometric identity, sin x cos (90 - x ), then r
are incorrect. They do not prove that the two and p are complementary angles. Therefore, cos r
triangles are similar. = sin p which means COST - sin p = 0-

Distraclor Explanations: Choices B, C, and


288. Level: Easy | Skill / Knowledge: Right triangles
D arc incorrect , these options may be due to
and IrigonomeLry |Testing Point: Using similar
conceptual issues.
triangles and trigonometric ratios to solve
problems

Key Explanation: Choice D is correct. Triangle


PQR is similar to triangle PST which means that
cos PTS - cos PRQ. Using SOHCAHTGA, cos
ST
PTS = Using Pythagorean theorem a1 + tT

= yields 9* +- ST1 - 15\ Subtracting P from



both sides of the equation yields Si* = 151 - 91 =
Md. Celling the square root of both sides of the
equation yields ST = 12cm. Substituting the value
,,
ofr ST yields cos PI S ~ —
ST 12 4
PT 15 5
-

Distractnr Explanations: Choice A is incorrect


This is the value of sin PTS. Choice R is
incorrect. The value of cosine cannot be greater
than 1. Choice C is incorrect. This is the value of
tan PTS .

.
iwW vibrantpublisbefS- tOftl 185

Telegram: SAT_pdf
I GEOMETHY AND TRIGONOMETRY
I
If a circle has a center (1, 3 ) and a radius of ,
which of the following points is an endpoint of
(x - 2 ) + ( y + 1) = 16
2 2

the circle? A student graphed the equation of the circle


above on the coordinate plane. Point O is the
A ) (1, 5) center of the circle and Point N is a point on the
circle. If segment ON is the radius of the circle,
B) (0> 7) which could NOT he a coordinate for Point N?
Q (7, 0)
A ) (2, 3 )
D) ( 7, 1)
B) ( 6, -1 )
C) (-2, -1 )
D ) ( 2, -6)

What is the tenter of Lhe circle represented by the


equation x - &x + yi - 2y - -8?
^
A ) (1, 4)
ft ) ( -4, -1 )

C) ( 4, 1 )
D) (3, 3 )

Note; Figqns not drawn to scale.


A circle at the origin has a radius of \f2 . Which In the circle above, Point A is the center and
ordered pair below represents a point on the
circle? the length of minor arc RC is - of the length
8
of the circumference of the circle. What is the
A ) (2, 0) difference in angles ( in degrees) subtended by
B) ( 1, 1) minor arc BC and major arc BC?

/
C) ( s 2 , 1)

D) (0, 0)

Digital SAT Math Practice Questions 186

Telegram: SAT_pdf
I Circles
I
If a sector's interior an pie is 72° and the radius is If the length of the minor arc subtended by the
5 cm , what is the area of the sector ? Round your angle BOC is 11 ti, what is the circumference of
answer the nearest tenth . the circle below?

The diameter of a circle is 2x\ What is the area of


the circle?

A) six1
A ) 13TT
B
c)
> ice
ice
-
13) 32 in
C) LlOn
D > ice D) 3<5JT

vibrdntpublisbefS- tOftl
itmt! 187

Telegram: SAT_pdf
I GEOMETHY AND TRIGONOMETRY
I
PQ is tangent to the circle below and OP = 12cm .
If the arc length subtended by the angle POQ is
equal to j , what is the value of angle PQO in
JT

degrees?

A ) 25
fl) 60
C) 30

D) fiS

The circumference of circle A is 2k -and the


circumference of circle B is 36fc . What is the ratio
of the area of circle A to circle fi?

A ) l to 36

B) Mo IK
C) 1 to 324

D) 4 to 324

Egg

What is the diameter of the circle whose equation


is ( x - 3) 3 + (y + 4)3 = 64 ?

Digital SAT Math Practice Questians 188

Telegram: SAT_pdf
I
290. Level: Medium | Ski 11/ Knowledge: Circles
Circles ( Answers )

from negating the values of h and ft. Choice


I
Testing Point: Using definitions, properties, and D is incorrect and may result from solving for
theorems relating to circles to solve problems the radius and using it as the coordinate of the
center .
Key Explanation: Choice J) is correct. To find
an endpoint of a circle, find the equation of a
292. Level : Easy | Skill / Knowledge: Circles | Testing
circle from the center and the radius: Choice
Point: Creating and interpreting the equation of
B is correct because it is the only answer, when
a circle
inserted into the above equation , to solve the
circle equation . Substitute x = 0 andy = 7 in the
Key Explanation : Choice B is correct. Writing
equation , we get , the equation is satisfied
an equation that represents the given circle with

Distraclor Explanations: Choices A , C, and


Jl
Lhc center at the origin and a radius of - yields
x1 + y* = 2 . Substituting ( 1, 1 ) into the equation
D are incorrect because when they arc inserted
yields F + V — 2. Simplifying the equation yields
into Lhc above equation , they do not solve the
2 = 2. Since the statement is true, then ( 1 , 1 ) lies
equation.
on the circle.

291. Level: Hard | Skill / Knowledge: Circles | Testing DisLraclur Explanations: Choice A is ineorrecl
Point; Finding the center of a circle using the because subs Lituling ( 2, 0) yields a false
equation of a circle and eompleling the square statement of 1= 2 . Choice C is mooned because
subsLiLuLing ( Vz , 1) yields a false statement of
Key Explanation: Choice C is correct. To find 3 = 2. Choice D is incorrect because (0, 0) is the
the center of the circle, complete the square and center of the circle.
rewrite the expression in the center form: ( jr - fc )J
+ (y - ft )* = rh
293. Level: Hard | Skill / Knowledge: Circles | Testing
To determine the number that must be added Point : Use knowledge of equation of cirde and
to complete the squares, divide the coefficient radius Lo solve problem
of * andy by 2 and square it. This yields


= ( 4)3 = 16 forxand | = (-l)S - l
for y Completing the squares by adding 16 and
,
Key Explanation; Choice 13 is correct. The
equation of a circle is (x - ft } 1 + (y - ft)!= r1,
with the center at ( hT ft ) and T being the radius.
1 to both sides of the equation and grouping the Therefore, the center of the cirde for ( x - 2}J + (y
squares yields (x2 - 8r + 16) + ( y* - 2y +1} = -8 + + 1)!= 16 is ( 2, -1) and the radius is 4. A radius
16 + 1. Factoring the left side and simplifying the of 4 means that a distance of 4 is the farthest
right side of the equation yields (JC - 4)2 + (y - l )1 any point on the cirde can be from the center of
= 9. Therefore, the center is (4, 1). the drele. Subtracting 4 from the y- coordinate
of the center of the cirde gives the point (2, -5)
Detractor Explanations; Choice A is incorrect which would be the minimum y value. Since
and may result from swapping the values of Choice D gives a y-coordinate of -6, this point
h and ft . Choice B is incorrect and may result is outside of the circle and cannot be point N

.
mrW vibrdntpublisbefS-tOftl 189

Telegram: SAT_pdf
I GEOMETHY AND TRIGONOMETRY

since point N is on the circle. All of the other


I
= - . fhcn, find the area of the full circle
answer choices have a -coordinate within 4 of 360 5
^
the center of the circle and therefore cannot he which is A = nfS)1 = 25fl.
outside of the circle.
Multiplying the fraction of the sector to the area
Uistractor Explanations: Choices A , U and C
are incorrect and reflect errors in understanding
of the whole circle yields 25 x x —1 = 5 JT = 15.7 cm2.

the equation of a circle.


296.Level: Easy|Skill/ Know Ledge; Circles|Testing
Point: Using properties of circles to solve area
294 . Level: Medium | Skill / Knowkdgc: Circles problems
Testing Point : Use circle proportionality
theorems to fold central angles Key Explanation : Choice D is correct. Tf the
diameter of a circle is 2x1 then the radius is
Key Explanation ; The correct answer is 90. Start IV
hy finding the degrees of the major and minor —
: =*
\
arts. Circle proportionality theorem states
Therefore, the area of the circle is ( ) = SJC"
JT JC* ,
that Lhe proportion Lhal an are length is to the
circumference of a circle is the same proportion Distractor Explanations: Choices A , B , and
as the central angle Lhal is subtended by the arc. C are incorrect and are the result of errors in
Thus : calculating the area of a circle with algebraic
The angle subtended by minor arc fiC
_—x 3 expression s.

360° = 135°.
Since minor arc BC and major arc BC comprise 297.Level! Easy|Skill / Knowledge: Circles|Testing
Point: Using circle theorems
the whole circle, the degree measure of major
arc BC can be found by subtracting the degree
Key Explanation: Choice D is correct, OB and
measure of minor arc BC from 360° as fallows:
OC arc the radii of the circle and arc equal. This
The angle subtended by major arc BC = 360 - makes angles OBC and GC0 equal as they arc
135 = 225°, base angles. The value of angle BOC is therefore
Therefore, the difference in the angles is 225 - ldO" - 351> - 35° = 11(F,
135 = 90°. The arc length of the sector can be found hy
a
C where C is th e ci rcu m fe re nee of the ci rele
3 hi I
295. Level: Easy|Skill/ Knowledge: Circles|Testing and Sis the subtended angle. Substituting ]1<J“
Point: binding the area of a sector of a circle to 6 and equating the formula to the given arc

Key Explanation ; The correct answer is 15.7cm2


length yields - — C = 1 lir. Multiplying 360 and
dividing 110 from both sides of the equation
First, use the degree measure of the interior angle
yields C = 36ir.
to determine the fraction relationship between
the sector and the full circle which yields

Digital SAT Math Practice Questions 190

Telegram: SAT_pdf
I Dinstructor Explanations: Choice A is incorrect
Circles ( Answers )

299. Level: Medium | Skill / Knowledge: Circles


I
and may result from solving half of the Testing Point: Working with similar figures
circumference. Choice B is incorrect and may
result from solving the area of the circle. Choice Key Explanation: Choice C is correct . The ratio
C is incorrect. These options may be a result of a of the circumferences is 2k : 36k which is 1:18 .
conceptual and calculation error. The ratio of their areas would be the square of
the ratio of their circumference. The ratio would

29ft. Level : Medium | Skill / Knowledge : Circles


-
therefore he l hlft^ or 1:324.

Testing point s Determining the measure of an Distracloi Explanations: Choice A and Choice
external angle to a circle D arc incorrect. These options may result from
calculation or conceptual errors . Choice J1 is
Key Explanation: Choice D is correct. If Ft?
incorrect. This is the ratio of the circumference.
is tangent to the circle, the angle QPQ is right-
angled. The are length of the sector can he found
by ?
T
369
JT D where D is the diameter of ihe circle.
300. Level : Easy | Skill / Krnwledge : Circles | Testing
Point : Using the equation of a circle
Since the OP - 12 cm , the radius is 12 cm. This
- cm. Substituting
makes the diameLcT equal Lo 21
Key Explanation; The standard equation of a
ihe diameleT and equating ihe formula Lo
circle is given by the equation [ x - h )1 + ( y - k )z
the given arc length yields ±
360
-( x )( 24 ) A
3^ = r1, where r is the radius of the circle. Since P =
64, then r - 8 - The diameter of a circle is twice its
Multiplying both sides of Lhe equation by
^5
:i ,c
yields 3- 25v. Angles inside a triangle add up lo
.
radius. Therefore, the diameter nf the above circle
will be 16.
180". Since two of the angles are 90° and 25°. The
thiTd angle ( PQOl is 65°,

Distraclor Explanations: Choice A is incorrect.


This is the value of POQ. Choices B and C
.
are incorrect These options may be a result of
conceptual or calculation errors.

.
wwW vibriintpublisbefS .tOftl 191

Telegram: SAT_pdf
This page is intentionally left blank

Telegram: SAT_pdf
Telegram: SAT_pdf
Module
1 1 l
Math
2 2 QUESTIONS | 3 5 MINUTES

DIRECTIONS

The questions in this section address a number of important math skills. Use of a calculator is permit -
ted for all questions.

- ffr - -
Unless otherwise indicated: All variables and expressions represent real numbers, Figures provided

numbers x for which ) is a real number.


-
are drawn to scale, All figures lie in a plane, The domain of a given function is the set of all real

REFERENCE

* 1'
*

A= itr 2 A= f.w Special Right Triangles


C = 2xr

l
V = lwh V = xr1h V=

The number of degrees of arc in a circle is 360-


Thenumberof radians of arc ina circle is 2 n.
The sum of the measures in degrees of the angles of a triangle is ISO.

Digital SAT Math Practice Questions 194

Telegram: SAT_pdf
n Module
1 1 l
For multiple - choice questions, solve each problem, choose the correct
answer from the choices provided, and then circle your answer in this
book. Circle only one answer for each question . If you change your mind,
completely erase the circle. You will not get credit for questions with more
than one answer circled, or for questions with no answers circled.

For student-produced response questions, solve each problem and write


your answer next to or under the questFon in the test book as described
below.
Once you 've written your answer, circle it clearly. You will not receive
credit for anything written outside the circle, or for any questions with
more than one circled answer.
If you find more than one correct answer, write and circle only one
answer.
Your answer can be up to 5 characters for a positive answer and up to
6 characters (including the negative sign) fora negative answer, but no
more.
If your answer is afraction that istoolong (over 5 characters for positive, 6
characters for negative}, write the decimal equivalent.
If your answer is a decimal that is too long [overS characters for positive,
6 characters for negative), truncate it or round at the fourth digit.
If your answer is a mixed number (such as 3J. 2 ), write it as an improper
fraction (7 2) or its decimal equivalent (3,5 ),
^
Don't include symbols such as a percent sign, comma, or dollar sign in
your circled answer.

vibrdntpublisbefS-tOftl
itmt! 195

Telegram: SAT_pdf
Module
1 1 l
If /U ) = -4 and /(5) = 12, whal is the value of the 3y - 2x = 1
y -interccpl of Lhc function

A) 4 Using the system of equations, whal is the value


13) -8
ofSx - y?

C) 2 A) 4
D) 8 B ) 17
C) 20
D) - 17

lhe average score in a high schools French


entrance exam was 65%. Three new students
appeared lor the entrance exam and scored 25%,
45%, and 70%; the new average score fell iu 60%. The function c( x] represents the total cost of a
How many students had taken the entrance exam company, lhc function represents the variable
before these three students? costs and luted costs of the company £r = s3 -.
750x + 90,000, when; x represents the number of
A ) 11 units produced by the company. How many units
R) 7 wuuld the company have to produce to incur a
minimum total cost ?
C) 8
D) 4 A ) 375
B ) 750
C) 150
D) 600

If sifl ( 4* + 6) = cost* + 4), what is the value of x?


-3 + ' fit
is a solution to the equation -2x* + 3JT
-A
+ 6 = 0, what is the value of n ?

Digital SAT Math Practice Questions 196

Telegram: SAT_pdf
n Module
1 1 l
R
What is the value of -n radians in degrees? lhe function *) = 3x - 4 is moved Lo Lhe Left by
^
4 units and up 2 units. What is Lhe value of its
A ) 960“ y-intercept ?
li ) 430“
C) 240“
19) 120“

.
If r and q are solutions tu 3JI4 + 6X - 21 = 0 what
is the value of r + q?

A) -7
\2x - 5 <3
li ) 2
llow many integer solutions does the system
above haw? C) -2

D) 7
A ) One solution
B ) Two solutions
C) No solution
D } Inin n itely many sol utions

Which of the following is equivalent Lo 7 iv ( p;c +


y&

A ) 7wpx + 7 wyz
R ) 7wpxyz
3y + 5r = 15
C) 7wpx + yz
What is the jr - intencept of the equation of the line D) 7pjc + wyz
shown above?

A) 5
B) -5

C) 3

D) -3

.wtirentpijWisherS fOrt}
wwW . 197

Telegram: SAT_pdf
Module
1 1 l
A cylinder with an open top has a radius of Hrrtf A study was conducted on students in a
and a height of lOcm . What is the value of llie siunmer class in city IV. 'Hie study showed dial
surface area of the cylinder ? 4 in 5 students in the summer class showed an
improvement in scores in their opening exam.
A ) 196071 Which of the following could be concluded from
the survey?
R ) 476TT
C) 67 In A ) All the students in city tV will improve their
entrance scores if they attend the summer
D) I 40ftr
class,
If ) The students who do noL pass the entrance
exam should attend the summer class to
improve their scores
C) Ihe summer class will improve the scores of
3 of the Students who attend the class,
3JC - + 11
If the expression it equivalent to
*- 4 D) Thereisnt a cause-effect relationship
3r + 6 + —
- , what is the value of lit ?
x—4
between the summer class and the entrance
exam .

A circle has a radius of bon , Tf a sector in the


circle has an area of 7.5, what is the value of ihc
angle thaL subtends the sector at ihe center in
radians?

A) —24 JT

5
B) - x
6
5
C) -
4*

D) — at
17

Digital SAT Math Practice Questians 198

Telegram: SAT_pdf
n Module
1 1 l
What is the solution of the system below 3(JC - 5) There are 54 animals (cither dogs, cals, or pigs)
= 2(1 - 2)? on a certain Saturday in a park. If there arc thrice
as many dogs as there are cats and half as many
pigs as there arc cats on that day. How many dogs
were in the park?

A) 6
13) 12
If line t (shown below ) and line m ( not shown ) C) IS
have no SuluLion, which of the fulluwiiig could be
D ) 3ft
the equation oflinc m1.

For how many acLual points dues the line of btsL


—- 3 fit predict a greater value than the actual value?

—2

A ) Ay - 10* = 6

B ) 10y + 4* = 6
C) 10* + 4y = 6 o
V n
D) lOy - 4* = 6 \
\

wwwt vibrdntpublisbefS- tOftl 199

Telegram: SAT_pdf
Module
1 1 l
3y - JC =3
, 4 2
7
3 3
Whal is iht value of * for the given syslcm of
equations?

*> !
B) 2
C) -3

T» 1

Tffl 2 jt + 3 ) - - j? + 4x + 6, what is the value of


jiW

A) 1
R ) 11
C) 9
D) 6

Digital SAT Math Practice Questions 200

Telegram: SAT_pdf
No Test Material Om This Page

Telegram: SAT_pdf
Module
2 1 l
Math
2 2 QUESTIONS | 3 5 MINUTES

DIRECTIONS

The q ue stion s in this sectio n a dd ress a number of i mporta nt math s kiIIs. Use of a calculator is permit-
ted far all questions.

- - -
Unless otherwise indicated:’ All variables and expressions rep resent real numbers, Figures provided
are drawn to scale. All figures lie in a plane. The domain of a given function is the set of all real
numbers x for which ffr) is a real number.

REFERENCE

2 A= f.w
A= Jtr Special Right Triangles
C = 2KT

I
V = £ wh Vr = JTT1/t V=

The number of degrees of arc in a circle is 36C,


The number of radians of arc in a circle is 2 n.
The sum of the measures in degrees of the angles of a triangle is lSO,

Digital SAT Math Practice Questions 202

Telegram: SAT_pdf
n Module
2 1 l
For multiple- choke questions, solve each problem, choose the correct
answer from the choices provided, and then circle your answer in this
.
book , Circle only one answer for each question If you change your mind,
.
completely erase the circle You will not get credit for questions with more
tha n one answer circled, or fo r q u estio n s with no answers circled.

For student- produced response questions, solve each problem and write
your answer next to or under the question in the test book as described
below.
Once you've written your answer, circle it clearly. You will not receive
credit for a nyth ing written outside the circ le, o r for a ny question s with
more than one circled answer,
If you find more than one correct answer, write and circle only one
answer.
Your answer can be up to 5 characters for a positive answer and up to
6 characters [including the negative sign) for a negative answer, but no
more.
If your answer is a fraction that is too long (over 5 characters for positive, 6
characters for negative), write the decimal equivalent.
If your answer is a decimal that is too long (over 5 characters for positive,
6 characters for negative), truncate it or round at the fourth digit
If your answer is a mixed number (such as 3.!. 2 ), write it as a n im proper
fraction (7/ 2) or its decimal equivalent ( 3.5).
Don't inclu de symbols s uch as a percent si gn, comma, o r dolla r si gn in
your circled answer.

.
wwvt vibrdntpublisbefS- tOftl 203

Telegram: SAT_pdf
Module
2 1 l
p(r) = 60,040(1.012)“ 4* - 2y = 10

The function p(i) models the population of city 3y - 2JC - 3


T from the year 2004 lo the year 2016. Which of
the following statements best represents ( 1.012)*? Using lhe systems of equation, what is the value
of r?
A} The population of city Y increases by 12%
every year from 2004 to 201 ft.
13) The population ofeiLy Y increases y L .2%
every year from 2004 to 2016.
C) The population of city Y decreases by 12%
every year from 2004 to 2016.
Triangle A BC is shown helow. If AB - 36cm and
D) The population ofeiLy Y decreases by 1.2% AC - 27on, whaL is Lhe value of DC?
every year from 2004 io 2016.

H 13 C
The number of people who own houses
compared Lo those who rent houses in city L is A ) 28.30
represented by the ratio 2:3, If the people in city B) 21.60
Kt with a population of 623,0(10 are twice as likely
.
Lo own a house compared to those in city L then
C) 19.29

approximately how many people rent houses in D) 25.71


city K7

A ) 124,600
B) 249,200
C) 373.800

D) 493,400

Digital SAT Math Practice Questions 204

Telegram: SAT_pdf
n Module
2 1 l
y= 2 - k 3y > 2x + 5

y - 3JC3 - 6* + 1 y x+3

If IHJC system of the equation above has two Which of the Mowing is a solution to the
distinct solutions, which of the following cannot inequality below?
he the value of A ?
A ) (3, 2 )
A) 1
B ) ( - 2, 1 )
B) 2
Q ( 2, 6}
C) 3
D > ( -1, 4}
D) 4

Which of the following expressions >s equivalent


x + 6x + yz - Sy = 24
1
to 2x - ft = 4x + 6?
What is the value of the x- coordinate of the
A ) - JC - 12 = 0
tenter of Lhe dintle above? '

B) 2 x = 6
A ) -3
C ) 3 x - 18- = 0
B) 3
D) 3JC + 18 = 0
C) 4

D) -A

What is the value of 32% of 500?

The length of a rectangular field is 7 meters


more than its width. If the area of the field is 120
square meters. What is the value of its length?

wwW, vibrdntpublisbefS- tOftl 205

Telegram: SAT_pdf
Module
2 1 l
5
+
_ 3_ If Lhe system of equations has no solution, what
2x - l 2x is lhe value of fc?
Which of the following expressions is equivalent
Lo lhe expression above?
(JC + 4)1 = JC + 3JC + k( x + 3)
1

161 - 3
A)
4 x1 - 2x
iOjt - 5
R)
4JC
3
-2X
7
Q
2x - l Which of the following statements best describes

D)
&
4x - l
2 x* + 4x + 6> ^=
lhe Lranslalionbelweeii ) 2(x + 5)1 log( jt) -

^
A) lhe graph x ) is translated 4 unils to Lhe
right and 4 units up
_
B) The graph /{*) is translated 1 unit to the right
and 6 units up

X M
^
C) lhe graph x ) is translated 4 unils to the
right and 6 units up

1
3
-2

4
and 4 units up ^
D) The graph x ) is translated 4 units to the left

5 10

What is the equation of the function *) as


shown above ? ^
A)
B)
^jx) = 3x - 5
{ x ) = -2x If
3 + 4i

2 i
= a + bit what is the value of

C) )Tx) = x + l

D ) f [ x ) = 2x

» 7

°I
D) 4.
T

Digital SAT Math Practice Questions 206

Telegram: SAT_pdf
n Module
2 1 l
|3JC - 5|= 7 Which of the following system of inequalities
represents the graph below?
What is the positive solution to the equation
below? 7
i
6 «s:

A mattress and bed shop tabulates the number of


orders it receives in a particular week.
'
1'
!
l
41
!_
Mattress No mattress 2 4 Ci a to
Bed 30 -15 -I
No Bed 22 IK
-2
WhaL is the probability thaL a customer buys a
mattress given that they do not buy a bed ? A ) / k -0- 4x + 4
y > 2x + 3
11
A) U) ySO.l t + 'l
2i )
y < 2x + 3
22
«) C) / -0-4* + 4
113
y < 2x + 3
4tl
C) D) y 0.4jf + 4
115
11
ix + 3
D)
26

sfT
If a = 2 b and A= = n \ what is the value offr ?
$2b

vibrdntpublisbefS-tOftl
itmt! 207

Telegram: SAT_pdf
Module
2 1 l
Marla traveled at 60 miles per hour for the first David ean spend at most $36 in a bookstore. A
two hours of the trip. She then traveled at a speed book costs $ 5 and a pen costs $1.50. If he buys
of SO miles per hour for the rest of the trip. The at least one book and three pens, what is the
average speed of the trip is 54 miles per hour, majuitium number of books that he can get ?

llow long was the whole trip In hours?


A) 4
A) 2
B) 5
13) 3
C) 6
C) 4
D) 7
D) 5

llow many solutions does the equation of the


graph below have with the line y - 5?

A} 0

R) i

C) 2
D) Infinitely many solutions

Digital SAT Math Practice Questions 208

Telegram: SAT_pdf
No Test Material Om This Page

Telegram: SAT_pdf
Answer Key
Math
Module 1 Module 2
-
£
i
!
- 1a _
g 9
ti i S E
I
O I3 3
C'
E
* sS
3
'
I a l 13
2 C 2 A
3 16 3 3
4 R 4 A
5 A 5 D
6 57 6 A
7 R 7 15
3 13 3 13
9 C 9 D
10 - 10 10 160
11 C 11 A
12 A 12 A
13 13 13 5
14 35 14 A
15 C 15 13
16 D 16 4
17 11 17 A
IK R IS C
19 D 7
19
20 4 15
21 B 20 D
22 C
R
21
22 C

Digital SAT Math Practice Questions 210

Telegram: SAT_pdf
I MATH Answers & Explanations Module 1
I
1. Level; Easy | Domain; ALGEBRA Since 3 students were added, there are x + 3
Skills Knowledge: Linear functions | Testing students now.
point: Solving for the y-intercept of the linear Adding the scores of the new students yields a
function new total of T + 25 + 45 + 70 or T + 140.
Since the new average score is 60, then
Key Explanation: Choice D is correct. The T + 140
= 60. Multiplying both sides of the
y-intercept is the value ofy when * is 0. First, *+ 3
find the equation of the line in slope- intercept equation by * + 3 yields T + 140 = 60(JC + 3) or T
form ofy = mx + b where m is the slope and b is + 140 = 60* + ISO.
thcy-interccpt. Given the points (1, -4) and (5, Since T = 6SJC, then 65ut + 140 = 60* + 180-
Subtracting 60* and 140 from both sides of the
12 ), the slope of the line is — - ^ =4 .
equation yields 5* = 40.
Determining the equation of the line using ( y - Dividing both sides of the equation by 5 yields x
y, ) = m ( x - *, ) yields (y -12) = 4 ( x - 5 ) . = 8.
Using distributive properly on the right side of Therefore, there are 8 students initially.
the equation yields y - 12 = 4x - 20.
Adding 12 to both sides of the equation yields D is tractor Explanations: Choke A is ineorrecl.
y - 4x - 8. This is the total number of students who took the
The equation — Ax - 8. The y- intercept is entrance exam, including the three new students.
therefore -ft. Choices B and D are incorrect . These options
may be due to miscalculations or conceptual
DistraclorExplanations: Choke A is incorrect errors.
'
Ihis is Lhc slope of the function J( jt). Choke C is
incorrect. 'Ihis is the jc-inierccpl of the function
3. Level: Easy |Domain: GEOMETRY AND
Jfx). Choke D is incorrect Ihis is the negative
TRIGONOMETRY
value of the option B.
Skill / Knowledge : Right angles and trigonometry
Testing point; Trigonometric Identity
2. Level: Medium|Domain: PROBLEM- SOLVING
AND DATA ANALYTICS Key Explanation: The trigonometric identity,
Skill / Knowledge: One- variable data: siu ( jt) = cos(90 - x ) , indicates dial the angles are
distributions and measures of center and spread complementary. This means that the sum of 4JC +
Testing point: Mean of data set 6 and x + 4 is equal to 90. Therefore, 4* + 6 + x +
4 = 90,
Key Explanation; Choice C is correct. Let x be Combining like terms yields 5* + 10 = 90.
the initial number of students and T be the total Subtracting Id from both sides of the equation
score of * number of students. yields 5x = 80 .
Since the average score of * students is 65, then Dividing both sides of the equation by 5 yields *
T
— = 65. Multiplying both sides of the equation = 16.
by x yields T = 65x,

.
wwW vibrdntpublisbefS- tOftl 211

Telegram: SAT_pdf
I MATH Answers & Explanations Module 1
I
4 Level: Medium | Domain: ALGEBRA Key Explanation: To solve for this quadratic
Skill Knowledge; Systems of two linear equation, use the quadratic formula
equations in two variables | Testing point :
x= —b ± Vf — 4ac
>
3
where a = -2, b = 3 and
Solving for linear systems simultaneously 2a
c = ft.
Key Explanation : Choice B is correct . To find Substituting the values yields
the value of 5x - y, we can subtract the 1st j32 - 4 (-2)(<i) V
equation from the 2nd equation. This yields 2 y +
-3 ±
2(-2 j

4
^ . therefore the

-
3x - 3y + 2r = 18 1. value of n = 57 .
Combining like terms yields 5 x - y = 1 7 .
1he refore, Lhe answer is 17,
'
7, Level: Easy | Domain: GEOMETRY AND
Distractor Explanations: Choices A, C, and D TRIGONOMETRY
are incorrect and may result from a conceptual Skill / Knowledge: Lines, angles, and triangles
or calculation error. Testing point: Converting radians to degrees

Key Explanation: Choice B is eurrecL TT radian


5, Level: Easy|Duraain: ADVANCED MATH is equal to 130". Ihurefore, Lu convert radians Lo
Skill/ Knowledge: Nonlinear functions | Testing degrees, subsLiLuLc n with 1&0" which yields
point: Determine the vertex of a quadratic A
* i 8(r = 43tr.
function 3
Dislraetor Explanations; Choices A, C, and
Key Explanation: Chuice A is correct, 'lhe
D are incorrect - These options may be due to
number of units that would represent the
calculation or conceptual errors.
minimum total cost is the x-coordinale of the
vertex of the given function, lhe x-eoordinale
of the vertex can be solved using the formula 3. Level: Medium|Domain: Advanced Math
b —a
2
where a and b are the coefficients of jf 1 and
Skill / Knowledgc: Nonlinear equations in
one variable and systems of equations in two
x, respectively. Substituting the values of a and b
variables | Testing point; Solving for absolute
1 = 375 units. values

Dis tractor Explanations: Choices B , C, and Key Explanation: Choice B is correct . To solve
D are incorrect. They may be inferred due to for this inequality, the content of the absolute
conceptual or calculation errors. value must be multiplied by 1 and -1 which
yields:
*|2x - 5|< "3 \2x - 5| < 3
*
6. level: Medium |Domain: ADVANCED MATH
Skill/ Knowledge: Nonlinear equations in 2JC - 5 < 3 -2x + 5 < 3
lx < 8 -2x ; -2
one variable and systems of equations in two
variables | Testing point: Solving for quadratic X< 4 x>1
equations using the quadratic formula

Digital SAT Math Practice QuCStidns 212

Telegram: SAT_pdf
I MATH Answers & Explanations Module 1
I
The integer solutions that fit the inequality 11. Level: Easy |Domain: ADVANCED MATH
1 < x < 4 are 2 and 3. Therefore, there are two SkiJJ / Knowlcdge: Nonlinear equations in
solutions. one variable and systems of equations in two
variables| resting point: Sum of the solutions /
Detractor Explanations: Choices A , C, and roots
D are incorrect. These options may be due to
calculation or conceptual errors. Key Explanation ; Choice C is correct . In a
quadratic equation , the sum of the solutions /
9. Level: Easy | Domain: ALGEBRA roots is given by
-b
—a where a and it arc the
Skill / Knowledge: I .inear equations with two euclheienls ofjd and jt, respectively. Referring to
variables | Testing point: Finding the x- irtercept ihc given equation, the values of o and it arc
of the line a = 3 anil b = 6 . lherefore, Lhc sum of the

Key Explanation: ChoiceC is correct The


solutions will be —36 — —2
-
.

jc -inLereepl of a line is found when the value of y DistractoT Explanations: Choice A is incorrect.
is equal to 0. Substituting 0 toy yields 3(0) + Sue - This option represents the value of the product of
15 or 5JC = 15. the solutions. Choices B and D are incorrect and
Dividing both sides of the equation by 5 yields may be due to conceptual or calculation errors.
jt
= 3.
Distractor Explanations: Choice A is incorrect. 12. Levelj Easy|Domai n : ADVANCED M ATH
This is Lhc value uf thcy-inLcncepl of the line. Skill / Knowledge: Equivalent expressions

Choices B and D arc incorrect may be due io Testing point: Distributive property of
conceptual or calculation errors. multiplication

Key Explanation: Choice A is correct. Using the


10, Level; Medium | Domain; ALGEBRA distributive property yields 7 w{ px + yz ) = 7 wpx
Shill / Knowledge: T .inear functions | Testing + 7wyz. This is option A.
point: Linear translation
Distractor Explanations: Choices B , C, and
Key Explanation: To move the function f( x ) to D are incorrect and may be due to conceptual
the left by 4 units and 2 units up, add 4 to x and errors.
add 2 to the constant which yields fix ) = 3( x +
4) - 4 + 2,
13, Level ; Easy | Domain ; GEOMETRY AND
Using distributive propertyyields /fx ) = 3* + 12
TRIGONOMETRY
- 2.
_ Skill / Knowledge; Area and Volume | Testing
Simplifying the equation yields flx} = 3x - 10,
They- intercept is, therefore, 10. point; Surface area of an open cylinder

Key Explanation: Choice B is correct. The lateral


area of a cylinder can be found by finding the

vibrdntpublisbefS- tOftl 213

Telegram: SAT_pdf
I MATH Answers & Explanations Module 1
I
circumference of the base circle and multiplying the summer class. Choice B is incorrect. It is
it by the height of the cylinder which yields ( n:D not guaranteed that student scores will improve
x height of ihe cylinder ) = *( 28) x 10 = 280*. after attending the summer class. Choice D is
Since the cylinder has an open top, add the area incorrect . There is a cause-effect relationship
of the circle at the bottom of the cylinder to find between students who attend the summer class
its surface area . 'Ihe area of the base circle is nr1 = and improvement in their entrance exam scores.
nf 14)1 = 19(5*. The total surface area of this open -
tnp cylinder would be 19(5* + 280* = 47(5*.
16 . Level: Hard | Domain: GEOMETRY AND
TRIGONOMETRY
Distraclor Explana Lions: Choice A is incorrect.
Skill ,''Knowledge: Lines, Angles and Triangles
This option gives the volume of the cylinder
Testing point : Sector of a circle
Choice C is incorrect. This option would give Lhc
surface area of the closed cylinder. Choice D is
Key Explanation : Choice D is correct. To find
inCOrreCL. This option may be due Lo calculation
the proportion the sector of the circle, first ,
or conceptual errors.
find the area of the circle which is 36*. The
7.5*
proportion would be - — Therefore, Lhc
.
36* 24
14. Level: Easy | Domain: ADVANCED MATll
angle that would he subtended at the center will
Skill/ Know ledge: Equivalent Expressions
5
Testing puinl: Remainder Theorem be — x 360 = 75" Co]iverting 75a to radians
.

75 _—5
vieids * 12 *
Key Explanation : Tire variable m is the -
180
remainder when - 6x + 11 is divided by JC - 4.
Distractor Explanations: Choice A is incorrect
Tt can be found by substituting 4, which is the
because this angle degrees will be 37,5".
in
value of * when the divisor is equated to 0( ,v - 4
Choice E is incorrect because this angle in
— 0), into the dividend.
This yields 3(4) i - 6( 4) + 11 = 35. degrees will be 150° .

Choiee C is incorrect because this angle in


degrees will bo 135°.
15. Level: Easy | Domain: PROBLEM -SOLVING
AND DATA ANALYSIS
17 , Level: Easy | Domain; ALGEBRA
Skill / Knowledge; Evaluating statistical claims:
Skill / Knowledge: Linear equations in one
observational studies and experiments | Testing
variable | Testing point: System of equations
point: Statistical claims

Key Explanation: Using the distributive proper -


Key Explanation: Choice C is correct. This
ty, you get Tt - 15 = lx - 4,
option is true as 4 out of 5 students who attended
Subtracting 2x and adding 15 to both sides of the
the summer class performed better in the
equation yields x = 11 ,
opening exam. This is equivalent to 80% ,
Therefore, the value of x is 11 .
Detractor Explanations: Choice A is incorrect.
Not all students improve their scores after taking

Digital SAT Math Practice Questions 214

Telegram: SAT_pdf
I MATH Answers & Explanations Module 1
I
IS. Level; Medium | Domain: ALGEBRA park that day. Choice B is incorrect, 'ihis is the
Skills Knowledge: Linear equations in two number of cats that were in the park that day.
variables | Testing point: Parallel lines Choice C is incorrect . This is the total number of
cals and pigs in the park that day.
Key Explanation: Choice E is correct. For a
system of equation to have no solution , the
20. Level : Easy|Domain : PROBLEM- SOLVING
two lines must he parallel Using the points
AND DAT A ANALYSIS
( 0, 4) and ( 10, Oh the slope of the given line
Skill/ Knowlcdgc: T wo- variable data: models
is ——— —— -
0-10 -10
= 0.4. Line m would also, and scatlerplots|Testing point; Solving for
therefore, have a slope of -0.4 . Only lOy 4 4x - absolute values
6 has a slope of -0.4 and would therefore be Lhc
Key Explanation! Count the points where the
correct answer.
actual value is lower than the value predicted by
Detractor Explanations: Choice A is incorrect. the line of best fit. These are the points below the
lire. Therefore, there are four points.
this line has a slope of — , the system would
be perpendicular and would have one solution.
Choice'S C and D arc incorrect and may result 21. Level : Medium | Domain: ALGEBRA
from a conceptual or calculation error. Skill / Knowledge: Linear equations in two
variables|Testing point: Solving for linear
systems simultaneously
19. Level: Medium | Domain: ALGEBRA
Skill / Knuwlcdge: Linear equations in two Key Explanation: Choice B is correct. Solve
variables | Testing point: Linear Word problems the equation simultaneously. Multiply the first
equation by 2 to gel 6y - lx = 6 and the second
Key Explanation: Choice D is correct. Given
equation by 3 to get Ay - 4* = 2,
that c represents the number of cats, the number
Subtract the second equation from the first
of dogs d would be d = 3c, The number of pigs p
equation which yields 2x = 4. Dividing both
would be

+
1
-f
2
= 54 .
^
p = c. Therefore, the total number of
animals is represented by c + p + d = 54 or c + 3c
sides of the equation by 2 yieldsx = 2,

Distruttur Explanations: Choice A is incorrect.


This option is the value of y. Choices C and D
Simplifying the equation yields —92 = 54 .
are incorrect and may be due to conceptual or
2 calculation errors.
Multiplying - to both sides of the equation
yields c = 12,
Hence, there were 12 cats. 22, Level: Hard |Domain: ADVANCED MATH
Therefore, the number of dogs is d = (12 X 3)= Skill/ Knuwledge: Nonlinear equations in
36, one variable and systems of equations in two
variables|Testing point: Evaluating quadratic
D istract nr Explanations: Choice A is incorrect. equation
This is the number of pigs that were in the

.
wwW vibrdntpublisbefS-tOftl 215

Telegram: SAT_pdf
I MATH Answers & Explanations Module 1
I
Key Explanation: Choice C is correct. To solve
for J 5) solve for x by equating 2 x + 3 to 5. This
^
yields x
T

2 + 3 = 5.
Subtracting 3 from both sides of the equation
yields 2x = 2 .
Dividing both sides of the equation by 2 yields
x = 1.
Substituting t for x in the given equation yields

^
ft.2( 1 } + 3 ) = - ( l + 4( l ) + (i = 9.
Therelbre., 5) = 9-
^
Distractor Explanations: Choice A is incorrect ,
litis would be the value of the function if you
'

substitute 5 directly into x . Choices B and D


arc incurrect. Ihesc options may be due to
conceptual or calculation errors.

Digital SAT Math Practice Questions 216

Telegram: SAT_pdf
I MATH Answers & Explanations Module 2
I
1. Level; Easy | Domain; ADVANCED MATH miscalculations and conceptual errors. Choice D
Skills Knowledge: Nonlinear functions|Testing is incorrect. This option represents the number
point: Interpreting exponential functions of people who own houses in city K .

Key Explanation: Choice E is correct.


3. Level! Easy|Domain! A LGEBRA
Exponential functions have a format of
Skill / Knowledge ; Systems of two linear
y = fl( 1 + r)' where a is the initial number,
equations in two variables | Testing point;
( 1 + r) is the growth factor, r is the rate of growth
Solving for linear systems simultaneously
and x is the time. Hence, ( 1.(112 )’ implies that the
population of the town is increasing by a growth
Key Explanation: To solve for xs solve for the
factor of 1.012. The growth rale is then 0.012 or
equations simultaneously. Re- arrange the second
1.2%. Option B is then correct.
-
equation to get - 2x - 3.
Multiply the second equation by 2 which yields
Distractnr Explanations: Choices A , C, andD
by - 4x = -6-
are incorrect. These options are false statements
Add the two equations to get 4y - 4.
and may he due to conceptual nr calculation
Dividing both sides of the equation by 4 yields
errors.
V -1 -
Substitute the y value into the second equuLiun
2. Level; Easy | Domain: PROR7 ,EM SOLVING yields 3(1) = 2x - 3.
AND DATA ANALYTICS Adding 3 Lo boLh sides of the equation yields
Skill / Knowledge: Ratios, rates, proportion al 6 - lx.
relationships, and units | Testing point; Using Dividing both sides of the equation by 2 yields
ratio in solving problems } = ror x = 3-

Key Explanation; Choice A is correct. The


proportion 4. Level: Medium | Domain: GEOMETRY AND
of people who own houses in city f . TRIGONOMETRY
percentage
2 Skill/ Knowledge: Right Triangles and
is represented hy — or 40% of the population. Trigonometry | Testing point: Proportions of a
Since people in city K are twice as likely to own
triangle
proportion
a house, the of people who own a
percentage
]
Key Explanation; Choice A is correct. First
house in city K is - or 80% of its population . find the value of JJC, which is the hypotenuse of

This would imply that only - of the population the triangle ABC. The length of the hypotenuse
5 can be found using the Pythagoras theorem
in city K rent houses. This would approximately
c= + , where a and b are 56 and 27 and
be — x 623,000 = 124,600 people in city K. c is BC, The value of efthe hypotenuse) will be c
= 45. Using proportions, the lengths of triangle
Distract or Explanations: Choices B and C ABC is directly proportional to the lengths of
are incorrect. These options would be due to

.
wwW ribriintpublisbefS-COftl 217

Telegram: SAT_pdf
I MATH Answers & Explanations Module 2
I
6. Level: Easy | Domain: GEOMETRY AND
triangle ABD. 'therefore, — =—
36 ED TRIGONOMETRY
ED = 28.80. Skill,'Knowledge: Circles | Testing point :
Equations of a circle
Distractor Explanations: Choice B is incorrect.
This option is the value of AD. Choices C and D Key Explanation : Choice A is correct. The
are incorrect and may be due to calculation or standard equation of a circle is given by [ x - h }1 +
conceptual errors. ( y - fc )s = r1 where [ /i, Jc ) is the center of the circle.
To convert our current equation to a standard
equation, complete Lhc perfect square trinomials
3. Level: Medium |Domain: ADVANCED MATH
Slrill / Knowfedge: Nonlinear equations in = 24.
in { )?- + fix) + (y1 - 8y)

one variable and systems of equations in two


Adding 9 and 16 lo both sides of the equation
variables | Testing point : Systems of Enear and yields (** + fix + 9) + (yd - 3y + 16) = 24 + 16 + 9,
quadratic equation Factoring the perfect Square Lrinomials yields
( x + 3)3 + (y - 4)3 = 49.
Key Explanation : Choice D is correct. To solve Since h - -3 and k = 4. then the center of the
tor the system, use substitution method which circle is ( -3, 4). The x-coordinate is therefore -3.

— — —
yields Z ic 3JC^ (Vrc -i- 1. Subtracting 1 and
Distractor Explanations: Choice B is incorrect
adding tto both sides of the equation would
and may residL from getting the value of
result in a quadratic equation: 0 = 3x3 - fix + k
-h . Choice C is incorrect, This option is the
- 1. For quadratic equations with two solutions,
the diseriminanL is greater than 0 (i?s - lac > 0) . y-eoordinale of the eenter of the circle. Choice D
is incorrect and may result horn a conceptual or
In the quadratic equal ion, u = 3, b = -6 and C =
calculation error.
.
Jt - 1, Therefore (-fi )3 - 4{3 ){ - 1) > 0 which
*
would translate to 36 - L 2fr + 12 > 0. Combining
the constants yields 43 - ilk > 0, Subtracting 43 7. Level: Medium|Domain: GEOMETRY AND
from both sides of the inequality yields TRIGONOMETRY
-12fc > -48 , Dividing both sides of the inequality Still,'Knowledge: Area and Volume | Testing
by -12 yields k < 4 Only option D is not less
r point: Area of a rectangle
than 4, Therefore, it is the correct answer.
Key Explanation : The area of a rectangular held
Distractor Explanations: Choices A, B , and C is given by length (fl x width ( w ) . If the width can
are incorrect. These options would result in two be given by w, then the length would then be tv
distinct solutions in the quadratic and would + 7, Therefore, the area would be w{ w + 7 ) = 120
therefore be incorrect. which would result to w3 + 7tr = 120.
Subtracting 120 from both sides of the equation
yields wd + 7 w -120 = 0.
Solve the quadratic equation to find the value of
w.
Splitting the middle term and grouping the

Digital SAT Math Practice Questions 218

Telegram: SAT_pdf
I MATH Answers & Explanations Module 2
I
binomials yields ( w2 + 15 w ) + (- 8w - 120) = 0. Subtracting Ax and 6 from both sides of the
Factoring out w and -8 yields w( w + 15) - 8( w + equation yields -2x - 12 = 0.
15) = 0. Dividing both sides of the equation by -2 yields
Factoring out (1^ + 15) yields ( tv + 15 )( w - 8) = G . JC + 6 = 0.
Solving the values of w yields tv = 8, - 15. Multiplying both sides of the equation by 3 yields
Since the width of a rectangle cannot be negative, 3JC + 18 = 0 which is Choice D.
then the value of the width is B. Therefore, 3x + lft = 0 is the correct answer.
Therefore, the length would be 8 + 7 = 15 .
Distractor Explanations: Choice A is incorrect.
Once solved the value of x - 20. which is nol
8. Level: Easy | Domain: ALGEBRA
equivalent Lo x = -6. Choke B is incorrect , Once
Skill/ Knowledge: Linear inequalities in one or
solved. Lhc value of x = 3. which is nol equivalent
two variables | Testing point : Solving for linear
Lo x = -6. Choke C is incorrect. Once solved, the
inequalities systems
value of X 6, which is nol equivalent LO X = -6 .
*

Key Explanation: Choice B is correct. To solve


(he system of inequality, subsLiLuLc Lhc solutions 10, Level: Easy | Domain: PROBLEM - SOLVING
into Lhc given inequalities and verify the AND DATA ANALYTICS
statements , Skill/ Knowledge; Percentages | Testing point:
For (-2. 1), Lhc QrsL inequality becomes Percentage of a value
3( 1 ) > 2( -2) + 5 which is equivalent lo 3 > 1 .
'
Ihis statement is true. Evaluating the second Key Explanation: To solve tor this, convert the
inequality yields, 1 -2 + 3 which is equivalent 32
percentage into a fraction that yields -— , Then
to 1 1 - This statement is also true. This makes
multiply the fraction by the given amount which
(-2, 1 ) a solution to the system of inequality'.
yields — *500 - 160.
(1 10
Distrac Lor Explanations: Choke A is incorrect ,
SubstiLuLing( 3 , 2 ) to Lhc 1 st inequality yields
11 , Level ; Medium | Domain; ADVANCED MATH
6 > 11 which is a false Statement. Choice C
Skill / Knowledge; Equivalent expressions
is incorrect. Substituting ( 2. 6 ) to the 2nd
Testing point; Adding rational expressions
inequality yields 6 5 which is a false statement.
Choke D is incorrect , Substituting ( - 1 , 4) to Key Explanation : Choice A is correct . To solve
the 2 nd inequality yields 4 2 which is a false this problem, add the two expressions by making
statement , their denominators the same.
5 ( 2JC ) + X2JC - 1)
This yields , Using distributive

9. Level: Easy | Domain ; ALGEBRA -


2 JE ( 2* 1)
SkiJJ / Knowledge: Linear equations with one ]0 jc + for - 3
property yields . Combining like
variable | Testing point: Solving for x 4 x : - 2x
16x - 3
terms yields „ Therefore, the answer is
Key Explanation: Choice D is correct . First , 4xJ -2 x
Choice A ,
solve for x in the equation 2 x - 5 = Ax + &.

mvW, vibrdntpublisbefS- tOftl 219

Telegram: SAT_pdf
I MATH Answers & Explanations Module 2
I
Detractor Explanations: Choices B, C, and the gradient on the right side of the equation are
D are incorrect. These options may be due to equivalent. Therefore k - 5.
calculation or conceptual errors.

14. Level: Hard|Domain: ADVANCED MATH


12. Level: Medium |Domain: ALGEBRA SkiLL / Knowledge: Nonlinear functions | Testing
Ski 11 / Knowledge : Linear functions|Testing point: Translation of hmetion
poinl : Finding the equation of Lhc function
Key Explanation: Choice A is correct. To
Key Explanation: Choice A is correct. To find compare quadratic functions, the functions
the equation of the function /( x), first, find the should be in Lhcir vertex form which is y - tf(x -
slope of the function. Using points [3, 4) and ( 5, Jt )2 + k where ( h , fc) is Lhc vertex. In ihe funcLion
- yy 10 - 4 6
yi yfr ) , the vertex is ( - 5, 0 ). To find the vertex of
10), the slope is --- 3. The
,
xz - x 5- 3 2 g( x ) , convert g( x ) to its vertex form.
equation of the line can Lhcn be found using the Factoring ouL 2 yields g( x) - 2 (1* + 2 x + 3) .
formula ( y - y3) = mfx - jq ) which would result Completing Lhc perfect Square trinomial by
in O' - 10 ) - 3(x - 5). -
grouping yields gfx ) 2 [ (id + 2r + 1 ) + 2 ].
Using distributive properly yields y - 10 = 3x - Factoring the perfect square trinomial yields g( x}
15. - 2[{J: + L )1 + 2] .
DisLribuLing 2 to Lhc conslanL yields g x ) - 2( x +
Adding 10 lu both sides of the equation yields y
- 3x - 5, 1): + 4.
^
Therefore, the answer is x ) - 3* - 5- FJence, the vertex of g( jr ) is (-1, 4). Since the x
^ coordinate translates from -5 to -1, the funcLion
moves 4 units to the right. Since the y coordinate
Dis tractor Explanations: Choices B, C, and D
are incurred. This may be due to calculation or translates from 0 to 4, the function moves 4 units
conceptual error. up.

Distraetor Explanations: Choices B, C, and


13. Level: Easy| Domain: ALGEBRA D are incurreel. These options may be due to
Skill / Knowledge: linear equations in one conceptual or correction errors.
variable | Testing point : System of equations

15, l vel; Easy | Domain: ADVANCED MATH


Key Explanation: Using distributive properly ^
yields x2 + 8x + 16 = x2 + 3x + kx + 3fc. Grouping SkiLIfKnowledge; Equivalent Expressions
like terms together and subtracting x3 from both Testing point: Complex Numbers
sides of the equation yields for + 16 = 3x + fcx +
Key Explanation: Choice B is correct . To solve
3ft.
Subtracting 3x from both sides of the equation this expression, rationalize the expressions by
yields 5x + 16 = fcx + multiplying the numerator and denominator by
4
For a system of equations to have no solution , 2+ i , This yields 3 + 4i 2 + i 6 + fif + 3i + 4i
the gradient on the left side of the equation and
- _ 2+111
64 lli 4

2 i 2+i 21 T—
Splicing terms yields
4 1 — 5

Digital SAT Math Practice Questions 220

Telegram: SAT_pdf
I MATH Answers & Explanations Module 2
I
-+
5 —
5
f. Hence, a = - and & =

—3 Favourable outcomes '
Total outcomes
Iherefore, the total

Detractor Explanations: Choice A is number of outcome is equal to the sum of orders


incorrect. This is the value of a.Choices C and that do nol include beds. Hence, 22 + 18 - 40 .
D are incorrect. These options may be due to The favorable outcomes would be the orders that
conceptual or calculation errors. include a mattress without a bed which is 22. The

16. Level: Easy | Domain! ADVANCED MATH


Skill / Knowledge: Nonlinear equations in
one variable and systems of equations in two
probability would be equivalent to

Distractor Explanations: Choices E, C, and


D arcincorrccLand maybe due tu calculation
^^ .

variahles | Testing point: Solving for absolute errors.


values
IS. Level: Medium | Domain; ALGEBRA
-
Key Explanation: To solve fur this absolute SLill/ KnOwlcdgc: Linear inequalities in one or
value, create Iwu cqua lions by ituJtiplying the Lwovariables | Testing point: Graphing linear
contents of the absolute value with +1 and - 1 .
inequalities
Tire first equation will become 5| = f
'
7-
Simplifying it yields 3x - 5 = 7- Key Explanation: Choice C is correct . Tn find
Adding - 5 to both sides of the equation yields 3x the correct system of inequalities, focus first or
- 12 , the graph with a solid line. Using points ( 0, 4)
4-0
Dividing both sides of the equation by 3 yields
JC = 4.
and ( 10, 0 ) , the slope of this line is
0 -10 — -0.4.
binding the second solution yields |3x - 5|= *7.
' Since Choices A and C have inequalities with

Simplifying the equatiun yields -3x + 5 = 7. the slope of -0.4, then one uf them mighL be the
Subtracting 5 from both sides of the equation curreet answer.

yields -3 x = 2. In the second graph with the dashed line, the


Dividing both sides of the equation by -3 yields shading is below it. Hence, the symbol used is < .
-2 Therefore, Choice C is the correct answer,
x= —3
Therefore, 4 is the positive solution to the Distractor Explanations: Choice A is incorrect.
absolute value. The graph of the second inequality is shaded
above the line. Choices B and D are incorrect
These options are incorrect as both equations
17. Level: Easy | Domain: PROBLEM SOLVING
have positive gradients.
AND DATA ANALYSIS
Skill / Knowledge: Probability and conditional
probability | Testing point: Probability 19, Level : Easy|Domai n ; A DVANCED M ATH
Skill / Knowledge: Equivalent Expressions
Key Explanation: Choice A is Testing point: Exponents
correct. Probability is equivalent to

.
wwW vibriintpublisbCfS-CiNtl 221

Telegram: SAT_pdf
I MATH Answers & Explanations Module 2
I
Key Explanation: Since a = 2(t, then Key Explanation: Choice R is correct . The line
y - 5 is a horizontal line parallel to the x-axis and
passes through point (0, 5). The line would only
& J pass once through the graph of the nonlinear
7 equation.
Since aLi = 0* , then b = —
15
Distractor Explanations: Choices A , C , and
20. level: Hard | Domain: PROBLEM SOLVING D are incorrect. These options may be due tn
AND DATA ANALYSTS conceptual errors .

Skill ./ Knowledge : Ratios , rates , proportional


relationships, and units | Testing point: Average
22 . Level: Hard | Domain: ALGEBRA
Speed
Ski! I / Knowledge: Linear inequalities in one or

Key Explanation : Choice D is correct Average


.
two variable | Testing point: Applying linear
Total Distance Travelled irequalities to solve word problem
speed is given by
Total lime taken
,

Key Explanation: Choke C is correct . The least


The distance in the lirsl Iwu hours is (60 / 2 ) -
amount of pens that David can buy is 3 which
120 miles. The distance covered on the rcsL of cost $4.50 . This would mean ihaL David ean
the trip is equivalent lo (50 \ r) = 50bc miles, spend at most $ 36 - $4.5 = $.31.50 on books. If *
where x is the number of hours it took to finish is the number of bonks , then 5x < 31 -50-

the trip. Total distance traveled = 120 + 5flx Dividirgboth sides of the inequality by 5 yields
miles. Total time taken - 2 + x hours. Therefore, x < 6- 3- Therefore, he van get a maximum of 6
120 + 50* books without exceeding his budget .
— 54 .
2+x
Multiplying both sides of the equation by 2 + x Distractor Explanations: Choices A and B are
yields 120 + 50* = 103 + 54r. incorrect . David can buy 4 or 5 books however
Subtracting 103 and 50JC from both sides of the this would not be the maximum number of
equation yields 12 = 4x. books he can get - Choice D is incorrect This
Dividing both sides of the equation by 4 yields 3 exceed the maximum number of books David
= xorx = 3, Therefore, the total time taken = 2 + can get without spending above $36,
3 = 5 hours.

Distractor Explanations; Choices A , R and


C are incorrect , These options may be due to
calculation or conceptual errors.

21 . Level: Easy | Domain: ADVANCED MATH


Skill/ Knowledge; Nonlinear functions | Testing
point: Systems of linear and nonlinear Functions

Digital SAT Math Practice Questions 222

Telegram: SAT_pdf
Telegram: SAT_pdf

You might also like